You are on page 1of 131

CRIMINAL LAW I

VI. COURSE OUTLINE


I. Fundamental Principles
A. Definition of Criminal Law
- Oh vs. Court of Appeals, 403 SCRA 300 (2003)
B. Kinds of crimes
1. Mala in se
- U.S. vs. Go Chico, 14 Phil. 128, 131 (1909)
2. Mala prohibita
- U.S. vs. Go Chico, 14 Phil. 128, 131 (1909)
- Lim vs. People, 340 SCRA 497 (2000)
- People vs. Lacerna, 278 SCRA 561 (1997)
Lack of intent as a defense in mala prohibita
- Garcia vs. Court of Appeals, 484 SCRA 617 (2006)
- Ysidoro vs. People, 685 SCRA 637
3. Felony
- Art. 3, RPC - People vs. Gonzales, 183 SCRA 309
(1990)
4. Offense
Distinction between felony and offense.
C. Sources of Philippine Criminal Law
1. Revised Penal Code (Act No. 3815) 2. Special
penal laws passed by Congress 3. Presidential
Decrees
D. Theories of Criminal Law
1. Classical Theory 2.
Positivist Theory 3.
Relative 4. Absolute
E. Power to Define and Punish Crimes
- U.S. vs. Pablo, 35 Phil. 94 (1916)
- People vs. Santiago, 43 Phil. 120 (1922)
F. Constitutional Limitations on Legislative Power to Enact Penal
Laws 1. Equal protection
1987 Constitution, Art. III, Sec. 1
2. Due process
- 1987 Constitution, Art. III, Sec. 14(1)
3. Cruel and unusual punishment
- 1987 Constitution, Art. III, Sec. 19(1) - R.A. No.
9346
4. Bill of attainder/Ex post facto law
- 1987 Constitution, Art. III, Sec. 22
- People vs. Villaraza, 81 SCRA 95 (1978)
G. Characteristics of Criminal Law
1. Prospective
- People vs. Tamayo, 61 Phil. 225
Effect of repeal of penal law
- People vs. Pastor, 77 Phil. 1000
- People vs. Almuete, 69 SCRA 410
2. General
- Civil Code of the Philippines, Art. 14 - U.S. vs.
Sweet, 1 Phil. 18
3. Territorial
- Art. 2, RPC - Reagan vs. Commissioner, 30 SCRA 968
(1969)

H. Construction of Penal Laws

- U.S. vs. Abad Santos, 36 Phil. 243 (1917)

No. 12262. February 10, 1917


THE UNITED STATES, plaintiff and appellee, vs.
ANTONIO ABAD SANTOS, defendant and appellant.
MORELAND, J.

FACTS:
ANTONIO ABAD SANTOS is the owner of a Printing establishment called, “The
Excelsior” and as such was required by the law that he should make the entries required by
Internal Revenue Law, Circular No.467, issued by the Collector of Internal Revenue; Circular
no.467 orders the printers, publishers, contractors, common carriers, etc. to keep a day book in
which he shall enter detail written in English or Spanish the amount of money received in the
conduct of his business and shall present it to the internal revenue agent or officer for approval.

The appellant failed to make any entry for the 5th Day of January, 1915, violating
Curricular No.467, therefore violating article 185 of Act No. 2339 (now section 2727 of
Administrative code), which penalizes a person who violates any Internal Revenue Law. Thus,
he was convicted and sentenced to pay a fine of P10 for violating the abovementioned Internal
Revenue Law

ISSUE:
Whether or not the court of First Instance of Pampanga err in convicting the appellant in
violation of Circular No. 467 of Internal Revenue Law.

RULING:
Yes, the court of First Instance of Pampanga err in convicting the appellant in violation of
Circular No. 467 of Internal Revenue Law.
The court said, “We are of the opinion that the accused must be acquitted. It appears
undisputed that he regularly employed a bookkeeper who was in complete charge of the book in
which the entries referred to should have been made and that the failure to make the entry
required by law was due to the omission of the bookkeeper of which appellant knew nothing.”
The court emphasized that Criminal statute must be strictly Construed. Further, the court
cited the case of S.U. vs. MADRIGAL, “No person should be brought within their terms who is
not clearly within them, nor should any act be pronounced criminal which is not clearly made so
by the statute.”
The court then held that the conviction is reversed and the accused is acquitted. Costs
are de officio.

- People vs. Gatchalian, 104 Phi. 664 (1958)

People v. Gatchalian / G.R. Nos. L-12011-14 / September 30, 1958


FACTS:
Alfonso Gatchalian was charged with a violation of Section 3 of Republic Act No. 602 when he
mishandled the wages of his employees.
The trial court dismissed the case with his counsel citing that the violation was did not constitute
a criminal offense but only civil liability and even if it was the law violated does not carry
any penalty.

ISSUE:

Whether the law at bar carries both criminal and civil liabilities.


RULING:
The Supreme Court points out that the intention of the law is clear. Sec. 18 of RA 602 provides
the penalty of the accused making the offense both civil and criminal.
The SC also stated that even though Sec. 18 was not mentioned in the information, it cannot be
used in deciding. There is no law which requires that in order that an accused may be convicted
the specific provision which penalizes that act charged be mentioned in the information. The SC
found the law clear and unambiguous. They decided that the trial court erred in dismissing the
case and be remanded to the court.  

- People vs. Sultan, 331 SCRA 216 (2000)

PEOPLE OF THE PHILIPPINES vs. FERNANDO SULTAN


G.R. No. 132470 April 27, 2000
FACTS:

On June 2, 1997, around 9:00 in the evening, Juditha Bautista was on her way home from
visiting her cousin when she was accosted by the accused Fernando Sultan. Fernando pointed
a sharp instrument at her neck and told her that this was a hold-up. Fearing for her life, she let
him grab and bring her to his house. Through intimidation and her fear for her life, she was
robbed and twice raped.
After the second rape, he told her he loved her and in her effort to release herself, she “agreed”
to elope with him. Convinced, Fernando let her go home to get her things. She then went to her
cousin, Antonette and narrated everything that happened. Antonette then called her brother
SPO1 Bautista who advised Juditha to continue with the elopement so that he and his
companions could stage an arrest. This went successful and Fernando was apprehended.
On June 5, 1997, Fernando was charged with the complex crime of robbery and rape but he
merely brushed this aside as simply sex between consenting adults.

RTC: Found him guilty on the charge against him.

Contention of the accused: There is no convincing proof that he is guilty of the crime charged.

ISSUE:
Whether the additional rape committed by the accused is considered as an aggravating
circumstance?

HELD:

No. The record shows that the prosecution has established that he committed both robbery and
rape with the intent to take personal property of another preceding the rape.
Complaining witness Bautista was raped twice on the occasion of the robbery. In the recent
case of People v. Regala, the Court held that the additional rapes committed should not be
appreciated as an aggravating circumstance despite a resultant "anomalous situation" wherein
robbery with rape would be on the same level as robbery with multiple rapes in terms of gravity.
The Court realized that there was no law providing for the additional rape/s or homicide/s for
that matter to be considered as aggravating circumstance. It further observed that the
enumeration of aggravating circumstances under Art. 14 of the Revised Penal Code is
exclusive, unlike in Art. 13 of the same Code which enumerates the mitigating circumstances
where analogous circumstances may be considered, hence, the remedy lies with the legislature.
Consequently, unless and until a law is passed providing that the additional rape/s or homicide/s
may be considered aggravating, the Court must construe the penal law in favor of the offender.
Under this view, the additional rape committed by accused-appellant is not considered an
aggravating circumstance. Applying Art. 63, par. (2), of the Revised Penal Code which provides
that "[i]n all cases in which the law prescribes a penalty composed of two indivisible penalties,
the following rules shall be observed in the application thereof . . . . 2. [w]hen there are neither
mitigating nor aggravating circumstances in the commission of the deed, the lesser penalty shall
be applied," the lower penalty of reclusion perpetua should be imposed on accused-appellant.
● Article 2.
Art. 2. Application of its provisions. — Except as provided in the treaties and laws of preferential
application, the provisions of this Code shall be enforced not only within the Philippine
Archipelago, including its atmosphere, its interior waters and maritime zone, but also outside of
its jurisdiction, against those who:
1. Should commit an offense while on a Philippine ship or airship
2. Should forge or counterfeit any coin or currency note of the Philippine Islands or obligations
and securities issued by the Government of the Philippine Islands; chan robles virtual law library
3. Should be liable for acts connected with the introduction into these islands of the obligations
and securities mentioned in the presiding number;
4. While being public officers or employees, should commit an offense in the exercise of their
functions; or
5. Should commit any of the crimes against national security and the law of nations, defined in
Title One of Book Two of this Code.

- Go vs. Dimagiba, 460 SCRA 451 (2005)

Go vs. Dimagiba, 460 SCRA 415 (2005)


SUSAN GO v. FERNANDO L. DIMAGIBA, GR NO. 151876, 2005-06-21

Facts:

Respondent Fernando L. Dimagiba issued to Petitioner Susan Go thirteen (13) checks which,
when presented to the drawee bank for encashment or payment on the due dates, were
dishonored for the reason "account closed.
Dimagiba was subsequently prosecuted for 13 counts of violation of BP 22
On February 27, 2001, Dimagiba filed a Motion for Reconsideration of the MTCC Order. He
prayed for the recall of the Order of Arrest and the modification of the final Decision, arguing
that the penalty of fine only, instead of imprisonment also, should have been imposed on... him.
The RTC held that this rule should be retroactively applied in favor of Dimagiba.[23] It further
noted that (1) he was a first-time... offender and an employer of at least 200 workers who would
be displaced as a result of his imprisonment; and (2) the civil liability had already been satisfied
through the levy of his properties.[24]... questions of law
Issues:
whether the Petition for habeas corpus was validly granted
Ruling:

The issue of retroactivity of SC-AC No. 12-2000 was settled in De Joya v. Jail Warden
In the present case, the Petition for a writ of habeas corpus was anchored on the ruling in Vaca
and on SC-AC No. 12-2000, which allegedly prescribed the imposition of a fine, not
imprisonment, for convictions under BP 22.
Respondent sought the retroactive... effect of those rulings, thereby effectively challenging the
penalty imposed on him for being excessive.
The following alternative penalties are imposable under BP 22: (1) imprisonment of not less
than 30 days, but not more than one year; (2) a fine of not less or more than double the amount
of the check, a fine that shall in no case exceed P200,000; or (3) both such fine and...
imprisonment, at the discretion of the court
When the circumstances of the case clearly indicate good faith or a clear mistake of fact...
without taint of negligence, the imposition of a fine alone may be considered as the preferred
penalty.
Should the... judge deem that imprisonment is appropriate, such penalty may be imposed
SC-AC No. 12-2000 did not delete the alternative penalty of imprisonment. The competence to
amend the law belongs to the legislature, not to this Court.
Petitioners argue that respondent is not entitled to the benevolent policy enunciated in SC-AC
No. 12-2000, because he is not a "first time offender... t is the trial court's discretion to impose
any penalty within the confines of the law. SC-AC No. 13-2001 explains thus:... good faith or a
clear mistake of fact... without taint of negligence, the imposition of a fine alone should be
considered as the more appropriate penalty. Needless to say, the determination of whether the
circumstances warrant the imposition of a fine alone rests solely upon the Judge.
The Judges concerned, may in the exercise of sound discretion, and taking into consideration
the peculiar circumstances of each case, determine whether the imposition of a fine alone would
best serve the interests of justice, or whether forbearing to impose imprisonment... would
depreciate the seriousness of the offense, work violence on the social order, or otherwise be
contrary to the imperatives of justice.

The Court notes that the Petition for a writ of habeas corpus relied mainly on the alleged
retroactivity of SC-AC No. 12-2000, which supposedly favored BP 22 offenders
The rule on retroactivity states that criminal laws may be applied retroactively if favorable to the
accused.

SC Admin. Circular No. 12-2000 is not a penal law; hence, Article 22 of the Revised Penal Code
is not applicable.
The circular applies only to those cases pending as of the date of its effectivity and not to cases
already terminated by final judgment.
SC Admin. Circular No. 12-2000 merely lays down a rule of preference in the application of the
penalties for violation of B.P. Blg. 22. It does not amend B.P. Blg. 22, nor defeat the legislative
intent behind... the law. SC Admin. Circular No. 12-2000 merely urges the courts to take into
account not only the purpose of the law but also the circumstances of the accused -- whether he
acted in good faith or on a clear mistake of fact without taint of negligence -- and such other...
circumstance which the trial court or the appellate court believes relevant to the penalty to be
imposed.

Because the Circular merely lays down a rule of preference, it serves only as a guideline for the
trial courts. Thus, it is addressed to the judges, who are directed to consider the factual
circumstances of each case prior to imposing the appropriate penalty.
In... other words, the Administrative Circular does not confer any new right in favor of the
accused, much less those convicted by final judgment.
Indeed, SC-AC No. 12-2000 necessarily requires a review of all factual circumstances of each
case. Such a review can no longer be done if the judgment has become final and executory.
Hence, RTC-Branch 5 did not have the jurisdiction to modify the lawful judgment in the guise of
granting a writ of habeas corpus.
Respondent seeks the retroactive application of SC-AC No. 12-2000 in his favor on the basis
alone of the alleged settlement of his civil liability.[
WHEREFORE, the Petition is GRANTED and the assailed Orders NULLIFIED.
Respondent's Petition for habeas corpus is hereby DENIED.

Principles:
writ of habeas corpus... required the imposition of a fine only... if the accused was not a
recidivist or a habitual delinquent.
The writ of habeas corpus applies to all cases of illegal confinement or detention in which
individuals are deprived of liberty.
The writ may not be availed of when the person in custody is under a judicial process or by
virtue of a valid judgment
However, as a post-conviction remedy, it may be allowed when, as a consequence of a judicial
proceeding, any of the following... exceptional circumstances is attendant: (1) there has been a
deprivation of a constitutional right resulting in the restraint of a person; (2) the court had no
jurisdiction to impose the sentence; or (3) the imposed penalty has been excessive, thus voiding
the... sentence as to such excess... it is merely... a rule of preference as to which penalty should
be imposed under the peculiar circumstances of a case.
The law punishes the act not as an offense against property but an offense against public
order."

Doctrine of Pro Reo


- People vs. Gonzales, 361 SCRA 350 (2001)

G.R. Nos. 113255-56  July 19, 2001


Lessons Applicable: Pro reo doctrine, indeterminate sentence law, buy-bust operation, buy-bust
operation.
Laws Applicable: indeterminate sentence law

FACTS:
 Early February 1991: the police received an information that Romeo Gonzales was
selling large quantities of marijuana. 
 February 13, 1991: After 4 days of surveillance, they conducted a buy-bust entrapment
operation.  Their informant introduced Sgt. Ortiz to Gonzales as a buyer (poseur-buyer)
of 1 kg. marijuana for P1,200. Then, Ortiz took out his handkerchief as a pre-arranged
signal so the team immediately rushed to the scene introducing themselves as Narcom
agents and arrested Gonzales.  Sgt. Ortiz handed over the bag of marijuana to Pfc. Danilo
Cruz.
 The team confiscated 1 more bag containing 2 blocks of marijuana weighing about 1.5 kg
and 10 medium size plastic bags containing 300 grams of marijuana. The tests yielded
positive indications for the presence of tetrahydrocannabinol, or THC
 Gonzales orally admitted that he was selling marijuana to different buyers, but claimed
that somebody else owned the marijuana he sold.  When asked to identify the owner, he
kept silent.
 2 information charging Gonzales with violation of RA 6425:
Crim. Case No. 91-180: possession, custody and control of 2 block size of marijuana weighing (1.5
kilos) and 10 medium size plastic bags of dry marijuana weighing (300 grams)
Crim. Case No. 91-181: selling more or less 1 kilo of high-grade marijuana
 RTC: Romeo Gonzales guilty for Violation of Sections 8 and 4, Art. II., RA 6425 and
imposes penalty of imprisonment of 6 years and 1 day and a fine of P6,000 for Criminal
Case No. 91-180 life imprisonment and a fine of P20,000 for Criminal Case No. 91-181.
 Gonzales: Victim of a frame-up since he was inside the comfort room of a neighbor from
whom he borrowed P100 to buy medicines for his sick mother and he was just wearing
underwear when he was brought out of the house.  – NOT proven
 A buy-bust operation, normally preceded by surveillance, is an effective mode of
apprehending drug pushers and, “if carried out with due regard to constitutional and legal
safeguards, it deserves judicial sanction.” A warrant of arrest is not essential because the
violator is caught in flagrante delicto.  Searches made incidental thereto are valid.

ISSUE: Whether or not the Indeterminate Sentence Law should apply to Crim. Case No. 91-180

HELD: YES. AFFIRMED with MODIFICATION.  In Criminal Case No. 91-181, life
imprisonment and fine of P20,000.  In Criminal Case No. 91-180, indeterminate penalty of 2
years and 4 months of prison correctional, as minimum, to 8 years and 1 day of prison mayor, as
maximum, and to pay a fine of P6,000.

 The Dangerous Drugs Act, Sec. 8 (special law) prescribes as penalty for possession of
Indian hemp (marijuana), regardless of amount, an imprisonment of 6 years and 1 day to
12 years, and a fine of P6,000 to P12,000.  Applying the pro reo doctrine in criminal law
(when in doubt favor the accused), we hold that the penalty prescribed in R. A. No. 6425,
Section 8 while not using the nomenclature of the penalties under the RPC is actually
prison mayor.  Consequently, it is the first part of Section 1 of the Indeterminate Sentence
Law, which shall apply in imposing the indeterminate sentence.

- Bongalon vs. People, 694 SCRA 12 (2013)


Bongalon v. People of the Philippines
G.R. no. 169533, March 20,2013

 
Facts:

Petitioner was convicted for the crime of child abuse under Section 10 (a) of Republic Act 7610,
for striking a twelve-year-old child, named Jayson, by hitting the latter at
the back, slapping his left cheek and uttering derogatory remarks to the child’s family.

The child joined a procession of Sto. Niño with his brother, and when they passed by the
house of the petitioner, the latter’s daughters threw stones at Jayson and uttering the words RTC
ordered the imprisonment of the petitioner for 6y and 1d to 8y of prision mayor in its minimum.
CA affirmed the conviction but modified the penalty to 4y 2m and 1d of prision correccional, as
minimum to 6y 8m and 1d of prision mayor as maximum. Petitioner then filed a petition for
certiorari under Rule 65. He’s defense was, we was just merely protecting her two minor
daughters.
Issue:
Whether Petitioner’s acts constitute “Child Abuse”
 
Held:
Supreme Court agreed the factual findings of RTC and CA that petitioner struck Jayson at the back
with his hand and slapped Jayson’s face, but disagreed with their holding that his acts constituted
child abuse within the purview of Section 10 (a) of RA 7610.The records did not establish
beyond reasonable doubt that his laying of hands on Jayson had been intended to debase the
"intrinsic worth and dignity" of Jayson as a human being, or that he had thereby intended to
humiliate or embarrass Jayson. The records showed the laying of hands on Jayson to have been
done at the spur of the moment and in anger, indicative of his being then overwhelmed by his
fatherly concern for the personal safety of his own minor daughters who had just suffered harm
at the hands of Jayson and Roldan. With the loss ofhis self-control, he lacked that specific intent
to debase, degrade or demean the intrinsic worth and dignity of a child as a human being that
was so essential in the crime of child abuse.

II. Preliminary Title


Art. 1
Art. 2 - What Constitutes Territory of the Philippines
- Art. 1, 1987 Constitution
- Art. 14, Civil Code of the Philippines
- US Bases Agreement
- RP-US Visiting Forces Accord
- R.A. No. 75
- Arts. 1, 29, 31, 1961 Vienna Convention on Diplomatic Relations
- Sec. 58, R.A. No. 9372 (Human Security Act of 2007 or Anti- Terrorism Act)
- R.A. No. 7055

- Minucher vs. Court of Appeals, G.R. No. 142396. February 11, 2003]

G.R. No. 142396. February 11, 2003


KHOSROW MINUCHER, petitioner, Vs HON. COURT OF APPEALS and ARTHUR SCALZO,
respondents

Ponente: Vitug, J.

Facts:

This is a petition of Khosrow Minucher for review on certiorari of a decision of the Court of
Appeals. Khosrow Minucher is an Iranian national. When the Shah of Iran was deposed,
Minucher became a refugee of United Nations and continued to stay in the Philippines. He
headed the Iranian National Resistance Movement in the Philippines. Arthur Scalzo is a worker
of US Embassy in the Philippines, as a special agent of the Drug Enforcement Administration,
Department of Justice, of the United States.

Violation of the “Dangerous Drugs Act of 1972”, was filed against Minucher following a “buy-bust
operation” conducted by Philippine police narcotics agents accompanied by Scalzo in the house
of Minucher. Where heroin was said to have been seized. The arrest of Minucher as a heroin
trafficker had been well publicized throughout the world. Minucher was indentified as an
international drug trafficker.

Minucher was later ACQUITED by the court

Minucher later on filed for damages due to trumped-up charges of drug trafficking made by
Scalzo.

Scalzo counterclaimed that he had acted in the dischatge if his official duties as being merely an
agent of the Drug Enforcement Administration of the United States Department of Justice.
Scalzo subsequently filed a motion to dismiss the complaint on the ground that, being a special
agent of the United States Drug Enforcement Administration, he was entitled to diplomatic
immunity.

Scalzo attached to his motion a Diplomatic Note of the United States Embassy addressed to
Department of Justice of the Philippines and a Certification of Vice Consul Donna Woodward,
certifying that the note is a true and faithful copy of its original.

The Trial Court DENIED the motion to dismiss.

Issue:

Whether or not Arthur Scalzo is indeed entitled to diplomatic immunity.

Ruling:

On the foregoing premises, the petition is DENIED. No costs.

YES. A forgoing agent, operating with a territory, can be cloaked with immunity from suit as long
as it can be established that he is acting within the directives of the sending state.

The consent or imprimatur of the Philippine Government to the activities of the United States
Drug Enforcement Agency, however, can be gleaned from the undisputable facts in the case.

The official exchange of communication between agencies of the government of the two
countries.

Certification from officials of both the Philippine Department of Foreign Affairs and the United
States Embassy.

Participation of members of the Philippine Narcotics Command in the “buy-bust operation”


conducted at the residence of Minucher at the behest of Scalzo.

These may be inadequate to support the “diplomatic status” of the latter but they give enough
indication that the Philippine government has given its imprimatur, if not consent, to the activities
within Philippine territory of agent Scalzo of the United States Drug Enforcement Agency.

The job description of Scalzo has tasked him to conduct surveillance on suspected drug
supplier and, after having ascertained the target, to inform local law enforcers who would then
be expected to make the arrest.

In conducting surveillance activities on Minucher, later acting as the poseur-buyer during the
buy-bust operation, and then becoming principal witness in the criminal case against Minucher,
agent Scalzo hardly can be said to have acted beyond the scope of his official function or
duties.

- U.S. vs. Bull, 15 Phil. 7

G.R. No. L-5270 January 15, 1910


THE UNITED STATES, plaintiff-appellee, vs. H. N. BULL, defendant-appellant.

Facts:

On December 2, 1908, a steamship vessel engaged in the transport of animals named Stanford
commanded by H.N. Bull docked in the port of Manila, Philippines. It was found that said vessel
from Ampieng, Formosa carried 674 heads of cattle without providing appropriate shelter and
proper suitable means for securing the animals which resulted for most of the animals to get
hurt and others to have died while in transit.

This cruelty to animals is said to be contrary to Acts No. 55 and No. 275 of the Philippine
Constitution. It is however contended that cases cannot be filed because neither was it said that
the court sitting where the animals were disembarked would take jurisdiction, nor did it say
about ships not licensed under Philippine laws, like the ships involved.

Issue:

Whether or not the court had jurisdiction over an offense committed on board a foreign ship
while inside the territorial waters of the Philippines.

Held:

Yes. When the vessel comes within 3 miles from the headlines which embrace the entrance of
Manila Bay, the vessel is within territorial waters and thus, the laws of the Philippines shall
apply. A crime committed on board a Norwegian merchant vessel sailing to the Philippines is
within the jurisdiction of the courts of the Philippines if the illegal conditions existed during the
time the ship was within the territorial waters - regardless of the fact that the same conditions
existed when the ship settled from the foreign port and while it was on the high seas,

RULING:

In light of the above restriction, the defendant was found guilty and sentenced to pay a fine of
two hundred and fifty pesos with subsidiary imprisonment in case of insolvency, and to pay the
costs.

- People vs. Livara, 94 Phil. 771

THE PEOPLE OF THE PHILIPPINES, plaintiff and appellee,


vs. FELIPE A. LIVARA, defendant and appellant Ponente Bengzon. J

Facts:

Felipe A. Livera was a provincial disbursing officer of the Constabulary in Roblon. As finance
and accountable officer, he took charge of paying the salaries and subsistence of the PC
officers and enlisted men of that region. On July 22, 1948, he came to Manila carrying some
money, and, having secured a Treasury Warrant from the finance officer at Camp Crame for
more than P8,000, he cashed the same. In November, 1948, an examination of his accounts
was conducted by Major Emilio Baldia, Chief of the Cash Examination and Inspection Branch of
the Finance Service, who found him with a net shortage of P9,597 unaccounted for. Days
afterwards, a board of officers was created formally to investigate the appellant. That board
found him accountable for P9,597, and recommended his prosecution before the civil courts for
malversation of public funds. In Felipe
Livera’s defense, he admitted his financial liability but asserted that he had lost the
money in Manila on his way to North Harbor to board a vessel for Romblon. Counsel for the
appellant also contends that the CFI of Romblon had no jurisdiction over the case, arguing that
that the alleged crime of malversation of public funds occurred during the incumbency of the
accused as an officer of the Philippine Constabulary, and questions the constitutionality of Art.
217 of the RPC.

Issue/s:

1. Whether or not the CFI of Romblon has jurisdiction over the case
2. whether or not Art. 217 is unconstitutional

Held:

1. No merit. The civil courts and courts-martial have concurrent jurisdiction over offenses
committed by a member of the Armed Forces in violation of military law and the public law. The
first court to take cognizance of the case does so to the exclusion of the other. The accused-
appellant having been first tried and convicted of the crime by the Court of First Instance of
Romblon he cannot now claim that the criminal action should have been brought before a court-
martial.

2. No merit. There is no constitutional objection to the passage of a law providing that


the presumption of innocence may be overcome by a contrary presumption founded upon the
experience of human conduct, and enacting what evidence shall be sufficient to overcome such
presumption of innocence. Wherefore, as this appellant is guilty of malversation of public funds
and as the penalty imposed on him accords with the law, we hereby affirm the judgment with
costs against him. So ordered.

SILVERIO VALDEZ, petitioner,vs. ANTONIO G. LUCERO, Judge of FirstInstance of Ilocos


Sur, and CELESTINOJIMENEZ, Provincial Warden of IlocosSur, respondents.
G.R. No. L-246 March 27,1946
FACTS:
Silverio Valdez was allegedly a member of are cognized guerrilla and a member of the
United States armed forces in the Philippines but was later absorbed in the Philippine army.
That, said Silverio Valdez, with an intent to kill, did then and there willfully, unlawfully and
feloniously with cruelty, by deliberately and inhumanly augmenting the suffering of one Juan
Ponce, kill the latter with bolo, dagger and other weapons and died instantly. The case was taken
in to the Court of First Instance where in the said petitioner argued that the fiscal had no
authority to file it and that the court acquired no jurisdiction of the defendant. The petitioner also
added that only the military courts may only take the case wherein a person subject to military
law who commits murder in time of war shall suffer death or imprisonment for life.

ISSUE:
Whether the civil courts have jurisdiction to take cognizance of and try the case for murderfiled
against petitioner Silverio Valdez.

HELD:
The Court ruled that that the civil courts of the Commonwealth of the Philippines are NOT
deprived of their jurisdiction over the petitioner herein, but have CONCURRENT
JURISDICTION with the military courts or general courts martial to try and take cognizance of
the case of murder against the petitioner herein. They are not deprived of their Jurisdiction over
murder cases committed by persons subject to military law.

- Gonzales vs. Abaya, 498 SCRA 445

LT. (SG) EUGENE GONZALES vs. GEN. NARCISO ABAYA


G.R. No. 164007
August 10, 2006

FACTS:
On July 27, 2003 at around 1:00 a.m., more than 300 heavily armed junior
officers and enlisted men of the AFP entered the premises of the Oakwood Premier Luxury
Apartments on Ayala Avenue, Makati City. They disarmed the security guards and planted
explosive devices around the building. They declared their withdrawal of support from their
Commander-in-Chief and called for her resignation and her cabinet members.
President Arroyo declared a state of rebellion (Proclamation No. 427) and directed the AFP and
PNP (Gen. Order No. 4) to suppress the rebellion. The negotiators sent by the government
succeeded in convincing a total of 321 soldiers to lay down their arms and defuse the
explosives placed around the premises of the Oakwood Apartments.
The National Bureau of Investigation (NBI) investigated the incident and
recommended that the military personnel involved be charged with coup d’etat. Pursuant to
Article 70 of the Articles of War, respondent General Narciso Abaya ordered the arrest and
detention of the soldiers involved in the Oakwood incident and directed the AFP to conduct its
own separate investigation.
On August 5, 2003, the DOJ filed with the Regional Trial Court (RTC), Makati
City an Information for coup d’etat against those soldiers.
On the same date, respondent Chief of Staff (Letter Order No. 625) created a
Pre-Trial Investigation Panel tasked to determine the propriety of filing with the military tribunal
charges for violations of the Articles of War under Commonwealth Act No. 408, as amended,
against the same military personnel. Specifically, the charges are: (a) violation of Article 63 for
disrespect toward the President, the Secretary of National Defense, etc., (b) violation of Article
64 for disrespect toward a superior officer, (c) violation of Article 67 for mutiny or sedition, (d)
violation of Article 96 for conduct unbecoming an officer and a gentleman, and (e) violation of
Article 97 for conduct prejudicial to good order and military discipline.
Of the original 321 accused, 148 filed an Omnibus Motion praying that the said
trial court assume jurisdiction over all the charges filed with the military tribunal invoking
Republic Act (R.A.) No. 7055.
In its Initial Report, the Pre-Trial Investigation Panel recommended that the military personnel
involved in the Oakwood incident be charged before a general court martial with violations of
Articles 63, 64, 67, 96, and 97 of the Articles of War. In its Final Pre-Trial Investigation Report to
the JAGO, it recommended that, following the "doctrine of absorption," those charged with coup
d’etat before the RTC should not be charged before the military tribunal for violation of the
Articles of War.
The DOJ, after conducting a reinvestigation, found probable cause against only 31 of the 321
accused and filed with the RTC an Amended Information, which the RTC admitted so it dropped
the charge of coup d’etat against the 290 accused.
The RTC issued an Order stating that "all charges before the court martial against the
accused…are hereby declared not service-connected, but rather absorbed and in furtherance of
the alleged crime of coup d’etat." The trial court then proceeded to hear petitioners’ applications
for bail.
Colonel Julius A. Magno(OIC of the JAGO), reviewed the findings of the Pre-Trial Investigation
Panel and recommended that 29 of the officers involved in the Oakwood incident be prosecuted
before a general court martial for violation of Article 96 (conduct unbecoming an officer and a
gentleman) of the Articles of War. The recommendation was approved by the AFP top brass.
Instead of complying, the petitioners filed the instant Petition for Prohibition praying that
respondents be ordered to desist from charging them with violation of Article 96 of the Articles of
War in relation to the Oakwood incident.
Petitioners maintain that since the RTC has made a determination in its Order of February 11,
2004 that the offense for violation of Article 96 (conduct unbecoming an officer and a
gentleman) of the Articles of War is not service-connected, but is absorbed in the crime of coup
d’etat, the military tribunal cannot compel them to submit to its jurisdiction.
ISSUES:
1. Whether or not the Conduct of Unbecoming an Officer (Article 96 of the
Articles of War) is service-connected and therefore cognizable by the court martial.

2. Whether or not the charge of Conduct of Unbecoming an Officer is absorbed in


the crime of coup d’etat.
HELD:
1. Yes. The Conduct of Unbecoming an Officer is service-connected and
therefore cognizable by the court martial. This is expressly provided in Section 1
(second paragraph) of R.A. No. 7055. It bears stressing that the charge against the
petitioners concerns the alleged violation of their solemn oath as officers to defend
the Constitution and the duly-constituted authorities. Such violation allegedly caused
dishonor and disrespect to the military profession. In short, the charge has a bearing
on their professional conduct or behavior as military officers. Equally indicative of the
"service-connected" nature of the offense is the penalty prescribed for the same –
dismissal from the service – imposable only by the military court. Such penalty is
purely disciplinary in character, evidently intended to cleanse the military profession
of misfits and to preserve the stringent standard of military discipline.

2. No. The Conduct of Unbecoming an Officer is not absorbed in the crime of


coup d’etat.
The RTC’s declaration that the act complained of is "not service-connected, but
rather absorbed and in furtherance of the alleged crime of coup d’etat,", practically
amended the law which expressly vests in the court martial the jurisdiction over
"service-connected crimes or offenses." What the law has conferred the court should
not take away. It is only the Constitution or the law that bestows jurisdiction on the
court, tribunal, body or officer over the subject matter or nature of an action which
can do so. And it is only through a constitutional amendment or legislative enactment
that such act can be done. The first and fundamental duty of the courts is merely to
apply the law "as they find it, not as they like it to be." Evidently, such declaration by
the RTC constitutes grave abuse of discretion tantamount to lack or excess of
jurisdiction and is, therefore, void.

The trial court aggravated its error when it justified its ruling by holding that the
charge of Conduct Unbecoming an Officer and a Gentleman is ‘absorbed and in
furtherance to the alleged crime of coup d’etat.’ Firstly, the doctrine of ‘absorption of
crimes’ is peculiar to criminal law and generally applies to crimes punished by the
same statute, unlike here where different statutes are involved. Secondly, the
doctrine applies only if the trial court has jurisdiction over both offenses. Here,
Section 1 of R.A. 7055 deprives civil courts of jurisdiction over service-connected
offenses, including Article 96 of the Articles of War. Thus, the doctrine of absorption
of crimes is not applicable to this case.

ARTICLE 3.

Art. 3. Definitions. — Acts and omissions punishable by law are felonies (delitos). Felonies are
committed not only be means of deceit (dolo) but also by means of fault (culpa).

There is deceit when the act is performed with deliberate intent and there is fault when the
wrongful act results from imprudence, negligence, lack of foresight, or lack of skill.

U.S. vs. Look Chaw, 18 Phil. 573

Look Chaw, 18 Phil. 573


G.R. No.L-5887. December 16, 1910.
ARELLANO, C. J.

Lesson: Crimes NOT involving a breach of public order committed on board a public vessel is
NOT triable by our courts

Laws Applicable: Art. 2 RPC, Opium Law

FACTS:

Ø Upon arrival of steamship Erroll of English nationality, that it came from Hongkong, and that it
was bound for Mexico, via the call ports of Manila and Cebu, 2 sacks of opium where found
during the inspection and search of the cargo.
o Smaller sack of opium on the cabin near the saloon
o larger sack in the hold
o Later on, there was also 4 cans of opium found on the part of the ship where the firemen
habitually sleep
§ the firemen and crew of foreign vessels, pursuant to the instructions he had from the Manila
custom-house, were permitted to retain certain amounts of opium, always provided it should not
be taken shore so it was returned
Ø 2 charges were filed against Look Chaw at the Court of First Instance of Cebu:
o unlawful possession of opium
o unlawful sale of opium
Ø Look Chaw admitted that he had bought these sacks of opium, in Hongkong with the
intention of selling them as contraband in Mexico or Vera Cruz, and that, as his hold had
already been searched several times for opium, he ordered two other Chinamen to keep the
sack.
Ø The court ruled that it did not lack jurisdiction, inasmuch as the crime had been committed
within its district, on the wharf of Cebu. The court sentenced him to5 years imprisonment, to pay
a fine of P10,000, with additional subsidiary imprisonment in case of insolvencyxxx It further
ordered the confiscation, in favor of the Insular Government.

ISSUE: W/N the Philippine court has jurisdiction.

HELD: YES. Modified by reducing the imprisonment and the fine imposed to six months and
P1,000

GR: mere possession of a thing of prohibited use in these Islands, aboard a foreign vessel in
transit, in any of their ports, does NOT constitute a crime triable by the courts of this country, on
account of such vessel being considered as an extension of its own nationality
EX: when the article, whose use is prohibited within the Philippine Islands, in the present case a
can of opium, is landed from the vessel upon Philippine soil, thus committing an open violation
of the laws of the land with respect to which, as it is a violation of the penal law in force at the
place of the commission of the crime, only the court established in that said place itself had
competent jurisdiction, in the absence of an agreement under an international treaty.

- US v. Ah Sing

THE UNITED STATES, plaintiff and appellee, vs. AH SING, defendant and appellant
No. 13005. October 10, 1917
Ponente: MALCOLM, J.:

FACTS:

Defendant, Ah Sing, is a fireman on the steamship Shun Chang, a foreign vessel which
arrived at the port of Cebu from a direct voyage from Saigon. Ah Sing was charged for
possession of eight cans of opium which were found by authorities during a port search. Though
he confessed that he is the owner, he did not confess, however, as to his purpose in buying the
opium. He did not say that it was his intention to import the prohibited drug into the Philippine
Islands. No other evidence direct or indirect, to show that the intention of the accused was to
import illegally this opium into the Philippine Islands, was introduced.

ISSUE:

Whether the defendant is guilty of illegal importation of opium into the Philippine Islands.

RULING:

Yes, it is to be noted that section 4 of Act No. 2381 begins, "Any person who shall
unlawfully import or bring any prohibited drug into the Philippine Islands." "Import" and "bring"
are synonymous terms. The Federal Courts of the United States have held that the mere act of
going into a port, without breaking bulk, is prima facie evidence of importation. (The Mary [U.
S.], 16 Fed. Cas., 932, 933.) And again, the importation is not the making entry of goods at the
custom house, but merely the bringing them into port; and the importation is complete before
entry of the Custom House.

Applying the Opium Law, the Supreme Court expressly held that any person unlawfully
imports or brings any prohibited drug into the Philippine Islands, when the prohibited drug is
found under this person's control on a vessel which has come direct from a foreign country and
is within the jurisdictional limits of the Philippine Islands. In such case, a person is guilty of
illegal importation of the drug unless contrary circumstances exist or the defense proves
otherwise. Applied to the facts herein, it would be absurd to think that the accused was merely
carrying opium back and forth between Saigon and Cebu for the mere pleasure of so doing. It
would likewise be impossible to conceive that the accused needed so large an amount of opium
for his personal use. No better explanation being possible, the logical deduction is that the
defendant intended this opium to be brought into the Philippine Islands. We accordingly find that
there was illegal importation of opium from a foreign country into the Philippine Islands. To
anticipate any possible misunderstanding, let it be said that these statements do not relate to
foreign vessels in transit, a situation not present.

- People v. Wong Cheng

The People of the Philippine Islands V Wong Cheng (alias WONG CHUN)
46 Phil 729

FACTS:

Appellee, Wong Cheng is accused of having illegally smoked opium aboard the merchant
vessel Changsa of English nationality while the vessel was anchored in Manila Bay. Appellant,
People of the Philippine Islands filed an appeal urges the revocation of a demurrer sustained by
the Court of First Instance of Manila presented by the appellee.

ISSUE:

Whether or not the Philippine courts have jurisdiction over a crime committed aboard the said
merchant vessel anchored in our territorial waters.

HELD:

Yes. The crime was committed in the territorial waters thus the Philippine courts have
jurisdiction over the crime herein involved.
The court said smoking opium within the territorial areas of the Philippines is a breach of the
public order because such drug produces its pernicious effects within the territory even if such
crime was committed in a foreign merchant vessel. Therefore, the order appealed from is
revoked and the Court ordered further proceedings

III. Title One: Felonies and Circumstances Which Affect Criminal Liability
A. Chapter One: Felonies
Arts. 3

- People vs. Gonzales, 183 SCRA 309 (1990)

PEOPLE OF THE PHILIPPINES, plaintiff-appelle, vs. FAUSTA GONZALES, AUGUSTO


GONZALES, CUSTODIO GONZALES, SR., CUSTODIO GONZALES, JR., NERIO
GONZALES and ROGELIO LANIDA, accused, CUSTODIO GONZALES, SR., accused-
appellant.

Facts:

On October 31, 1984, the decision of Regional Trial Court of Iloilo, Branch XXXVIII (38) found
all the accused, except Rogelio Lanida who eluded the arrest, guilty beyond reasonable doubt
of the crime of murder as defined under Article 248 of RPC.

They were sentenced “to suffer the penalty of imprisonment of 12 years and 1 day to 17 years
and 4 months of reclusion temporal, to indemnify the heirs of the deceased victim in the amount
of P40,000.00, plus moral damages in the sim of P14,000.00 and to pay the costs. Lloyd
Peñacerreda was the victim, 44, land owner, and a resident of Barangay Aspera, Sara, Iloilo.

All the accused except Rogelio Lanida, who eluded, filed a notice of appeal from the trial court’s
decision. During the pendency of their appeal and before judgment thereon could be rendered
by the Court of Appeals, however, all the accused-appellants, except Custodio Gonzales, Sr.,
withdrew their appeal and chose instead to pursue their respective applications for parole before
the then Ministry, now Department, of Justice, Parole Division.

On October 27, 1987, Court of Appeals rendered decision on the appeal of Custodio Gonzales,
Sr. It modified the appealed decision in that the lone appellant was sentenced to reclusion
perpetua and to indemnify the heirs of Lloyd Peñacerrada in the amount of P30,000.00. In all
other respect, the decision of the trial court was affirmed. Further, on the basis of our ruling in
People vs. Ramos.

Antecedent facts:

Around 9pm on February 1981, the barangay captain of Barangay Tipacla Iloilo was awakened
from his sleep by the spouses Augusto and Fausta Gonzales. Augusto informed Paja that his
wife had just killed their landlord Lloyd Penacerrada and thus would like to surrender to the
authorities. Seeing Augusto still holding the knife allegedly used in the killing and Fausta with
her dress smeared in blood, Paja immediately ordered his nephew to take the spouses to the
police authorities at their municipal hall.
Thus, an investigation was made behind the killing, and two days after the said incident,
Augusto appeared before the police station and voluntarily surrendered for detention and
protective custody for “having been involved” in the killing of Lloyd. He requested that he be
taken in the same headquarters where his wife Fausta was detained. During arraignment, the
spouses entered a plea of ‘not guilty’. Before trial however, Huntoria, who claimed to have
witnessed the killing, presented himself to Nanie Penacerrada—the victim’s widow, and
volunteered to testify for the prosecution.
The autopsy report showed that Dr. Rojas found 16 wounds, 5 of which are fatal because they
penetrated in the internal organs – heart, lungs, and intestine of the Llyod Peñacerrada.
Huntoria testified that he clearly saw all the accused, including the appellant, take turns in
hacking and stabbing Lloyd Peñacerrada.

Issue:

Whether or not Custodio Gonzales, Sr. is guilty of murder, based on the Hustoria’s account
where the prosecution’s case rested.

Ruling: The Decision of the Court of Appeals is REVERSED and SET ASIDE and the appellant
is hereby ACQUITTED. Costs de oficio.

The elements of felonies in general are: (1) there must be an act or omission; (2) the act or
omission must be punishable under the Revised Penal Code; and (3) the act is performed or the
omission incurred by means of deceit or fault.

Article 3 defines how felony is committed—which is either by means of deceit (dolo) or by


means of fault (culpa). Thus there is deceit when the act performed is by deliberate intent, while
there is fault when the wrongful act results from imprudence, negligence, lack of foresight, or
lack of skill.

In this case, while the prosecution accuses and the two lower courts both found that the
appellant has committed felony in Lloyd’s death, there is paucity of proof as to what act was
performed by the appellant. Yet, Huntoria, as earlier emphasized, admitted candidly that he
failed to see who stabbed or hacked the victim. In fact, he does not even know what specific act
was performed in the killing. This lack of specificity then makes the case fall short of the test laid
down by Article 3 of the Revised Penal Code.

Huntoria’s credibility as witness is tarnished by the fact that he only came out eight months
since he allegedly saw the incident. He also failed to explain satisfactorily the reason for his
lo4ng delay in revealing what he allegedly witnessed.

-People v. Hassan

G.R. No. L-68969 January 22, 1988


PEOPLE OF THE PHILIPPINES, petitioner, vs. USMAN HASSAN y AYUN, respondent.
PONENTE: SARMIENTO, J.
Facts:

Usman Hassan was accused of murder for stabbing to death Ramon Pichel, Jr. y Uro, 24,
single, and a resident of Zamboanga City. 3 At the time of his death on July 23,1981, the
deceased was employed as manager of the sand and gravel business of his father. On the
other hand, Hassan was an illiterate, 15-year-old pushcart cargador.

Usman was convicted on the bases of the testimony of a lone eyewitness for the prosecution
and the sloppiness of the investigation conducted by the police investigator, Police Corporal
Rogelio Carpio of the Homicide and Arson Section of the Zamboanga City Police Station, who
also testified for the prosecution

The lone eyewitness for the prosecution is Jose Samson, 24 years old when he testified,
married, and a resident of Zamboanga City. On the day of the killing, he was employed at the
sand and gravel business of the father of the deceased but was jobless at the time of his
examination-in-chief on February 3, 1982.

ISSUE:

Whether or not the accused guilty beyond reasonable doubt by just the statements of lone
eyewitness which is Jose Samson

HELD:

He is not guilty beyond reasonable doubt, Moreover, without any verification done on the part of
the statement of lone eyewitness it undergone the process of confrontation arranged by the
police investigator between the self-proclaimed eyewitness and the accused did violence to the
right of the latter to counsel in all stages of the investigation into the commission of a crime
especially at its most crucial stage — the Identification of the accused. Hence, the
accused/appellant was deprived as due to his status as underserved and poor, as because
cries of justice is merely observed with the absence of listening the side of the accused of
marginalized and poor.

RULING:

WHEREFORE, the decision is hereby REVERSED, and the accused Usman Hassan y Ayun is
ACQUITTED of the crime charged. His release from confinement is hereby Ordered, unless he
is held for another legal cause. With costs de oficio.

- People v. Delos Santos, GR 135919

PEOPLE OF THE PHILIPPINES, appellee, vs. DANNY DELOS SANTOS Y FERNANDEZ,


appellant.
G.R. No. 135919. May 9, 2003
Facts:
On November 6, 1997, in the Municipality of San Jose, Del Monte, Province of Bulacan,
Philippines, appellant Danny Delos Santos Fernandez accused guilty with intent to kill Rod
Flores Juanitas, with evident premeditation, treachery and taking advantage of superior
strength, willfully, unlawfully and feloniously attack, assault and stab, hitting him on the different
parts ofhis body which directly caused his death. That on November 1997, De Leon witnessed
the gruesome killing of Flores, while drinking with three men (Salvador, Tablate & Rainier) in
Sarmiento Homes, San Jose del Monte, Bulacan.

Issue:
Whether or not the testimonies of the witnesses are credible even after two month period

Held:
Yes. The court ruled that the two-month delay is hardly an indicium of a concocted story.
It is but natural for witnesses toavoid being involved in a criminal proceeding particularly when
the crime committed is of such gravity as to show the cruelty ofthe perpetrator. Born of human
experience, the fear of retaliation can have a paralyzing effect to the witnesses. Thus, inPeople
vs.Dacibar, we held that the initial reluctance of witnesses to volunteer information about a
criminal case is of common knowledgeand has been judicially declared as insufficient to affect
credibility, especially when a valid reason exists for such hesitance.Besides, settled is the rule
that positive identification prevails over alibi and denial. Decision is affirmed with modification.

People vs. Castillo, 76 Phil. 72

CA-No. 227, February 01, 1946- ART. 3


THE PEOPLE OF THE PHILIPPINES, PLAINTIFF AND APPELLEE, VS. LEON CASTILLO ET
AL., ACCUSED. NENA TANALEGA RAYMUNDO, APPELLANT

FACTS:

Defendant-appellant Nena Tanalega Raymundo was accused of the crime of frustrated murder
by poisoning through reckless imprudence, in the Court of First Instance of Laguna, under the
following information:

"The undersigned Acting Provincial Fiscal accuses Dr. Leon Castillo and Mrs. Nena Tanalega
Raymundo of the crime of 'FRUSTRATED MURDER BY POISONING THROUGH
RECKLESS IMPRUDENCE,' "That on or about the 18th day of February, 1941, in San Pablo
City, Commonwealth of the Philippines, and within the jurisdiction of this Honorable Court, Mr.
Silvino Belarmino presented for dispensation at the Escudero Drug Store and bought one-third
formula of the following prescription of Dr. Antonio G. Sison, to wit:@

"Sparteinsulphat ................................................................ 1.0


e .......... 0
"Phenobarbital ................................................................ 0.5
........... 0
"Carbromal ................................................................ 5.0
........... 0
"Mx. Div. et pone in cap. No. XV
"Sig.; One capsule once a day

"Dr. Leon Castillo and Mrs. Nena Tanalega Raymundo mutually helped each other prepare and
dispense the said prescription, but instead of mixing the required proportion of Spartein
Sulphate, through carelessness and reckless imprudence, willfully, unlawfully and feloniously,
they mixed and compounded in the formula a toxic dose of Strychnine Sulphate, which is a
poisonous substance, and as a result of such carelessness and imprudence, Mr. Belarmino,
upon taking one capsule of the medicine on the very same day, was poisoned and would have
died, had it not been for causes independent of the will of the accused; that had the act
committed by the accused been intentional it would constitute the crime of frustrated murder.
After the corresponding trial, the Honorable Jose Ma. Paredes, presiding judge of the Court of
First Instance of Laguna, on October 12, 1942, rendered judgment, finding said accused guilty
of the crime of frustrated homicide through reckless imprudence, and sentenced her to suffer
four (4) months of arresto mayor and to pay the costs of the proceedings, at the same time
acquitting her co-accused Dr. Leon Castillo.

ISSUE: WON felony was committed by defendant Nena Tanalega.

RULING:

YES, Art. 3 of the Revised Penal code provides that Felonies are committed not only by means
of deceit (dolo) but also by means of fault. A person who caused an injury, without intention to
cause an evil, shall be held liable for culpable felony.
In the case at Bar, the Court declares that the preparation of one medicine for another, by a
pharmacy clerk, under a false name, whether it be through negligence, accident or mistake, is
punishable under the law.
Defendant and appellant Nena Tanalega Raymundo is found guilty of having prepared one
medicine for another, under a false name, in violation of the provisions of section 751 of the
Revised Administrative Code; and, in accordance with the provisions of section 2676 thereof,
she is hereby sentenced to pay a fine of two hundred pesos (P200), with subsidiary
imprisonment in case of insolvency, and to pay the costs of this instance; and with this
modification the judgment of the Court of Appeals for Southern Luzon is upheld, and the petition
for reconsideration is consequently denied.

People vs. Pacana, 47 Phil. 48

THE PEOPLE OF THE PHILIPPINE ISLANDS, plaintiff-appellee, vs. PEDRO A. PACANA,


defendant-appellant.

Ordinarily, evil intent must unite with an unlawful act for there to be a crime. Actus non facit
reum, nisi mens sit rea. There can be no crime when the criminal mind is wanting. Ignorance or
mistake as to particular facts, honest and real, will, as a general rule, exempt the doer neglect in
the discharge of a duty or indifference to consequences, which is equivalent to a criminal intent.
The element of malicious intent is supplied by the element of negligence and imprudence.

FACTS:

These are five related criminal cases for the crimes of falsification of public documents and
estafa committed by means of falsification of public documents.

1. The charge in the first numbered case against Pedro A. Pacana relates to the falsification
by the accused of minutes of meeting of the provincial board on June 9, 1923, for the alleged
purpose of permitting the district engineer to incur illegal expenses in the reconstruction of a
provincial road.

2. The charge in the second case against the same accused relates to the falsification of
minutes of the provincial board on June 16, 1923.

3. The charge in the third case against the same accused relates to the falsification of an
excerpt from the minutes of the provincial board of June 9, 1923.

4. And the last cases, one against provincial board member Isidro Adorable and Pedro A.
Pacana, and (5)the other against provincial board member Vicente P. Castro and Pedro A.
Pacana, relate to the crimes of estafa committed by means of falsification of public documents,
whereby it is alleged Adorable and Castro were each able to collect the sum of P25 as per
diems for two fictitious meetings of the provincial board. Since the first three cases were tried
together and the last two together, and since the facts of all of them are closely interwoven, for
convenience sake a general statement will first be made, leaving for special mention certain
circumstances affecting particular cases.

ISSUE:

Whether there was an intentional and deliberate falsification of public documents on the part of
the accused. (NO)

RULING:

The whole case impresses as a job bunglingly performed by the provincial secretary. He is a
man who should not be entrusted with official responsibility. He has none of the qualifications
which fit one for public office. But it is a far cry from hopeless ineptitude and hopeless stupidity
to criminal intent and criminal responsibility. Still, even under the most favorable aspect, the
facts skirt perilously near to the Penal Code crime of reckless imprudence.

Ordinarily, evil intent must unite with an unlawful act for there to be a crime. Actus non facit
reum, nisi mens sit rea. There can be no crime when the criminal mind is wanting. Ignorance or
mistake as to particular facts, honest and real, will, as a general rule, exempt the doer neglect in
the discharge of a duty or indifference to consequences, which is equivalent to a criminal intent.
The element of malicious intent is supplied by the element of negligence and imprudence.

It is a serious matter to be responsible for sending the accused to prison for long terms. All
reasonable doubt intended to demonstrate error and not crime should be indulged in to the
benefit of the prisoners at bar. The Government has suffered no loss. If the inculpatory facts and
circumstances are capable of two or more explanations, one of which is consistent with the
innocence of the accused of the crime charged and the other consistent with their guilt, then the
evidence does not fulfill the test of moral certainty and is not sufficient to support a conviction.
Therefore, the SC is constrained to acquit the accused of the charges laid against them.

People vs. Bandian, 63 Phil. 530

PEOPLE vs. BANDIAN G.R. No. 45186 September 30, 1936 Infanticide, Article 12
Exempting Circumstances
NOVEMBER 20, 2017

FACTS:
At About 7 in the morning of January 31, 1936, Valentine Aguilar, the apellant’s
neighbor, saw the appelant go to the thicket about four or five brazas from her house,
apparently to respond to a call of nature because it was there that the people of the
place used to go for that purpose. A few minutes later, he then again saw her emerge
from the thicket with her clothes stained with blood both in front and back, stagerring
and visibly showing signs of not being able to support herself. He ran to her aid and
having noted that she was very weak and dizzy, he supported and helped her go up to
her house and placed her in her bed.
Upon being asked before Aguilar brought her to her house, what had happened to her,
the appellant answered that she was very dizzy. Not wishing to be alone with the
appellant in such circumstances, Aguilar called Adriano Comcom who lived nearby to
be there and help him and the appellant. He asked Comcom to take bamboo leaves to
stop the hemhorrage which had come upon the appellant. Comcom had scarcely gone
about five brazas when he saw the body of newborn baby near a path adjoining the
thicket where the appellant had gone a few moments before. Comcom informed Aguilar
of it and the latter told him to bring the body to the appellant’s house. Upon being askes
whether the babywhich was shown to her was hers or not, the appellant answered in
the affirmative.
In the afternoon of the said day, Dr. Emilio Nepomuceno, president of the sanitary
division went to the appellant’s house and found her still lying in bed still bleeding. In his
opinion, the physician declared that the appellant gave birth in her house, and
afterwhich, he threw the child into the thicket to kill it for the purpose of concealing her
dishonor from the man, Luis Kirol, with whom she was married to, because the child
was not his but with another man with whom she had previously has amorous relations.
Nepomuceno testified that the appellant admitted killing her child.

ISSUE:
What was the crime committed by appellant?

RULING:

The evidence certainly does not show that the appellant , in causing her child’s death in
one way or another, or in abandoning it in the thicket, did so willfully, consciously, or
imprudently. She had no cause to kill or abandon it, to expose it to death , because her
affair with a former lover, Luis Kirol took place three years before the incident. The
husband of the appellant testified at the trial affirming the belief that the child was his.
Infanticide and abandonment of a minor, to be punishable must be committed willfully
and consciously, or at least it must be the result of a voluntary, conscious and free act
or omission. Even in cases where said crimes are committed through mere imprudence,
the person who commits them, under said circumstances, must be in the full enjoyment
of his mental facilities, or must be conscious of his acts, in order that he may be held
liable.
The law exempts from criminal liability any person who acts under the circumstances in
which the appellant acted in this case, by giving birth to a child in the thicket and later
abandoning it, not because of imprudence or any other cause than that she was
overcome by severe dizziness and extreme debility, with no fault or intention on her
part. She has in her favor the fourth and the seventh exempting circumstances.
In conclusion, taking into account the foregoing facts and considerations, and granting
that the appellant was aware of her involuntary childbirth in the thicket and that she later
failed to take her child therefrom, having been so prevented by reason of causes
entirely independent of her will, it should be held that… under such circumstances,
appellant has the fourth and seventh exempting circumstances of article 12 of the
Revised Penal Code in her favor.

- U.S. vs. Ah Chong, 15 Phil. 488

THE UNITED STATES, plaintiff and appellee,


vs. AH CHONG, defendant and appellant
G.R No. 5272. March 19, 1910

Facts:
The defendant, Ah Chong, was employed as a cook in one of the Officers’ quarters at
Fort McKinley, Rizal Province. Together living with him in the said quarters was the deceased,
Pascual Gualberto, who was employed as a houseboy. There had been several robberies in
Fort McKinley prior to the incident thus prompting the defendant and his roommate to reinforce
the flimsy hook used to lock the door of their room by placing a chair against it. The defendant
and the deceased had an understanding that when either returned at night, he should knock on
the door and say his name. On the night of Aug. 14, 1908, Ah Chong, who was alone in his
room, was awakened by someone trying to force open the door of the room. The defendant
called out twice, asking the identity of the person but heard no answer. Fearing that the intruder
was a robber or a thief, the defendant called out that he would kill the intruder if he tried to enter.
At that moment, the door was forced open and the defendant was struck first above the knee by
the edge of the chair. Because of the darkness of the room, the defendant thought he was being
hit by the intruder and tried to defend himself by striking wildly at the intruder using a common
kitchen knife which he kept under his pillow. It turned out that the said intruder was actually the
defendant’s roommate, Pascual Gualberto. The roommate was brought to the military hospital
where he died from the effects of the wound the following day.

Issue:

WON the defendant was criminally liable for committing a felony.


Held:
Defendant was not criminally liable and exonerated. In order for mistake of fact to be
held as a valid defense, there has to be several requisites. One, that the act done would have
been lawful had the facts been as the accused believed them to be. Two, that the intention of
the accused in performing the act should be lawful, and lastly, that the mistake must be without
fault or carelessness on the part of the accused. I n the case at bar, had the intruder been a
robber as the defendant believed him to be, then Ah Chong acted in good faith, without malice
or criminal intent, and would have been wholly exempt from criminal liability and that he cannot
be said to have been guilty of negligence or recklessness.

- U.S. vs. Apego, 23 Phil. 391

People vs. Oanis, 74 Phil. 257

G.R. No. L-47722 July 27, 1943


THE PEOPLE OF THE PHILIPPINES, plaintiff-appellee, vs. ANTONIO Z. OANIS and
ALBERTO GALANTA, defendants-appellants.

Antonio Z. Oanis in his own behalf.


Maximo L. Valenzuela for appellant Galanta.
Acting Solicitor-General Ibañez and Assistant Attorney Torres for appellee.
(ART. 3 – FELONIES (MISTAKE OF FACT))

Facts:

Chief of police Antonio Oanis and Alberto Galanta were instructed to arrest a notorious
criminal and escaped convict, Anselmo Balagtas, and if overpowered, to get him dead
or alive. Proceeding to the suspect’s house, they went to a room and on seeing a man
sleeping with his back towards the door where they were, simultaneously or
successively fired at him without making any reasonable inquiry as to his identity which
resulted to the victim’s death. The supposedly Balagtas turned out to be Serepio
Tecson, an innocent man. During the trial, the accused invoked the Ah Chong case of
the fulfillment of their duty.

Issue:

Whether or not they may, upon such fact, be held responsible for the death thus caused
to Tecson.

Ruling:

No. The accused are guilty of murder. Ah Chong case does not apply to the case at bar.
Mistake of the fact applies only when the mistake is committed without fault or
carelessness. The person in the room being then asleep, Oanis and Galanta had ample
time and opportunity to ascertain his identity without hazard to themselves, and could
even effect a bloodless arrest if any reasonable effort to that end had been made, as
the victim was unarmed.

Other Version/ Case Digest

Oanis, 74 Phil. 257


G.R. No.L-47722 July 27, 1943
MORAN, J.

Lesson applicable: mitigating circumstances

FACTS:

Captain Godofredo Monsod, Constabulary Provincial Inspector at Cabanatuan, Nueva


Ecija, received from Major Guido a telegram of the following tenor: "Information received
escaped convict Anselmo Balagtas with bailarina and Irene in Cabanatuan get him dead or
alive." Captain Monsod accordingly called for his first sergeant and asked that he be given four
men.

The same instruction was given to the chief of police Oanis who was likewise called by
the Provincial Inspector.

Defendants Oanis and Galanta then went to the room of Irene, and an seeing a man
sleeping with his back towards the door where they were, simultaneously or successively fired
at him with their .32 and .45 caliber revolvers. Awakened by the gunshots, Irene saw her
paramour already wounded, and looking at the door where the shots came, she saw the
defendants still firing at him. Shocked by the entire scene. Irene fainted; it turned out later that
the person shot and killed was not the notorious criminal Anselmo Balagtas but a peaceful and
innocent citizen named Serapio Tecson, Irene's paramour.

According to Appellant Galanta, when he and chief of police Oanis arrived at the house,
the latter asked Brigida where Irene's room was. Brigida indicated the place, and upon further
inquiry as to the whereabouts of Anselmo Balagtas, she said that he too was sleeping in the
same room.

ISSUE: W/N they may, upon such fact, be held responsible for the death thus caused to Tecson

HELD: appellants are hereby declared guilty of murder with the mitigating circumstance

YES. Ignorantia facti excusat, but this applies only when the mistake is committed without fault
or carelessness appellants found no circumstances whatsoever which would press them to
immediate action. The person in the room being then asleep, appellants had ample time and
opportunity to ascertain his identity without hazard to themselves, and could even effect a
bloodless arrest if any reasonable effort to that end had been made, as the victim was unarmed.
"No unnecessary or unreasonable force shall be used in making an arrest, and the
person arrested shall not be subject to any greater restraint than is necessary for his detention."

a peace officer cannot claim exemption from criminal liability if he uses unnecessary
force or violence in making an arrest

The crime committed by appellants is not merely criminal negligence, the killing being
intentional and not accidental. In criminal negligence, the injury caused to another should be
unintentional, it being simply the incident of another act performed without malice.

2 requisites in order that the circumstance may be taken as a justifying one:

1. offender acted in the performance of a duty or in the lawful exercise of a right-present

2. injury or offense committed be the necessary consequence of the due performance of


such duty or the lawful exercise of such right or office.-not present

According to article 69 of the Revised Penal Code, the penalty lower by 1 or 2 degrees
than that prescribed by law shall, in such case, be imposed.

- U.S. vs. Barnes, 12 Phil. 93

US V A.H. BARNES G.R. No. 4774 Nov 18, 1908


PONENTE: TORRES, J.

FACTS:

· At about 10 o'clock on the morning of the 15th of December, 1906, A. H. Barnes, J. A. Ryan,
and three Filipinos, one of whom was Pedro Leonardo, were out duck shooting and went
through the barrio of San Pablo, sitio of Muyot, in the municipality of San Antonio, Nueva Ecija;

·When they were about to return it was noticed that there were ducks on an estero or stream;
Barnes at once fired his gun twice, the said Pedro Leonardo being near to him at the time;
when trying to reload the weapon, the cartridge would not go in easily, and Barnes had to force
it by closing the breech of the gun which is automatic, pressing the same upon his knee, at
which moment the gun was discharged, Barnes being still on his knees;

·When he rose to look for the ducks he saw the said Leonardo sinking beneath the water, for
which reason he left his gun on the ground and told his friend Ryan that it looked as if the said
individual was hurt; they went into the river and with the assistance of the other men recovered
the body of Pedro Leonardo who was already dead; a surgeon who examined the body found a
gunshot wound in the back of the head; bones had been broken, and the wound was of
necessity a mortal one.

ISSUE:

Whether or not the accused is liable for reckless imprudence for the death of Pedro
Leonardo

HELD:
Based on the commotion, Barnes was not directly pointing towards the duck as he just
pressing it against his knee therefore, imprudence or negligence cannot be derived from the
situation as it should be voluntary. and inasmuch as it has not been proven, even by
circumstantial evidence, that the accused saw or was aware that the deceased Leonardo,
who stood behind him, had moved near him and in front of the muzzle of the gun; the
conclusion to be arrived at must necessarily be that the said death was not the result of a
voluntary and criminal act, nor of an omission or reckless negligence, but an involuntary act
devoid of a criminal character, that is a regretable and unfortunate accident without any
effort of the will.

RULING:

In view of the foregoing, and as the facts proven in the case do not constitute a crime, not even
that of homicide through imprudence, it is our opinion that the judgment appealed from should
be reversed, and that A. H. Barnes should be acquitted and he is hereby acquitted of the
charge, with the costs of both instances de oficio. So ordered

- People v. Sara, 55 Phil. 939

THE PEOPLE OF THE PHILIPPINE ISLANDS, plaintiff and appellee,


vs. FRANCISCO SARA, defendant and appellant
G.R No. 34140. August 15, 1931

Facts:
Francisco Sara, armed with a shotgun, was out in a barrio with the design of shooting
birds. At the same time a couple was out collecting bananas. A witness saw the incident where
the discharge of a gun was heard, and the saw the wife’s husband stretched on the ground as
he was hit in the abdomen. They saw the accused running away carrying a gun.

Issue:
Whether or not there was deliberate intent to kill by the accused.

Ruling:
No, there was no intent although the accused was responsible for homicide.
Homicide resulted from the discharge of the gun in the hands of the accused but the relations
between them were such as to negate any direct intention on his part to do bodily harm.

Dungo v. People, GR 209464

Dungo v. People G.R. No. 209464, July 01, 2015 R. A. No. 8049 Anti-Hazing
Law, Malum Prohibitum

FACTS:
Villanueva, a UP Los Baños student, was a neophyte of the APO – Theta Chapter Fraternity
and that Dungo and Sibal, as members of the said fraternity, together with the other fraternity
members, officers and alumni, brought and transported Villanueva and two other neophytes to
Villa Novaliches Resort at Barangay Pansol, Calamba City, for the final initiation rites
conducted inside the resort, performed under the cover of darkness and secrecy.

Due to the injuries sustained by Villanueva, the fraternity members and the other two neophytes
haphazardly left the resort, while Dungo and Sibal boarded a tricycle and brought the lifeless
body of Villanueva to JP Rizal Hospital, where Villanueva was pronounced dead.

The RTC found Dungo and Sibal guilty of the crime of violating Section 4 of the Anti-Hazing Law
and sentenced them to suffer the penalty of reclusion perpetua.

Upon appeal, the CA ruled that the appeal of Dungo and Sibal was bereft of merit.

ISSUE:

Whether or not the absence of proof of intent to kill the victim will affect the liability of the
accused.

RULING:

The crime of hazing under R.A. No. 8049 is malum prohibitum.

R.A. No. 8049, or the Anti-Hazing Law of 1995, has been enacted to regulate hazing and other
forms of initiation rites in fraternities, sororities, and other organizations. It was in response to
the rising incidents of death of hazing victims.

Criminal law has long divided crimes into acts wrong in themselves called acts mala in se; and
acts which would not be wrong but for the fact that positive law forbids them, called acts mala
prohibita. This distinction is important with reference to the intent with which a wrongful act is
done. The rule on the subject is that in acts mala in se, the intent governs; but in acts mala
prohibita, the only inquiry is, has the law been violated? When an act is illegal, the intent of the

Art. 4

- US v Valdez, 41 Phil 497


US v Valdez, GR No. L-14128 December 10, 2018 – FRUSTRATED ARSON

Facts: The house of a certain Mrs. Lewis was seen to have smoke issuing from its lower floor.
When she ordered Banal to look for the source, it was found out that it came between a post of
the house and a partition of the entresol, a piece of a jute sack and a rag which were burning. At
that moment the defendant Valdes was in the entresol, engaged in his work of cleaning. He was
later on arrested and made a statement that he had set the fire to the said rag and piece of sack
under the house due to the inducement of other prisoners and that he had started the several
other fires which had occurred in said house on previous days.

Issue: WN the accused has committed the crime of frustrated arson?

Ruling: Yes. The fact of setting fire to a jute sack and a rag, soaked with kerosene oil and
placed beside an upright of the house and a partition of the entresol of the building, thus
endangering the burning of the latter, constitutes the crime of frustrated arson of an inhabited
house, on an occasion when some of its inmates were inside of it.

The crime is classified only as frustrated arson, inasmuch as the defendant performed all the
acts conceive to the burning of said house, but nevertheless., owing to causes independent of
his will, the criminal act which he intended was not produced. The offense committed cannot be
classified as consummated arson by the burning of said inhabited house, for the reason that no
part of the building had yet commenced to burn, although, as the piece of sack and the rag,
soaked in kerosene oil, had been placed near partition of the entresol, the partition might have
started to burn, had the fire not been put out on time.

People v. Bindoy, 56 Phil 15

[ GR No. 34665, Aug 28, 1931 ]


PEOPLE v. DONATO BINDOY
DECISION. 56 Phil. 15

VILLAMOR, J.:

The appellant was sentenced by the Court of First In- stance of Occidental Misamis to the
penalty of twelve years and one day of reclusion temporal, with the accessories of law, to
indemnify the heirs of the deceased in the amount of P1,000, and to pay the costs. The crime
charged against the accused is homicide, according to the following information:

"That on or about the 6th of May, 1930, in the barrio of Calunod, municipality of Baliangao,
Province of Occidental Misamis, the accused Donato Bindoy willfully, unlawfully, and
feloniously attacked and with his bolo wounded Emigdio Omamdam, inflicting upon the latter
a serious wound in the chest which caused his instant death, in violation of article 404 of the
Penal Code."

The accused appealed from the judgment of the trial court, and his counsel in this instance
contends that the court erred in finding him guilty beyond a reasonable doubt, and in convicting
him of the crime of homicide.

The record shows that in the afternoon of May 6, 1930, a disturbance arose in a tuba wineshop
in the barrio market of Calunod, municipality of Baliangao, Province of Occidental Misamis,
started by some of the tuba drinkers. There were Faustino Pacas (alias Agaton), and his wife
called Tibay. One Donato Bindoy, who was also there, offered some tuba to Pacas' wife; and
as she refused to drink having already done so, Bindoy threatened to injure her if she did not
accept. There ensued an interchange of words between Tibay and Bindoy, and Pacas stepped
in to defend his wife, attempting to take away from Bindoy the bolo he carried. This
occasioned a disturbance which "attracted the attention of Emigdio Omamdam, who, with his
family, lived near the market. Emigdio left his house to see what was happening* while Bindoy
and Pacas were struggling for the bolo. In the course of this struggle, Bindoy succeeded in
disengaging himself from Pacas, wrenching the bolo from the latter's hand towards the left
behind the accused, with such violence that the point of the bolo reached Emigdio
Omamdam's chest, who was then behind Bindoy.

There is no evidence that Emigdio took part in the fight between Bindoy and Pacas. Neither is
there any indication that the accused was aware of Emigdio Omamdam's presence in the place,
for, according to the testimony of the witnesses, the latter passed behind the combatants when
he left his house to satisfy his curiosity. There was no disagreement or ill feeling between
Bindoy and Omamdam, on the contrary, it appears they were nephew and uncle, respectively,
and were on good terms with each other. Bindoy did not try to wound Pacas, and instead of
wounding him, he hit Omamdam; he was only defending his possession of the bolo, which
Pacas was trying to wrench away from him, and his conduct was perfectly lawful.

The wound which Omamdam received in the chest, judging by the description given by the
sanitary inspector who attended him as he lay dying, tallies with the size of the point of Bindoy's
bolo.

There is no doubt that the latter caused the wound which produced Emigdio Omamdam's death,
but the defendant alleges that it was caused accidentally and without malicious intent.

Pacas and the widow of the deceased, Carmen Angot, testified having seen the accused stab
Omamdam with his bolo. Such testimony is not incompatible with that of the accused, to the
effect that he wounded Omamdam by accident. The widow testified that she knew of her
husband's wound being caused by Bindoy from his statement to her before his death.

The testimony of the witnesses for the prosecution tends to show that the accused stabbed
Omamdam in the chest with his bolo on that occasion. The defendant, indeed, in his effort to
free himself of Pacas, who was endeavoring to wrench his bolo from him, hit Omamdam in the
chest; but, as we have stated, there is no evidence to show that he did so deliberately and with
the intention of committing a crime. If, in his struggle with Pacas, the defendant had attempted
to wound his opponent, and instead of doing so, had wounded Omamdam, he would have had
to answer for his act, since whoever willfully commits a felony or a misdemeanor incurs criminal
liability, although the wrongful act done be different from that which he intended. (Art. 1 of the
Penal Code.) But, as we have said, this is not the case.

The witness for the defense, Gaudencio Cenas, corroborates the defendant to the effect that
Pacas and Bindoy were actually struggling for the possession of the bolo, and that when the
latter let go, the former had pulled so violently that it flew towards his left side, at the very
moment when Emigdio Omamdam came up, who was therefore hit in the chest, without
Donato's seeing him, because Emigdio had passed behind him. The same witness adds that
he went to see Omamdam at his home later, and asked him about his wound when he replied:
"I think I shall die of this wound." And then continued: "Please look after my wife when I die:
See that she doesn't starve,'* adding further: "This wound was an accident. Donato did not aim
at me, nor I at him: It was a mishap." The testimony of this witness was not contradicted by any
rebuttal evidence adduced by the fiscal.

We have searched the record in vain for the motive of this kind, which, had it existed, would
have greatly facilitated the solution of this case. And we deem it well to repeat what this court
said in United States vs. Carlos (15 Phil., 47), to wit:

"The attention of prosecuting officers, and especially of provincial fiscals, directed to the
importance of definitely ascertaining and proving, when possible, the motives which actuated
the commission of a crime under investigation.

"In many criminal cases one of the most important aids in completing the proof of the
commission of the crime by the accused is the introduction of evidence disclosing the motives
which tempted the mind of the guilty person to indulge the criminal act."

In view of the evidence before us, we are of opinion and so hold, that the appellant is entitle to
acquittal according to article 8, No. 8, Penal Code. Wherefore, the judgment appealed from is
reversed, and the accused Donato Bindoy is hereby acquitted with costs de oficio.
So ordered.

Avanceña, C. J., Johnson, Street, Malcolm, Romualdez, Villa-Real, and Imperial, JJ., concur.

- People v. Villacorta, GR 186412

PEOPLE OF THE PHILIPPINES, PLAINTIFF-APPELLEE, VS. ORLITO VILLACORTA,


ACCUSED-APPELLANT.
G.R. No. 186412, September 07, 2011
Ponente: LEONARDO-DE CASTRO, J.

FACTS:

Orlito Villacorta is found guilty beyond reasonable doubt and sentenced to suffer reclusion
perpetua before the Trial Court of Malabon for the murder of Danilo Cruz using a sharpened
bamboo stick, inflicting serious wound which caused his immediate death.

The herein petitioner appealed to assail the said conviction and contended that, in the event he
is found to have indeed stabbed Cruz, he should only be held liable for slight physical injuries
for the proximate cause of Cruz's death is the tetanus infection and not the stab wound that he
inflicted.

ISSUE:

Whether or not the felony committed by Villacorta is the proximate cause of the death of Danilo
Cruz

RULING:

As to the claim that the proximate cause of the death of Danilo Cruz is the act committed by
Villacorta, The Court held that there had been an interval of 22 days between the date of the
stabbing and the date when Cruz was rushed to San Lazaro Hospital, exhibiting symptoms of
severe tetanus infection. If Cruz acquired severe tetanus infection from the stabbing, then the
symptoms would have appeared a lot sooner than 22 days later.
As the Court noted in Urbano v. Intermediate Appellate Court, 157 SCRA 10, severe tetanus
infection has a short incubation period, less than 14 days; and those that exhibit symptoms
within two to three days from the injury, have one hundred percent (100%) mortality.

Ultimately, it can only deduce that Cruz's stab wound was merely the remote cause, and its
subsequent infection with tetanus might have been the proximate cause of Cruz's death. The
infection of Cruz's stab wound by tetanus was an efficient intervening cause later or between
the time Cruz was stabbed to the time of his death.

Thus, the Court set aside the conviction of Villacorta for murder but the herein petitioner is guilty
of the crime of slight physical injuries.

- People vs. Faller, 67 Phil. 629

People vs. Faller


67 Phil 629

FACTS:

Appellant, Restituto Faller was charged with crime of damage caused to another’s property
maliciously and willfully but, the Court of First Instance of Rizal found it reckless imprudence.
The CFI of Rizal sentenced him to pay a penalty of P38.00 and to indemnify the offended party
who is Ramon Diokno with the same amount along with a subsidiary imprisonment in case of
insolvency.
Faller argued that he is punished with a sentence for a crime he was not charged. He claimed
that a crime maliciously and willfully committed is different from that reckless imprudence
committed.
The court has not committed this error. The information alleged that he acted maliciously,
unlawfully, and criminally. The allegation that Faller acted unlawfully and criminally includes the
charge that he acted with negligence.

ISSUE:
Whether or not Faller was wrongly sentenced to a crime he was charged?

HELD:
No, the appellant was convicted of the same crime of damage to property with which he is
charged. Reckless imprudence is not a crime in itself. It is simply a way of committing it and
merely determines a lower degree of criminal liability. Therefore, the judgment was affirmed.

People vs. Acosta, 60 Phil. 158

[G.R NO. 126351. February 18, 2000]


FACTS:
 
Complainant’s grandson, Elmer Montesclaros, in the belief that Acosta and his wife were the
ones hiding his live-in partner from him, stormed the house of appellant and burned their
belongings. In the afternoon of February 27, 1996, witness Aquino saw Acosta and approached
him. When she asked why he was carrying a stove and a knife, he replied that he would burn the
house of complainant. In the morning of February 28, 1996, witness Videña, saw complainant's
house burning. She noticed the presence of appellant standing
alone in front of the burning house. Appellant was just watching the blaze and not doing
anything to contain it. When the fire truck arrived, the house was already razed to the ground.
After the conduct of the investigation, the investigator did not find any incendiary device; hence,
the cause of fire remained undetermined. However, trial court found appellant guilty with the
crime of arson.

ISSUE:
Whether circumstantial evidences can justify appellant’s conviction for the crime of arson.
 
HELD:
YES. Appellant's conviction for the crime of arson rests on circumstantial evidence. Pertinently,
Section 4 of Rule 133 of the Rules of Court provides:

Circumstantial evidence is sufficient for conviction if:


(a) There is more than one circumstance;
(b) The facts from which the inferences are derived are proven;
(c) The combination of all the circumstances is such as to produce a conviction beyond
reasonable doubt.

In order to justify a conviction upon circumstantial evidence, the combination of circumstances


must be such as to leave no reasonable doubt in the mind as to the criminal responsibility of the
accused.
Although there is no direct evidence linking appellant to the arson, SC agreed with the trial court
holding him guilty thereof in the light of the following circumstances duly proved and on record:
 
First, appellant had the motive to commit the arson. It is not absolutely necessary, nevertheless
in a case of arson like the present, the existence or non-existence of a sufficient motive is a fact
affecting the credibility of the witnesses. It was duly proved that at around 4:30 in the afternoon
of February 27, 1996, complainant's grandson, stormed the house of appellant and his wife and
burned their belongings. When appellant was informed of the incident, he got mad, and as his
common-law wife testified, appellant threw a tantrum.

Second, appellant's intent to commit the arson was established by his previous attempt to set on
fire a bed inside the same house (private complainant's) which was burned later in the night.
Witness Aquino testified that at around 5:00 in the afternoon of the same day, she asked
appellant what he was going to do with the stove he was carrying that time, he answered that he
was going to burn the house of private complainant.
 
When she peeped in the kitchen, she saw that appellant entered the house of private complainant
and started pouring gas on a bed and then lighted a fire with a disposable lighter. Appellant's
wife rushed in and extinguished the fire with a broomstick. The two later left the house.

Third  , appellant was not only present at the locus criminis before the incident, he was seen
inside the yard of the burning house during the height of the fire. At around 1:00 in the morning
of February 28, 1996, witness Videña, through the holes of the GI sheets, saw appellant latter
that day standing alone inside private complainant's yard watching the house burning.
 Appellant even looked happy with a canine smile and crazy-looking expression.
Fourth, appellant's actions subsequent to the incident further point to his culpability. At around
12:00 noon of February 28, 1996, private complainant went with witness Videña to the place of
Kagawad Tecson. They whereabout to leave when appellant arrived. Private complainant asked
him why he burned her house and appellant answered, "So what if I burned your house?"

Lastly, it would not be amiss here to point out that "in the crime of arson, the enormity of the
offense is not measured by the value of the property that may be destroyed but rather by the
human lives exposed to destruction."

People vs. Cornel, 78 Phil. 458

[ GR No. L-204, May 16, 1947 ]


PEOPLE v. GERARDO CORNEL +
DECISION J. PARAS
Facts:

· The first feature of appellant's case as presented by his counsel de oficio, refers to the
alleged inadequacy of the evidence for the prosecution establishing appellant's identity. Trinidad
Coral, however, personally saw (1) the appellant suddenly assault her deceased husband
(Fabian Burac) with a bolo as the latter was descending the stairs of his house in the barrio of
San Miguel, municipality of Tabaco, Province of Albay, at about 6 o'clock in the afternoon of
June 8, 1945; (2) after Fabian Burac (then wounded in the forehead) fell, the appellant throw a
stone which hit Fabian's right clavicle, and (3) the appellant thereafter fled in the direction of his
house.
· The positive testimony of Trinidad was given full credit by the trial court, and the latter's
advantage of observing and hearing the witness should not be disregarded particularly where,
as in this case, Trinidad knew the appellant well and the latter merely relies on the conjecture
that Trinidad might have made a mistake in identifying her husband's assailant, considering the
time of the attack.
ISSUE:
W / N the appellant is guilty beyond reasonable doubt? In which the appellant raised an alibi of
being not in the scene of the death of the victim.
HELD:
· therefore, from the testimony of another witness for the Government (Caspara Bendicio)
to the effect that when she asked Fabian not long after the incident in question as to what had
happened, Fabian replied that he had been boloed by the appellant, which testimony (alleged
by the appellant to be inadmissible) was accepted by the trial court under the rule of res gestae,
there is sufficient proof regarding appellant's identity.
· Moreover, it should be remembered that the appellant was prosecuted, though only for
physical injuries, even before Fabian's death which occurred several days after June 8, 1945.
· Contrary to appellant's pretension, the death of Fabian Burac is established by the
testimony of his wife and mother-in-law. The certificate of the civil registrar of Tabaco dated
August 3, 1945, to the effect that the matter had not been registered in his office, merely shows
that no report was made up to the date mentioned, but it cannot conclusively negative the fact of
Fabian's death.
· We have no doubt that Fabian Burac died, as certified by Dr. Mariano Cruel, "of tetanus
secondary to the infected wound." When Fabian last reported for treatment on June 15, 1945,
Dr. Cruel already noticed Fabian's rigid muscles and slight lock-jaw, and this is the very reason
why he prescribed anti-tetanic serum, which, not being then available in the place, was never
actually administered on the patient.
· Appellant's surmise that Fabian might not have died of tetanus, because there are other
diseases sometimes exhibiting symptoms of tetanus, cannot prevail against the conclusion of
Dr. Cruel who in fact treated Fabian's wound and saw the manifestations of tetanus. The
appellant must of course be held responsible for the natural consequences of his unlawful act.
(People vs. Borbano, 76 Phil., 702.)

RULING:
The appealed judgment is hereby affirmed, with costs against the appellant. So ordered.
Pablo, Perfecto, Bengzon, Hontiveros, and Tuason, JJ., concur.

- People vs. Quianzon, 62 Phil. 162

G.R. No. 42607, September 28, 1935


THE PEOPLE OF THE PHILIPPINE ISLANDS, plaintiff-appellee, vs. JUAN
QUIANZON, defendant-appellant.
J. RECTO

FACTS:

On February 1, 1934, a novena for the suffrage of the soul of the deceased person was
being held in the house of Victoria Cacpal in a barrio, near the poblacion, of the
municipality of Paoay, Ilocos Norte, with the usual attendance of the relatives and friends.
The incident that led to the filling of these charges took place between 3 to 4 o'clock in the
afternoon. Andres Aribuabo, one of the persons present, went to ask for food of Juan
Quianzon, then in the kitchen, who, to all appearances, had the victuals in his care. It was
the second or third time that Aribuabo approached Quianzon with the same purpose
whereupon the latter, greatly peeved, took hold of a firebrand and applied it to Aribuabo’s
neck who ran to the place where the people were gathered exclaiming that he is wounded
and was dying. Raising his shirt, he showed to those present a wound in his abdomen
below the navel. Aribuabo died as a result of this wound on the tenth day after the incident
It is contended by the defense that even granting that it was the accused who inflicted the
wound which resulted in Aribuabo's death, he should not be convicted of homicide but only
of serious physical injuries because said wound was not necessarily fatal and the
deceased would have survived it had he not twice removed the drainage which Dr.
Mendoza had placed to control or isolate the infection.

ISSUE:

Whether or not the accused is criminally liable for the death of Aribuabo.

RULING:

YES. The contention is without merit. According to the physician who examined whether
he could survive or not." It was a wound in the abdomen which occasionally results in
traumatic peritonitis. The infection was cause by the fecal matter from the large intestine
which has been perforated. The possibility, admitted by said physician that the patient
might have survived said wound had he not removed the drainage, does not mean that the
act of the patient was the real cause of his death. Even without said act the fatal
consequence could have followed, and the fact that the patient had so acted in a
paroxysm of pain does not alter the juridical consequences of the punishable act of the
accused.

One who inflicts an injury on another is deemed by the law to be guilty of homicide
if the injury contributes mediately or immediately to the death of such other. The
fact that the other causes contribute to the death does not relieve the actor of
responsibility. . . . (13 R. C.L., 748.)

The grounds for this rule of jurisprudence are correctly set forth in 13 R.C.L., 751, as
follows:

While the courts may have vacilated from time to time it may be taken to be settled rule of the
common law that on who inflicts an injury on another will be held responsible for his death,
although it may appear that the deceased might have recovered if he had taken proper care of
himself, or submitted to a surgical operation, or that unskilled or improper treatment aggravated
the wound and contributed to the death, or that death was immediately caused by a surgical
operation rendered necessary by the condition of the wound. The principle on which this rule is
founded is one of universal application, and lies at the foundation of the criminal jurisprudence.
It is, that every person is to be held to contemplate and to be responsible for the natural
consequences of his own acts. If a person inflicts a wound with a deadly weapon in such a
manner as to put life in jeopardy, and death follows as a consequence of this felonious and
wicked act, it does not alter its nature or diminish its criminality to prove that other causes co-
operated in producing the fatal result. Indeed, it may be said that neglect of the wound or its
unskillful and improper treatment, which are of themselves consequences of the criminal act,
which might naturally follow in any case, must in law be deemed to have been among those
which were in contemplation of the guilty party, and for which he is to be held responsible. But,
however, this may be, the rule surely seems to have its foundation in a wise and practical policy.
A different doctrine would tend to give immunity to crime and to take away from human life a
salutary and essential safeguard. Amid the conflicting theories of the medical men, and the
uncertainties attendant upon the treatment of bodily ailments and injuries, it would be easy in
many cases of homicide to raise a doubt as to the immediate cause of death, and thereby to
open a wide door by which persons guilty of the highest crime might escape conviction and
punishment.

U.S. vs. Villanueva, 31 Phil. 412

31 Phil. 412
G. R. No. 10606, September 11, 1915
THE UNITED STATES, PLAINTIFF AND APPELLEE, VS. ANDRES VILLANUEVA,
DEFENDANT AND APPELLANT.

FACTS:

Benter himself testified that he had agreed to take Villanueva to the town of Pola in his boat;
that on arriving at the landing the boat was not there; that, thereupon he told Villanueva that to
avoid being late the latter ought to start right away; that while both of them were standing with
their arms folded it occurred to Villanueva to take hold of the bolo which complainant carried at
his belt; that the complainant tried to retain it and that in doing so he caught it by the blade and
cut himself in the palm of the right hand; that without a word being said, either before or
afterwards, Villanueva being in the best of temper, on finding himself injured the complainant left
the spot while Villanueva remained there.
But it turns out that Villanueva was not Benter's opponent nor was there any quarrel between
the two. Neither did Villanueva inflict any wound upon Benter. The latter injured himself by an
accident arising out of his own act.
ISSUE: WON the appellant Andres Villanueva can be held Criminally liable as felonious act?

HELD:
Pursuant to Art. 4, Par 1 that no felony is committed when the act or emission is not punishable
by the revised penal code and when the act is covered by any of the justifying circumstances.
The accused were not held criminally liable, because Villanueva were not committing felony
when they cause injury. No felony is committed when the act or omission is not punishable by
the revised penal code or when the act is covered by any of the justifying circumstances.

U.S. vs. Divino, 12 Phil. 175


US vs DIVINO, 12 Phil 175

Facts: The Court of First Instance convicted Feliciano Divino for the crime of lesions graves and
was sentenced to two years and eleven months and suspension from public office.

A girl named Alfonsa, about 13 years of age when the incident happened, was a servant for
Feliciano Divino and his family. She is a member of the Bagobo tribe, and because of lack of
proper nutrients in the tribe, her body was full of scars. Her feet were the focus in this case
because it is said that Feliciano Divino burned her feet, in a very unorthodox and harmful way in
a sense, by setting her feet on fire after applying petroleum to it and tying her down to the floor.
He argued in the court that when Alfonsa came to their home, her body was full of scars and
ulcers, and that the ulcer in her body was cured, through his efforts, except those on her feet.
He tried many times to cure her feet but failed because the girl would always walk barefoot
outside and that she would run whenever she was being cured because of the pain being
inflicted in the process. On the day of the incident, Feliciano said to have tied her up so she
won’t run during the process and then left her feet burning for about an hour and then
afterwards locked her up. He argued that the ulcer was getting worse and smelled quite
offensively.

Issue: Whether Feliciano Divino can be acquitted because he argued that he acted in good faith
and did not mean any harm to the girl, except to help her get cured.

Held: Medical arguments were brought in the light of the decision and that a doctor clearly
identified that the scars on Alfonsa’s feet were indeed because of burns and that the wounds
became worse on account of Feliciano’s efforts to cure them. Certainly it was found certain that
the acts of the guilty person do not seem to have been intended to cause an evil, but rather as a
remedy. However, article 568 or the Penal Code clearly states that a person that undertakes
medical assistance to another person is liable for any injuries resulting from such treatment, and
the fact that he acted in good faith and according to the best of his ability does not relieve him
from responsibility, although his ignorance may be considered as a mitigating circumstance. In
lieu, the Court reversed the ruling of the Court of First Instance and sentence Divino to simple
imprudence to the penalty of four months and suspension from office and right to suffrage and
to pay the costs of both instances.

U.S. vs. Mendieta, 34 Phil. 242

[ GR No. 11000, Mar 14, 1916 ]


US v. VALERIO MENDIETA
DECISION
34 Phil. 242

JOHNSON, J.:

This defendant was charged with the crime of assassination. The complaint alleged:
"That the said accused Valerio Mendieta, on or about the 22d of February, 1914, in the
municipality of Cauayan, Isabela, did, willfully, unlawfully, treacherously and criminally, assault
Pedro Acierto with a lance, thereby causing him a serious wound in the left side, as a result of
which the said Pedro Acierto died. An act committed with violation of law."

The defendant was first arrested, taken before a justice of the peace and given a preliminary
examination, at the close of which the justice of the peace found from the evidence that there
was probable cause for believing that the defendant was guilty of the crime charged and held
him for trial in the Court of First Instance.

Upon the above complaint the defendant was brought to trial in the Court of First Instance of the
Province of Isabela; he was duly arraigned, pleaded not guilty, was tried, found guilty of the
crime of assassination, with the qualifying circumstance of treachery, with the mitigating
circumstance of having executed the crime in vindication of an offense committed against him
by one Hilario Lauigan, and was sentenced to be imprisoned for a period of seventeen years
four months and one day of cadena temporal, with the accessory penalty provided for in article
56 of the Penal Code, to indemnify the heirs of the deceased in the sum of P1,000, and to pay
the costs. From that sentence the defendant appealed to this court The only question
presented by the appellant is one of fact. The appellant, through his attorney de officio,
attempts to show that the evidence adduced during the trial of the cause was not sufficient to
show, beyond a reasonable doubt, that he was guilty of the crime charged.

The Attorney-General, in a very interesting and carefully prepared brief, reached the conclusion
that the defendant was guilty of the crime charged in the complaint, with the qualifying
circumstance of treachery, without any mitigating circumstances, and recommended that the
decision of the lower court be modified, and that he be sentenced with the penalty of cadena
perpetua, with the accessory penalties provided for by law, to indemnify the heirs of the
deceased in the sum of P1,000 and to pay the costs.

From an examination of the record we find certain facts which stand undisputed:

First. That on the 22d day of February, 1914, the defendant, the deceased Pedro Acierto, Hilario
Lauigan, together with many others, were in a cockpit in the barrio of Barringin, municipality of
Cauayan, Province of lsabela.

Second. That while said parties were in said cockpit a quarrel arose concerning a bet made on
certain fighting roosters between a number of persons present, especially between the
defendant Valerio Mendieta and Hilario Lauigan. The proof does not show positively that the
deceased, Pedro Acierto, was in any way whatever connected with said quarrel.

Third. That some time after said quarrel took place in the cockpit, Pedro Acierto, together with
others, left the cockpit for the purpose of returning to their respective homes; that soon after
Pedro Acierto and his companions had left the cockpit and while they were yet within a few
yards of the cockpit, the defendant, Valerio Mendieta, rushed up behind him and stabbed him in
the back with a lance, which lance penetrated his body and entered his intestines, as a result of
which wound Pedro Acierto died on the 17th of March, 1914.
The defendant makes a feeble effort to show that the wound which he caused to Pedro Acierto
was caused in defending himself. He also attempts to show that the wound which he inflicted
upon Pedro Acierto was intended for Hilario Lauigan.

The first contention of the defendant, that he injured Pedro Acierto in self-defense, in our
opinion, is not supported by the record. Two or three witnesses were present and saw him inflict
the wound upon Pedro Acierto. No offense had been offered by Pedro Acierto to the defendant,
neither did he offer any resistance to the attack of the defendant upon him. In fact the record
shows that Pedro Acierto did not know that he was being pursued by the defendant at the time
the mortal wound was inflicted. Even admitting that the defendant intended to injure Hilario
Lauigan instead of Pedro Acierto, even that, in view of the mortal wound which he inflicted upon
the latter, in no way could be considered as a relief from his criminal act. That he made a
mistake in killing one man instead of another, when it is proved that he acted maliciously and
willfully, cannot relieve him from criminal responsibility. Neither do we believe that the fact that
he made a mistake in killing the wrong man should be considered as a mitigating circumstance.
We are therefore of the opinion that the recommendation of the Attorney-General should be
followed.

After a careful examination of the record, we are of the opinion that the record shows beyond a
reasonable doubt that the defendant is guilty of the crime charged, with the qualifying
circumstances of treachery and that there were no mitigating circumstances attending the
commission of the crime. For that reason the judgment of the lower court should be modified
and the defendant should be sentenced to be imprisoned with the penalty of cadena perpetua,
with the accessory penalties provided for by law, to indemnify the heirs of the deceased in the
sum of P1,000 and to pay the costs.

So ordered.

Torres, Moreland, Trent, and Araullo, JJ., concur.

Urbano v. Intermediate Appellate Court, 157 SCRA 10

FILOMENO URBANO, PETITIONER, VS. HON. INTERMEDIATE APPELLATE COURT AND


PEOPLE OF THE PHILIPPINES, RESPONDENTS.
G.R. No. 72964, January 07, 1988

FACTS:

Urbano hacked Javier using his bolo hitting the latter’s right palm for the very reason that Javier
was responsible for opening the irrigation canal that caused damage to Urbano’s palay. Upon
intercession, Urbano and Javier agreed to settle their differences and formalize their amicable
settlement.

Almost a month after the incident, Javier was rushed to the Hospital in a very serious condition
and later he died caused by tetanus toxin where the attending Doctor noticed the presence of a
healing wound in Javier's palm which could have been infected by tetanus.
Filomeno Urbano is found guilty for the crime of homicide after the death of Marcelo Javier
before the Trial Court.

The herein petitioner contended that the proximate cause of the death of Marcelo Javier was
due to his own negligence and record does not clearly show that the wound inflicted by Urbano
was infected with tetanus at the time of the infliction of the wound.

ISSUE:

Whether or not Javier’s death was the natural and logical consequence of Urbano’s lawful act?
Whether or not the herein petitioner may still be civilly liable if when the criminal liability is
reversed and acquitted.

RULING:

Javier’s death was NOT the natural and logical consequence of Urbano’s lawful act

The rule is that the death of the victim must be the direct, natural, and logical-consequence of
the wounds inflicted upon him by the accused. (People v. Cardenas, supra). Javier suffered a 2-
inch incised wound on his right palm when he parried the bolo which Urbano used in hacking
him. After 22 days, he suffered the symptoms of tetanus, like lockjaw and muscle spasms. The
following day, he died. The more credible conclusion is that at the time Javier's wound was
inflicted by the appellant, the severe form of tetanus that killed him was not yet present.
Consequently, Javier's wound could have been infected with tetanus after the hacking incident.
Considering the circumstances surrounding Javier's death, his wound could have been infected
by tetanus 2 or 3 or a few but not 20 to 22 days before he died.

Moreover, the proof that the accused caused the victim's-death must convince a rational mind
beyond reasonable doubt. The medical findings, however, lead us to a distinct possibility that
the infection of the wound by tetanus was an efficient intervening cause later or between the
time Javier was wounded to the time of his death. The infection was, therefore, distinct and
foreign to the crime. (People v. Rellin, 77 Phil. 1038).

The herein petitioner MAY STILL BE CIVILLY LIABLE if when the criminal liability is reversed
and acquitted.

Although the Court acquitted the petitioner for the crime of homicide, the well-settled doctrine is
that a person, while not criminally liable, may still be civilly liable.

U.S. vs. Maisa, 8 Phil. 597

United States vs. Maisa


8 Phil 597

FACTS:
Defendant, Anastasio Maisa was charged of criminal liability of a wrongful act or misdemeanor
against Isaac Monrayo. Maisa and Jose Machon were engaged in a fight and Isaac Monrayo
tried to separate them and pushed Maisa which caused the latter to fall to the fournd. Maisa hit
Monrayo in the face specifically hitting him in his right eye which became completely disabled.

ISSUE: Whether or not Maisa criminally liable for the offense charged when he claimed that the
blow was aimed at Machon and not Monrayo?

HELD: Yes, Maison is criminally liable. According to paragraph 3 of Article 1 of Old Penal
Code, although the wrongful act be committed against a person other than the one whom it was
intended to injure, this fact does not excuse the offender from criminal liability- liability for the
voluntary commission of a wrongful act or misdemeanor.
Therefore, Maison is sentenced to 2 years and one day of prision correccional, and to pay
Monrayo an indemnity of P50.00 or to suffer subsidiary imprisonment in case of insolvency.

People vs. Balmores, 85 Phil. 493

PEOPLE OF THE PHILIPPINES, Plaintiff-Appellee, Vs. RAFAEL BALMORES Y CAYA,


defendant-appellant.
G.R. No. L-1896; February 16, 1950
Ponente: OZAETA, J.

FACTS:

Appellant Rafael Balmores y Caya, waiving the right to counsel, pleaded guilty for the

commission of the crime of estafa through falsification of a security. On September 22, 1947,
the accused tore off the bottom of a genuine 1/8 unit Philippine Charity Sweepstakes ticket
thereby removing its true and real unidentified number. The defendant then substituted the
number and wrote in ink at the bottom of the ticket the number 074000 which is a prize-winning
number in the PCSO draw for July 29, 1947. He then presented and attempted to cash the
ticket in PCSO. However, Bayani Miller, the employee to whom the said ticket was presented,
discovered that the said ticket was falsified and immediately called for a policeman who arrested
the accused right then and there. Appellant contended the following facts: (1) that there could
be no genuine 1/8 unit ticket for June 29, 1947 and that the court has judicial notice that PCSO
issued only four ¼ units for each ticket for the said draw date and (2) that the substitution and
writing in ink of the said number 074000 was not considered as falsification if the true and real
number of the ticket was indeed 074000. Moreover, appellant contended that the trial court has
no jurisdiction to convict him on a plea of guilty because, for being illiterate, he was not assisted
by counsel.

ISSUES:

1. Whether or not the ticket is inconclusive

2. Whether or not the trial court lacked jurisdiction to convict him on a plea of guilty for not

being assisted by counsel due to being illiterate.


RULING:

1. No. The ticket is not inconclusive. The Supreme Court stated in its decision that the
appellant’s contentions are based on assumptions and not borne out by record. As to the
appellant’s first contention, the court cannot take judicial notice of what is not of common
knowledge as the number of sweepstakes ticket. But if it is true that the PCSO did not issue 1/8
on the mentioned draw date, then it would only strengthen the theory of the prosecution that the
1/8 unit of a ticket which appellant presented to the PCSO was spurious.

As to his second contention, the Court stated that “it is obvious that there would have been no
need of removal and substitution if the original number on the ticket was the same as that which
appellant wrote in ink in lieu thereof.” The removal of the true and real unidentified number of
sweepstakes ticket and substitution and writing in ink thereon of the number of a winning ticket
and the attempt to cash the ticket so altered as a prize-winning number constitutes the crime of
falsification of government obligation.

2. No. The fact that an accused is illiterate does not deprive the trial court of jurisdiction to
convict him on a plea of guilty although he is not assisted by counsel, when it appears that the
accused has waive such right.

The Supreme Court, therefore, affirmed the sentence appealed from, with costs against the

appellant.

Intod vs. Court of Appeals, 215 SCRA 52

SULPICIO INTOD v. CA, GR No. 103119, 1992-10-21


Ponente: Justice Jose C. Campos Jr.

Facts:
· In the morning of February 4, 1979, Sulpicio Intod, Jorge Pangasian, Santos
Tubio and Avelino Daligdig went to Salvador Mandaya's house.nd asked him to go with
them to the house of Bernardina Palangpangan.
· Thereafter, Mandaya and Intod, Pangasian, Tubio and Daligdig had a meeting
with Aniceto Dumalagan. He told Mandaya that he wanted Palangpangan to be killed
because of a land dispute between them and that Mandaya should accompany the four
(4) men, otherwise, he would also be killed.
· At about 10:00 o'clock in the evening of the same day,... Mandaya, Pangasian,
Tubio and Daligdig, all armed with firearms, arrived at Palangpangan's house
· At the instance of his companions, Mandaya pointed the location... of
Palangpangan's bedroom. Thereafter, Petitioner, Pangasian, Tubio and Daligdig fired at
said room. It turned out; however, that Palangpangan was in another City and her home
was then occupied by her son-in-law and his family. No one was in the room when the
accused fired the... shots. No one was hit by the gun fire.
· After trial, the Regional Trial Court convicted Intod of attempted murder.
· Petitioner seeks from this Court a modification of the judgment by holding him
liable only for an impossible... crime
Issues:
Whether or not the acts of the accused falls under the impossible crime?
Ruling:
The Revised Penal Code, inspired by the Positivist School, recognizes in the offender his
formidability [7] and now penalizes an act which were it not aimed at something quite impossible
or carried out with means which prove inadequate, would constitute a felony against person or
against property. [8] The rationale of Article 4(2) is to... punish such criminal tendencies.
Legal impossibility occurs where the intended acts, even if completed, would not amount
to a crime.[13] Thus:
Legal impossibility would apply to those circumstances where (1) the motive, desire and
expectation is to perform an act in violation of the law; (2) there is intention to perform the
physical act; (3) there is a performance of the intended physical act; and (4) the consequence
resulting from the intended act does not amount to a crime.
The impossibility of killing a person already dead [15] falls in this category.
On the other hand, factual impossibility occurs when extraneous circumstances
unknown to the actor or beyond his control prevent the consummation of the intended
crime.[16]
One example is the man who puts his hand in the coat... pocket of another with the intention to
steal the latter's wallet and finds the pocket empty.
The case at bar belongs to this category. Petitioner shoots the place where he thought his victim
would be, although in reality, the victim was not present in said place and thus, the petitioner
failed to accomplish his end.
In the Philippines, the Revised Penal Code, in Article 4(2), expressly provided for
impossible crimes and made them punishable.
RULING:
WE hereby hold Petitioner guilty of an impossible crime The rationale of Article 4(2) is to...
punish such criminal tendencies Having in mind the social danger and degree of criminality
shown by Petitioner, this Court sentences him to suffer the penalty of six (6) months of arresto
mayor, together with the accessory penalties... provided by the law, and to pay the costs.

Art. 5

People vs. Orifon, 57 Phil. 594

G.R. No. 36173, November 25, 1932


THE PEOPLE OF THE PHILIPPINES, plaintiff-appellee, -versus- MARIA ORIFON,
defendant-appellant.
BUTTE, J.

FACTS:
Maria Orifon was sentenced to cadena perpetua for the murder of her father. She pleaded
guilty to the charge at the preliminary investigation but on the arraignment upon the
information filed in the Court of First Instance she pleaded not guilty. The principal
evidence against the accused consisted of her confession which she wrote out in her own
handwriting and in her own dialect (Ilocano). It appears in the record in Spanish. Amember
of the court, who has personal knowledge of the Ilocano dialect, has assured that the
Spanish translation of said confession is substantiall correct. No question is raised on this
appeal as to said confession being the free and voluntary act and declaration of the
accused. The accused was sentenced with the penalty of cadena perpetua.

ISSUE:
Whether or not the penalty imposed was excessive. (YES)

RULING:
In view of the fact that the penalty of cadena perpetua no longer exists under the Revised
Penal Code, the sentence must be modified to reclusion perpetua with the accessory
penalties provided by law.

In view of the horrible wrong which this young woman suffered at the hands of her father
and of the obviously depressed state of mind and of body which she must have suffered
when she premeditated the act of madness and revenge for which she is now condemned
under the latter of the law to suffer life imprisonment, the court, invoking the provisions of
article 5, second paragraph, of the Revised Penal Code, submits to the Chief Executive
through the Department of Justice, its sincere opinion that the penalty imposed in this case
(and the law does not permit any lower penalty) is a clearly excessive penalty, having
regard to the condition of the accused and the circumstances which impelled her to
commit the crime for which she stands convicted.

- Director of Prisons v. Ang Cho Kio, 33 SCRA 494 (1970)

The judicial power shall be vested in one Supreme Court and in such lower courts as may be established
by law.

Judicial power includes the duty of the courts of justice to settle actual controversies involving rights which
are legally demandable and enforceable, and to determine whether or not there has been a grave abuse of
discretion amounting to lack or excess of jurisdiction on the part of any branch or instrumentality of the
Government. (Sec. 1, Art. VIII, 1987 Constitution)

Facts: 
Ang Cho Kio was convicted of various offenses and was granted conditional pardon in 1959. He
was never to return to the Philippines. In violation of his pardon, he returned in 1966 under the
name "Ang Ming Huy". However, he was identified by an inspector of the Immigration Bureau.
Thus, he was arrested. The Executive Secretary, by authority of the President, ordered him
recommitted to prison to serve the unexpired portion of the sentence that were imposed on him,
for having violated the conditioned of his pardon.

He filed a petition for habeas corpus which the CFI of Rizal denied. The CA affirmed
the decision but made a recommendation that Ang may be allowed to leave the country on the
first available transportation abroad.

The Solicitor General filed a motion for reconsideration praying for the deletion of the
recommendation. The Solicitor General maintains that the recommendation is not a part of
the decision and was uncalled for; that it gives the decision a political complexion, because
courts are not empowered to make such a recommendation, nor is it inherent or incidental in the
exercise of judicial powers. He also contends that allowing convicted aliens to leave the country
is an act of the state exercises solely in the discretion of the Chief Executive. It is urged that the
act of sending an undesirable alien out of the country is political in character, and the courts
should not interfere with, nor attempt to influence, the political acts of the President.

Issues:

1.Was the CA decision was proper?

2. Can the CA make recommendations?

Held:

1. No. The case before the CA was for habeas corpus. The only question to be resolved by the
CA was whether, or not, the CFI of Rizal, had rightly dismissed the petition of Ang Cho Kio for
habeas corpus. The CA was not called upon to review any sentence imposed upon Ang Cho
Kio. The sentence against him had long become final, and, in fact, he was pardoned. The
majority opinion should have been limited to the affirmance of the decision of the lower court,
and no more.

2. The recommendatory power of the courts in this jurisdiction are limited to those expressly
provided in the law — and such law is the provision of Section 5 of the Revised Penal Code as
follows:
Whenever a court has knowledge of any act which it may deem proper to repress and which is
not punishable by law, it shall render the proper decision, and shall report to the Chief
Executive, through the Department of Justice, the reasons which induce the court to believe
that said act should be made the subject of penal legislation.
In the same way the court shall submit to the Chief Executive, through the Department of
Justice such statement as may be deemed proper, without suspending the execution of the
sentence, when a strict enforcement of the provisions of this Code would result in the imposition
of a clearly excessive penalty, taking into consideration the degree of malice and the injury
caused by the offense.
Certainly, the recommendation in the majority opinion of the special division of the CA, now in
question, is not authorized under the aforequoted provision of Article 5 of the Revised Penal
Code. The CA was not called upon to review any sentence that was imposed on Ang Cho Kio. It
was simply called upon to determine whether Ang Cho Kio was illegally confined, or not, in the
insular penitentiary under the Director of Prisons.

It was improper for the CA justices to make a recommendation that would suggest a
modification or a correction of the act of the Chief Executive. The matter of whether
an alien who violated the laws in this country may remain or be deported is a political question
that should be left entirely to the Chief Executive to decide. Under the principle of separation of
powers, it is not within the province of the judiciary to express an opinion, or express a
suggestion, that would reflect on the wisdom or propriety of the action of the Chief Executive on
matters purely political in nature.

After all, courts are not concerned with the wisdom or morality of laws, but only in the
interpretation and application of the law. We believe that judges should refrain from expressing
irrelevant opinions in their decisions which may only reflect unfavorably upon their competence
and the propriety of their judicial actuations. (Director of Prisons vs. Ang Cho Kio, G.R. No. L-
30001, June 23, 1970)

Art. 6

- People vs. Lamahang, 61 Phil.703

PEOPLE vs. LAMAHANG


August 3, 1935
RECTO, J.
Mica Maurinne M. Adao

Facts were too short to provide for a short version.

FACTS: At early dawn on March 2, 1935, policeman Jose Tomambing, who was patrolling,
caught accused Lamahang in the act of making an opening with an iron bar on the wall of a
store of cheap goods. At that time the owner of the store, Tan Yu, was sleeping inside with
another Chinaman. The accused had only succeeded in breaking one board and in unfastening
another from the wall, when the policeman showed up, who instantly arrested him and placed
him under custody.

CFI found him guilty of attempted robbery and sentenced him to suffer 2 years and 4 months of
prision correccional and to an additional penalty of 10 years and 1 day of prision mayor for
being a habitual delinquent, with the accessory penalties of the law.

ISSUE: Does the act committed constitute attempted robbery?

RULING: No. It is attempted trespass to dwelling.


The attempt to commit an offense which the Penal Code punishes is that which has a logical
relation to a particular, concrete offense; that, which is the beginning of the execution of the
offense by overt acts of the perpetrator, leading directly to its realization and consummation.
The attempt to commit an indeterminate offense, inasmuch as its nature in relation to its
objective is ambiguous, is not a juridical fact from the standpoint of the Penal Code.
There is no doubt that in the case at bar it was the intention of the accused to enter Tan Yu's
store by means of violence, passing through the opening which he had started to make on the
wall, in order to commit an offense which, due to the timely arrival of policeman Tomambing, did
not develop beyond the first steps of its execution. But it is not sufficient, for the purpose of
imposing penal sanction, that an act objectively performed constitute a mere beginning of
execution; it is necessary to establish its unavoidable connection, like the logical and natural
relation of the cause and its effect, with the deed which, upon its consummation, will develop
into one of the offenses defined and punished by the Code; it is necessary to prove that said
beginning of execution, if carried to its complete termination following its natural course, without
being frustrated by external obstacles nor by the voluntary desistance of the perpetrator, will
logically and necessarily ripen into a concrete offense.

Thus, in case of robbery, in order that the simple act of entering by means of force or violence
another person's dwelling may be considered an attempt to commit this offense, it must be
shown that the offender clearly intended to take possession, for the purpose of gain, of some
personal property belonging to another. In the instant case, there is nothing in the record from
which such purpose of the accused may reasonably be inferred. From the fact established and
stated in the decision, that the accused on the day in question was making an opening by
means of an iron bar on the wall of Tan Yu's store, it may only be inferred as a logical
conclusion that his evident intention was to enter by means of force said store against the will of
its owner. That his final objective, once he succeeded in entering the store, was to rob, to cause
physical injury to the inmates, or to commit any other offense, there is nothing in the record to
justify a concrete finding.

In offenses not consummated, as the material damage is wanting, the nature of the action
intended (accion fin) cannot exactly be ascertained, but the same must be inferred from the
nature of the acts executed (accion medio). Hence, the necessity that these acts be such that
by their very nature, by the facts to which they are related, by the circumstances of the persons
performing the same, and by the things connected therewith, they must show without any doubt,
that they are aimed at the consummation of a crime. Acts susceptible of double interpretation,
that is, in favor as well as against the culprit, and which show an innocent as well as a
punishable act, must not and cannot furnish grounds by themselves for attempted nor frustrated
crimes. The relation existing between the facts submitted for appreciation and the offense which
said facts are supposed to produce must be direct; the intention must be ascertained from the
facts and therefore it is necessary, in order to avoid regrettable instances of injustice, that the
mind be able to directly infer from them the intention of the perpetrator to cause a particular
injury.

The fact under consideration does not constitute attempted robbery but attempted trespass to
dwelling. Under article 280 of the Revised Penal Code, this offense is committed when a private
person shall enter the dwelling of another against the latter's will. The accused may be
convicted and sentenced for an attempt to commit this offense in accordance with the evidence
and the following allegation contained in the information: "... the accused armed with an iron bar
forced the wall of said store by breaking a board and unfastening another for the purpose of
entering said store ... and that the accused did not succeed in entering the store due to the
presence of the policeman on beat Jose Tomambing, who upon hearing the noise produced by
the breaking of the wall, promptly approached the accused ... ." Under the circumstances of this
case the prohibition of the owner or inmate is presumed. Against the accused must be taken
into consideration the aggravating circumstances of nighttime and former convictions, —
inasmuch as the record shows that several final judgments for robbery and theft have been
rendered against him — and in his favor, the mitigating circumstance of lack of instruction. The
breaking of the wall should not be taken into consideration as an aggravating circumstance
inasmuch as this is the very fact which in this case constitutes the offense of attempted trespass
to dwelling.

The penalty provided by the Revised Penal Code for the consummated offense of trespass to
dwelling, if committed with force, is prision correccional in its medium and maximum periods and
a fine not exceeding P1,000 (art. 280, par. 2); therefore the penalty corresponding to attempted
trespass to dwelling is to degrees lower (art. 51), or, arresto mayor in its minimum and medium
periods. Because of the presence of two aggravating circumstances and one mitigating
circumstance the penalty must be imposed in its maximum period. Pursuant to article 29 of the
same Code, the accused is not entitled to credit for one-half of his preventive imprisonment.

Wherefore, the sentence appealed from is revoked and the accused is hereby held guilty of
attempted trespass to dwelling, committed by means of force, with the aforesaid aggravating
and mitigating circumstances and sentenced to three months and one day of arresto mayor,
with the accessory penalties thereof and to pay the costs.

- People v. Campuhan, GR 129433

PEOPLE v. PRIMO CAMPUHAN Y BELLO, GR No. 129433, 2000-03-30

Facts:

on 25 April 1996, at around 4 o'clock in the afternoon, Ma. Corazon P. Pamintuan, mother of
four (4)-year old Crysthel Pamintuan, went down from the second floor of their house to prepare
Milo
At the ground floor she met Primo Campuhan who was then busy filling small plastic bags with
water to be frozen into ice. Primo was a helper of Conrado Plata Jr., brother of Corazon.
Corazon was busy preparing the drinks,... she heard one of her daughters cry, "Ayo'ko,
ayo'ko!"[7] prompting Corazon to rush upstairs... she saw Primo Campuhan inside her children's
room kneeling before Crysthel whose pajamas or "jogging pants" and panty were already
removed, while... his short pants were down to his knees.
According to Corazon, Primo was forcing his penis into Crysthel's vagina
He evaded her blows and pulled up his pants.
Corazon then ran out and shouted for help... later, Primo was apprehended by those who
answered Corazon's call for help
Physical examination of the victim yielded negative results. No evident sign of extra-genital
physical injury was... noted by the medico-legal officer on Crysthel's body as her hymen was
intact... the basis of the conviction of the accused was Crysthel's answer to the question of the
court -
Q
Did the penis of Primo touch your organ?
A:
Yes, sir.
But when asked further whether his penis penetrated her organ, she readily said, "No." Thus -
Q
But did his penis penetrate your organ?
A:
No, sir.
Crysthel made a categorical statement denying... penetration... the possibility of Primo's penis
having breached Crysthel's vagina is belied by the child's own assertion that she resisted
Primo's advances by putting her legs close together

Issues:
Campuhan y Bello was found guilty of statutory rape

Ruling:
He is instead found guilty of ATTEMPTED RAPE

Principles:
Under Art. 6, in relation to Art. 335, of the Revised Penal Code, rape is attempted when the
offender commences the commission of rape directly by overt acts, and does not perform all the
acts of execution which should produce the crime of rape by reason of some cause or...
accident other than his own spontaneous desistance. All the elements of attempted rape - and
only of attempted rape - are present in the instant case, hence, the accused should be punished
only for it.

- U.S. vs. Eduave, 36 Phil. 209

THE UNITED STATES, PLAINTIFF AND APPELLEE, VS. PROTASIO EDUAVE,


DEFENDANT AND APPELLANT.
G.R. No. 12155, February 02, 1917
Ponente: MORELAND, J.

FACTS:

Protasio Eduave is accused of frustrated murder for having intent to kill a girl by a sudden attack
and a terrible blow in the back and side with his bolo.

ISSUE:

Is the defendant-appellant guilty for the crime of frustrated or attempted murder?

RULING:

The case is frustrated murder and the defendant-appellant is guilty of such. For the
defendant performed all of the acts which should have resulted in the consummated crime and
voluntarily desisted from further acts. Moreover, there is no intervention of a foreign or
extraneous cause or agency between the beginning of the commission of the crime and the
moment when all of the acts have been performed which should result in the consummated
crime.
Lastly, the subjective phase is completely passed. Subjectively the crime is complete. Nothing
interrupted the offender while he was passing through the subjective phase. The crime,
however, is not consummated by reason of the intervention of causes independent of the will of
the offender. He did all that was necessary to commit the crime. If the crime did not result as a
consequence it was due to something beyond his control.

The subjective phase is that portion of the acts constituting the crime included between the act
which begins the commission of the crime and the last act performed by the offender which, with
the prior acts, should result in the consummated crime. From that time forward the phase is
objective. It may also be said to be that period occupied by the acts of the offender over which
he has control—that period between the point where he begins and the point where the point
where he voluntarily desists. If between these two points the offender is stopped by reason of
any cause outside of his own voluntary desistance, the subjective phase has not been passed
and it is an attempt. If he is not so stopped but continues until he performs the last act, it is
frustrated murder.

- People vs. Borinaga, 55 Phil. 433

- People v. Kalalo, 59 Phil. 714

Valenzuela v. People, 525 SCRA 306

Valenzuela V. People GR No. 160188 June 21, 2007


Ponente: TINGA, J.:

FACTS:

On 19 May 1994, at around 4:30 p.m., petitioner and Calderon were sighted outside the Super
Sale Club, a supermarket within the ShoeMart (SM) complex along North EDSA, by Lorenzo
Lago (Lago), a security guard who was then manning his post at the open parking area of the
supermarket.

Lago saw petitioner, who was wearing an identification card with the mark "Receiving
Dispatching Unit (RDU)," hauling a push cart with cases of detergent of the well-known "Tide"
brand. Petitioner unloaded these cases in an open parking space, where Calderon was waiting.

Petitioner then returned inside the supermarket, and after five (5) minutes, emerged with more
cartons of Tide Ultramatic and again unloaded these boxes to the same area in the open
parking space.7

Thereafter, petitioner left the parking area and haled a taxi. He boarded the cab and directed it
towards the parking space where Calderon was waiting. Calderon loaded the cartons of Tide
Ultramatic inside the taxi, then boarded the vehicle. All these acts were eyed by Lago, who
proceeded to stop the taxi as it was leaving the open parking area.
When Lago asked petitioner for a receipt of the merchandise, petitioner and Calderon reacted
by fleeing on foot, but Lago fired a warning shot to alert his fellow security guards of the
incident. Petitioner and Calderon were apprehended at the scene, and the stolen merchandise
recovered.

The filched items seized from the duo were four (4) cases of Tide Ultramatic, one (1) case of
Ultra 25 grams, and three (3) additional cases of detergent, the goods with an aggregate value
of ₱12,090.00.9

Both accused filed their respective Notices of Appeal, 18 but only petitioner filed a brief 19 with the
Court of Appeals, causing the appellate court to deem Calderon’s appeal as abandoned and
consequently dismissed. Before the Court of Appeals, petitioner argued that he should
only be convicted of frustrated theft since at the time he was apprehended, he was never
placed in a position to freely dispose of the articles stolen. 20 However, in its Decision dated
19 June 2003,21 the Court of Appeals rejected this contention and affirmed petitioner’s
conviction.22 Hence the present Petition for Review, 23 which expressly seeks that petitioner’s
conviction "be modified to only of Frustrated Theft."24

ISSUE:

Whether or not they shall be indemnified with frustrated theft?

HELD:

No. Article 6 of the RPC provides that a felony is consummated when all the elements
necessary for its execution and accomplishment are present. In the crime of theft, the following
elements should be present –
(1) that there be taking of personal property;
(2) that said property belongs to another;
(3) that the taking be done with intent to gain;
(4) that the taking be done without the consent of the owner; and
(5) that the taking be accomplished without the use of violence against or intimidating of
persons or force upon things.

The court held that theft is produced when there is deprivation of personal property by
one with intent to gain. Thus, it is immaterial that the offender is able or unable to freely
dispose the property stolen since he has already committed all the acts of execution and the
deprivation from the owner has already ensued from such acts. Therefore, theft cannot have a
frustrated stage, and can only be attempted or consummated.

RULING:
WHEREFORE, the petition is DENIED. Costs against petitioner.

Art. 8

People v. Nacional, 248 SCRA 122

G.R. Nos. 111294-95. September 7, 1995


PEOPLE OF THE PHILIPPINES, plaintiff-appellee, vs.
WALTER NACIONAL alias “KA DENNIS,” ABSALON
MILLAMINA alias “KA ALVIN,” EFREN MUSA, RUDY
LUCES, JAVIER MIRABETE alias “COMMANDER,” and
ZACARIAS MILITANTE alias “CARE,” accused. JAVIER
MIRABETE alias “COMMANDER,” accused-appellant.

PUNO, J.:

FACT:

On or about the 21st day of February 1985 at Brgy. Salvacion, Municipality of Daraga,
Province of Albay, Philippines, and within the jurisdiction of this Honorable Court, the
abovenamed accused, with intent to kill, conspiring, confederating and helping one another, with
evident premeditation and taking advantage of superior strength, did then and there wilfully,
unlawfully and feloniously shoot with a firearm QUIRINO LAGASON and JOEL LAGASON,
inflicting injuries resulting to their death, to the damage and prejudice of his immediate heirs.

When arraigned, all the accused, except Absalon Millamina pleaded “not guilty.”

In this appeal, accused-appellant Javier Mirabete insists on his claim that he was merely
watching a volleyball game when the shooting happened. He denies being a member of the
NPA or any rebel organization. He likewise denies the existence of a plot and a conspiracy to
kill the Lagasons. Accused-appellant claims that he is a mere farmer, already 69 years old and
had barely finished Third Grade in school. According to him, his advanced age made it
impossible for him to join the NPA at the time of the incident. He contends that the testimonies
of Bienvenida Lagason and Crisanto Miranda identifying him with the group that killed the
Lagasons are unreliable and hearsay because both witnesses never knew him.

ISSUE:

Whether or not accused-appellant Javier Mirabete conspired in the killing of Lagasons.

RULING:

Yes. The records were reviewed and the court confirmed that accused-appellant was
part of the group that conspired to kill and actually killed the Lagasons. The identification of
accused-appellant was made not only by Bienvenida Lagason and Crisanto Miranda but also
the prosecution witnesses Elevino Rincopan, Genita Miranda, and appellant’s co-accused
Walter Nacional himself.
The conspiracy in the instant case was established at the meeting of February 25, 1985
at 4:00 P.M. which they called pulong-pulong. Apparently, nobody disagreed with the plan to
shoot the victims because immediately after the meeting, all the accused and Wilson Lita were
seen walking as a group towards Barangay Salvacion. When they saw their intended victims,
they shot them and fled towards the RCPI building. Those left at the waiting shed including
accused-appellant Javier Mirabete, likewise fled towards the same direction. Clearly, the
shooting of the Lagasons was characterized by a unity of purpose, intention and design.
The court said conspiracy, once established, makes each of the conspirators liable for
the acts of the others. All because in contemplation of law, the act of one is the act of all.
Article 8 of the RPC described that conspiracy exists when two or more persons come to
an agreement concerning the commission of a felony and decide to commit it.
The court likewise find no mitigating circumstance in the commission of the crimes. The
analogous circumstance of age of over 70 years as mentioned in Article 12 circumstances
number 2 in RPC cannot be considered mitigating because accused-appellant was only 59
years old at the time of the commission of the offense.
The court then affirmed the criminal liability of the accused-appellant and likewise
affirmed the civil indemnity of P50,000.00 imposed by the trial court for each of the deaths of
Quirino and Joel Lagason, and must be shared solidarily by all the accused.

- People v. Gambao, GR 172707

People v. Morilla, GR 189833

PEOPLE OF THE PHILIPPINES, Plaintiff-Appellee, vs. JAVIER MORILLA Y AVELLANO,


Accused-Appellant.
G.R. No. 189833 February 5, 2014
PEREZ, J.:

Note: The accused was charged under R.A. 6425 (The Dangerous Drugs Act of 1972) and not
R.A. 9165 (2002).

FACTS:

The criminal information is as follows: Morilla, Mitra (Mayor of Quezon Province), Yang and
Dequilla, who all belong to an organized/syndicate crime group as they all help one another, for
purposes of gain in the transport of illegal drugs, and in fact, conspiring and confederating
together and mutually aiding and abetting one another, did then and there wilfully and
feloniously transport by means of two (2) motor vehicles, namely a Starex van with
commemorative plate to read "Mayor" and a municipal ambulance of Quezon Province, shabu.

The ruling of conspiracy between Mayor Mitra and Morilla in the trial court was based on the
testimonies of the four accused themselves. It was that the two vehicles, the Starex van driven
by Mayor Mitra and the ambulance van driven by Morilla, left Infanta, Quezon en route to
Manila. The Starex van which was ahead of the ambulance was able to pass the checkpoint set
up by the police officers. However, the ambulance driven by Morilla was stopped by police
officers. Through the untinted window, one of the police officers noticed several sacks inside the
van. Upon inquiry of the contents, Morilla replied that the sacks contained narra wooden tiles.

Unconvinced, the police officers requested Morilla to open the rear door of the car for further
inspection. When it was opened, the operatives noticed that white crystalline granules were
scattered on the floor, prompting them to request Morilla to open the sacks. At this moment,
Morilla told the police officers that he was with Mayor Mitra in an attempt to persuade them to let
him pass. His request was rejected by the police officers and upon inspection, the contents of
the sacks turned out to be sacks of shabu. This discovery prompted the operatives to chase the
Starex van of Mayor Mitra. They then inquired if the mayor knew Morilla. On plain view, the
operatives noticed that his van was also loaded with sacks like the ones found in the
ambulance. Thus, Mayor Mitra was also requested to open the door of the vehicle for
inspection. At this instance, Mayor Mitra offered to settle the matter but the same was rejected.

ISSUE:

Whether the accused may be convicted for conspiracy to commit the offense of illegal transport
of shabu

RULING:

A conspiracy exists when two or more persons come to an agreement concerning the
commission of a felony and decide to commit it. To determine conspiracy, there must be a
common design to commit a felony.

Morilla argues that the mere act of driving the ambulance on the date he was apprehended is
not sufficient to prove that he was part of a syndicated group involved in the illegal
transportation of dangerous drugs. This argument is misplaced.

In conspiracy, it need not be shown that the parties actually came together and agreed in
express terms to enter into and pursue a common design. The assent of the minds may be and,
from the secrecy of the crime, usually inferred from proof of facts and circumstances which,
taken together, indicate that they are parts of some complete whole. In this case, the totality of
the factual circumstances leads to a conclusion that Morilla conspired with Mayor Mitra in a
common desire to transport the dangerous drugs. Both vehicles loaded with several sacks of
dangerous drugs, were on convoy from Quezon to Manila. Mayor Mitra was able to drive
through the checkpoint set up by the police operatives. When it was Morilla’s turn to pass
through the checkpoint, he was requested to open the rear door for a routinary check. Noticing
white granules scattered on the floor, the police officers requested Morilla to open the sacks. If
indeed he was not involved in conspiracy with Mayor Mitra, he would not have told the police
officers that he was with the mayor.

His insistence that he was without any knowledge of the contents of the sacks and he just
obeyed the instruction of his immediate superior Mayor Mitra in driving the said vehicle likewise
bears no merit.

Here, Morilla and Mayor Mitra were caught in flagrante delicto in the act of transporting the
dangerous drugs on board their vehicles. "Transport" as used under the Dangerous Drugs Act
means "to carry or convey from one place to another." It was well established during trial that
Morilla was driving the ambulance following the lead of Mayor Mitra, who was driving a Starex
van going to Manila. The very act of transporting methamphetamine hydrochloride is malum
prohibitum since it is punished as an offense under a special law. The fact of transportation of
the sacks containing dangerous drugs need not be accompanied by proof of criminal intent,
motive or knowledge.

Yongco v. People, GR 209373

JOEL YONGCO v. PEOPLE, GR No. 209373, 2014-07-30


Facts:

The prosecution presented as one of its witnesses a casual employee of the city government,
Pablo Salosod, who testified that on April 16, 2005 at around 1:30 a.m., while attending a wake
at the Cosmopolitan Funeral Parlor, he was fetched and requested by petitioner Tangian to...
accompany him to the CEO. At the office garage, Salosod and his fellow garbage collectors
were allegedly directed by petitioners Tangian and Yongco to load car parts that petitioners
considered as waste items, the subject items of the theft, on the truck driven by Tangian. They...
then drove to Tominobo, Iligan City where the materials were unloaded in front of Delfin Junk
Store, and before the truck left the shop, Salosod allegedly saw petitioner Lañojan giving a
thumbs-up sign to Tangian. On the way back, Tangian allegedly confessed to Salosod that it...
was Lañojan who requested that the items be brought at the junk shop. Another employee,
Rommel Ocaonilla, corroborated the testimony of Salosod.

Prosecution witness Oliveros Garcia meanwhile testified witnessing the unloading of the items
in front of the junk store, after which, Lañojan covered the items up with a sack. The following
morning, he allegedly saw Lañojan's brother-in-law, who coincidentally works at the... shop,
take the items inside.

Witnesses Dioscoro Galorio and Atty. Ulysses Lagcao, employee and consultant of the city
government, respectively, testified that they conducted investigations relative to the incident and
found out that the items stolen consisted of one Nissan transmission, one unit boom, one

Nissan I-beam, and one differential of Tamaraw, with total valuation of PhP 12,000. Upon their
investigation, they recommended to the city legal officer the filing of the present criminal case
against the three petitioners.

Issues:

whether or not the CA erred in sustaining petitioners' conviction. Central to resolving this issue
is determining whether or not there indeed existed conspiracy between petitioners in committing
the offense... charged.

Ruling:

There is no dispute that the items (transmission, boom arm, differential assembly, and I-beam)
which are the subject matter of this case belong to the CEO of Iligan City. There is no dispute
that these items, although considered "heap of scrap," have not yet been... declared
unserviceable or waste by the proper authority or office. Nor have they been marked for proper
disposal. Unless properly disposed in accordance with Section 379 of the Local Government
Code, these items are still government properties or owned by the City of Iligan.

There is also no dispute that these items were taken away from the CEO and were already
under complete and effective control of the persons taking the same. This is because these
items were loaded onto the garbage truck driven by Tangian and brought to Tominobo at the
Delfin Junk Store.
Apparently, the taking of these items was without the consent of the CEO of Iligan City because
there was no gate pass issued to that effect. Evidence shows that when the garbage truck left
the premises of the CEO, no gate pass was surrendered by Tangian. Yongco did not bother... to
ask for a gate pass on the pretext that there was another guard on duty at the gate.

Addressing the issue head on, We uphold the findings of the appellate court. No error can be
ascribed to the CA when it determined the existence of conspiracy between and among
petitioners in this case.

In the case at bar, even though there is no showing of a prior agreement among the accused,
their separate acts taken and viewed together are actually connected and complemented each
other indicating a unity of criminal design and purpose.[10

Tangian's complicity in the illicit deed was manifest from the fact, as he himself admitted, that he
was the one who personally transported the stolen items from the CEO to the junkshop. His
claim that he was not aware of any irregularity in the act he performed is rendered... dubious by
his 16 years of service as truck driver for the City of Iligan. To be sure, his record of service
argues against his claim of ignorance of the standard protocol that a gate pass to be issued by
the CEO property custodian should first be secured before taking out items... from the CEO
compound, including alleged waste materials. He should also know better than to assume that
Lañojan can authorize the withdrawal of items without the requisite gate pass since Lañojan's
duty, as security guard, is precisely to prevent the same.

Similarly, Yongco's claim of good faith is belied by his own admission that he knew of the office
procedure that a gate pass is required every time something is taken out of the CEO premises.
In fact, four gate passes were given to him that morning by Lañojan, covering waste... materials
withdrawn during the latter's shift. At the very least, this should have reminded him of his duty to
demand a gate pass for property leaving the CEO premises. Neither memory lapses or lapses
in the performance of his duty will explain Yongco's failure to demand a gate... pass. The only
viable explanation is that he was in connivance with other petitioners.[

The character and effect of conspiracy are not to be adjudged by dismembering it and viewing
its separate parts but only by... looking at it as a whole acts done to give effect to conspiracy
may be, in fact, wholly innocent acts.[14] Applying this doctrine in the case at bench, it can
reasonably be concluded that despite Lañojan's lack of physical participation in hauling the
items... to Tangian's truck and bringing them to the junk shop, he can still be liable for Qualified
Theft via conspiracy. All told, there is no cogent reason for us to disturb the findings of the
appellate court, affirmatory of those of the trial court.

Estrada v. Sandiganbayan, G.R. No. 148965. February 26, 2002

JOSE “JINGGOY” E. ESTRADA, PETITIONER, VS. SANDIGANBAYAN (THIRD DIVISION),


PEOPLE OF THE PHILIPPINES AND OFFICE OF THE OMBUDSMAN, RESPONDENTS.
G.R. No. 148965, February 26, 2002
Ponente: PUNO, J.
FACTS:

As an offshoot of the impeachment proceedings against the former President of the Philippines
Joseph Ejercito Estrada, five criminal complaints against the former President and members of
his family, his associates, friends and conspirators were filed with the respondent Office of the
Ombudsman.
On April 4, 2001, the Ombudsman issued a Joint Resolution finding probable cause warranting
the filing with the Sandiganbayan of several criminal charges against the former President and
the other respondents therein. One of the charges was for the plunder under Republic Act No.
7080 and among the respondents was the former’s president’s son the petitioner in this case
Jose "Jinggoy" Estrada, then mayor of San Juan, Metro Manila.
The charge was amended and filed on April 18, 2001. Docketed as Criminal Case No. 26558,
the case was assigned to the Third Division of the Sandiganbayan. The arraignment of the
accused was set on July 10, 2001. No bail for petitioner's provisional liberty was fixed. On April
24, 2001, petitioner filed a "Motion to Quash or Suspend" the Amended Information on the
ground that the Anti-Plunder Law, Republic Act No. 7080, is unconstitutional and that it charged
more than one offense. The Respondent Ombudsman opposed the motion.
On April 25, 2001, the respondent court issued a warrant of arrest for the petitioner and his co-
accused. On its basis, the petitioner and his co-accused were placed in custody of the law. On
April 30, 2001, petitioner filed a "Very Urgent Omnibus Motion" alleging that: (1) no probable
cause exists to put him on trial and hold him liable for plunder, it appearing that he was only
allegedly involved in illegal gambling and not in a "series or combination of overt or criminal
acts" as required in R.A. No. 7080; and (2) he is entitled to bail as a matter of right.
On July 9, 2001, the Sandiganbayan issued a Resolution denying petitioner's "Motion to Quash
and Suspend" and "Very Urgent Omnibus Motion." Petitioner's alternative prayer to post bail
was set for hearing after arraignment of all the accused.
The Amended Information is divided into three parts: (1) the first paragraph charges former
President Joseph E. Estrada with the crime of plunder together with petitioner Jose "Jinggoy"
Estrada, Charlie "Atong" Ang, Edward Serapio, Yolanda Ricaforte and others; (2) the second
paragraph spells out in general terms how the accused conspired in committing the crime of
plunder; and (3) the four sub-paragraphs (a) to (d) describe in detail the predicate acts
constitutive of the crime of plunder pursuant to items (1) to (6) of R.A. No. 7080, and state the
names of the accused who committed each act.
Pertinent to the case at bar is the predicate act alleged in subparagraph (a) of the Amended
Information which is of "receiving or collecting, directly or indirectly, on several instances, money
in the aggregate amount of P545,000,000.00 for illegal gambling in the form of gift, share,
percentage, kickback or any form of pecuniary benefit…" In this subparagraph (a), petitioner, in
conspiracy with former President Estrada, is charged with the act of receiving or collecting
money from illegal gambling amounting to P545 million

ISSUES:

(a) Whether or not the Anti-Plunder Law, Republic Act No. 7080, is unconstitutional;
(b) Whether or not should the charge against petitioner be dismissed on the ground that the
allegation of conspiracy in the Information is too general?; and
(c) Whether or not the petitioner is entitled to bail as a matter of right.

RULING:

(a) The constitutionality of Republic Act No. 7080 has already been settled in the case of
Joseph Estrada v. Sandiganbayan.
(b) With respect to the second issue, while it is clear that all the accused named in sub-
paragraphs (a) to (d) thru their individual acts conspired with the former President Estrada to
enable the latter to amass, accumulate or acquire ill-gotten wealth in the aggregate amount of
P4,097,804,173.17, as the Amended Information is worded, however, it is not certain whether
the accused persons named in sub-paragraphs (a) to (d) conspired with each other to enable
the former President to amass the subject ill-gotten wealth. In view of the lack of clarity in the
Information, the Court held petitioner Jose “Jinggoy” Estrada cannot be penalized for the
conspiracy entered into by the other accused with the former President as related in the second
paragraph of the Amended Information in relation to its sub-paragraphs (b) to (d).
For a conspiracy should be understood on two levels. As a general rule, conspiracy is not a
crime in our jurisdiction. It is punished as a crime only when the law fixes a penalty for its
commission such as in conspiracy to commit treason, rebellion, and sedition. In contrast, under
American criminal law, the agreement or conspiracy itself is the gravamen of the offense. When
conspiracy is charged as a crime, the act of conspiring and all the elements of the said crime
must be set forth in the complaint or information. The requirements on the sufficiency of
allegations are different when conspiracy is not charged as a crime in itself but only as the mode
of committing the crime as in the case at bar.
There is less necessity of reciting its particularities in the Information because conspiracy is not
the gravamen of the offense charged. The conspiracy is significant only because it changes the
criminal liability of all the accused in the conspiracy and makes them answerable as co-
principals regardless of the degree of their participation in the crime. The liability of the
conspirators is collective and each participant will be equally responsible for the acts of others,
for the act of one is the act of all.
A conspiracy indictment need not, of course, aver all the components of conspiracy or allege all
the details thereof, like the part that each of the parties therein has performed, the evidence
proving the common design or the facts connecting all the accused with one another in the web
of the conspiracy. Neither is it necessary to describe conspiracy with the same degree of
particularity required in describing a substantive offense. It is enough that the indictment
contains a statement of facts relied upon to be constitutive of the offense in ordinary and
concise language, with as much certainty as the nature of the case will admit, in a manner that
can enable a person of common understanding to know what is intended, and with such
precision that the accused may plead his acquittal or conviction to a subsequent indictment
based on the same facts.
It is enough to allege conspiracy as a mode in the commission of an offense in either of the
following manner: (1) by use of the word conspire, or its derivatives or synonyms, such as
confederate, connive, collude, etc; or (2) by allegations of basic facts constituting the conspiracy
in a manner that a person of common understanding would know what is intended, and with
such precision as would enable the accused to competently enter a plea to a subsequent
indictment based on the same facts.
Instead, the petitioner can be held accountable only for the predicate acts that he allegedly
committed as related in sub-paragraph (a) of the Amended Information which were allegedly
done in conspiracy with the former President whose design was to amass ill-gotten wealth
amounting to more than P4 billion.
However, if the allegation should be proven, the penalty of the petitioner cannot be unclear. It.
will be no different from that of the former President for in conspiracy, the act of one is the act of
the other. The imposable penalty is provided in Section 2 of Republic Act No. 7080, viz.:
"Section 2. Any public officer who, by himself or in connivance with the members of his family,
relatives by affinity or consanguinity, business associates, subordinates or other persons,
amasses, accumulates or acquires ill-gotten wealth through a combination or series of overt or
criminal acts as described in Section 1 (d) hereof in the aggregate amount or total value of at
least Fifty million pesos (P50,000,000.00) (now P75,000,000.00 under RA 7080, as amended)
shall be guilty of the crime of plunder and shall be punished by reclusion perpetua to death. Any
person who participated with the said public officer in the commission of an offense contributing
to the crime of plunder shall likewise be punished for such offense. In the imposition of
penalties, the degree of participation and the attendance of mitigating and extenuating
circumstances, as provided by the Revised Penal Code, shall be considered by the court."
The Court added that it cannot fault the Ombudsman for including the predicate offenses
alleged in sub-paragraphs (a) to (d) of the Amended information in one and not four separate
Informations. The court explained the history of the Anti-Plunder Law, thus:
“A study of the history of R.A. No. 7080 will show that the law was crafted to avoid the mischief
and folly of filing multiple informations. The Anti-Plunder Law was enacted in the aftermath of
the Marcos regime where charges of ill-gotten wealth were filed against former President
Marcos and his alleged cronies. Government prosecutors found no appropriate law to deal with
the multitude and magnitude of the acts allegedly committed by the former President to acquire
illegal wealth. They also found that under the then existing laws such as the Anti-Graft and
Corrupt Practices Act, the Revised Penal Code and other special laws, the acts involved
different transactions, different time and different personalities. Every transaction constituted a
separate crime and required a separate case and the over-all conspiracy had to be broken
down into several criminal and graft charges. The preparation of multiple Informations was a
legal nightmare but eventually, thirty-nine (39) separate and independent cases were filed
against practically the same accused before the Sandiganbayan. R.A. No. 7080 or the Anti-
Plunder Law was enacted precisely to address this procedural problem. This is pellucid in the
Explanatory Note to Senate Bill No. 733, viz.:
"Plunder, a term chosen from other equally apt terminologies like kleptocracy and economic
treason, punishes the use of high office for personal enrichment, committed thru a series of acts
done not in the public eye but in stealth and secrecy over a period of time, that may involve so
many persons, here and abroad, and which touch so many states and territorial units. The acts
and/or omissions sought to be penalized do not involve simple cases of malversation of public
funds, bribery, extortion, theft and graft but constitute plunder of an entire nation resulting in
material damage to the national economy. The above-described crime does not yet exist in
Philippine statute books. Thus, the need to come up with a legislation as a safeguard against
the possible recurrence of the depravities of the previous regime and as a deterrent to those
with similar inclination to succumb to the corrupting influence of power.”
(c) The Sandiganbayan submitted its Resolution (dated December 20, 2001) denying
petitioner's motion for bail for "lack of factual basis." Based on the earlier testimony of Dr.
Anastacio, the Sandiganbayan found that petitioner "failed to submit sufficient evidence to
convince the court that the medical condition of the accused requires that he be confined at
home and for that purpose that he be allowed to post bail."
The Court clarified that the crime of plunder is punished with the penalty of reclusion
perpetua to death. Under the Revised Rules of Court, offenses punishable by death,
reclusion perpetua or life imprisonment are non-bailable when the evidence of guilt is strong,
to wit:

· "Sec. 7. Capital offense or an offense punishable by reclusion perpetua or life


imprisonment, not bailable. – No person charged with a capital offense, or an offense
punishable by reclusion perpetua or life imprisonment, shall be admitted to bail when evidence
of guilt is strong, regardless of the stage of the criminal prosecution." Section 7, Rule 114 of the
Revised Rules of Criminal Procedure is based on Section 13, Article III of the 1987 Constitution
which reads:
"Sec. 13. All persons, except those charged with offenses punishable by reclusion perpetua
when evidence of guilt is strong, shall, before conviction be bailable by sufficient sureties, or be
released on recognizance as may be provided by law. The right to bail shall not be impaired
even when the privilege of the writ of habeas corpus is suspended. Excessive bail shall not be
required."
Thus, the constitutional mandate makes the grant or denial of bail in capital offenses hinge on
the issue of whether or not the evidence of guilt of the accused is strong. The trial court is
required to conduct bail hearings wherein both the prosecution and the defense will be afforded
sufficient opportunity to present their respective evidence. The burden of proof lies with the
prosecution to show that the evidence of guilt is strong.
The hearings on which respondent court based its Resolution of December 20, 2001 involved
the reception of medical evidence only and which evidence was given five months earlier in
September 2001. The records do not show that evidence on petitioner's guilt was presented
before the lower court. Thus, the Sandiganbayan was ordered to conduct hearings to ascertain
whether evidence of petitioner's guilt is strong to determine whether to grant bail to the latter.

B. Chapter Two: Justifying and Exempting Circumstances

Art. 11
- Art. 100
- R.A. No. 9262, Sec. 26

R.A. No. 9344, Sec. 6, as amended by R.A. No. 10630 Self-defense

[REPUBLIC ACT NO. 10630]

AN ACT STRENGTHENING THE JUVENILE JUSTICE SYSTEM IN THE PHILIPPINES,


AMENDING FOR THE PURPOSE REPUBLIC ACT NO. 9344, OTHERWISE KNOWN AS THE
“JUVENILE JUSTICE AND WELFARE ACT OF 2006” AND APPROPRIATING FUNDS
THEREFOR

SEC. 6. Section 20 of Republic Act No. 9344 is hereby amended to read as follows:

“SEC. 20. Children Below the Age of Criminal Responsibility. – If it has been determined that
the child taken into custody is fifteen (15) years old or below, the authority which will have an
initial contact with the child, in consultation with the local social welfare and development officer,
has the duty to immediately release the child to the custody of his/her parents or guardian, or in
the absence thereof, the child’s nearest relative. The child shall be subjected to a community-
based intervention program supervised by the local social welfare and development officer,
unless the best interest of the child requires the referral of the child to a youth care facility or
‘Bahay Pag-asa’ managed by LGUs or licensed and/or accredited NGOs monitored by the
DSWD.

“The local social welfare and development officer shall determine the appropriate programs for
the child who has been released, in consultation with the child and the person having custody
over the child. If the parents, guardians or nearest relatives cannot be located, or if they refuse
to take custody, the child may be released to any of the following:

“(a) A duly registered nongovernmental or religious organization;

“(b) A barangay official or a member of the Barangay Council for the Protection of Children
(BCPC);

“(c) A local social welfare and development officer; or, when and where appropriate, the DSWD.
“If the child has been found by the local social welfare and development officer to be dependent,
abandoned, neglected or abused by his/her parents and the best interest of the child requires
that he/she be placed in a youth care facility or ‘Bahay Pag-asa’, the child’s parents or
guardians shall execute a written authorization for the voluntary commitment of the child:
Provided, That if the child has no parents or guardians or if they refuse or fail to execute the
written authorization for voluntary commitment, the proper petition for involuntary commitment
shall be immediately filed by the DSWD or the Local Social Welfare and Development Office
(LSWDO) pursuant to Presidential Decree No. 603, as amended, otherwise known as ‘The
Child and Youth Welfare Code’ and the Supreme Court rule on commitment of children:
Provided, further, That the minimum age for children committed to a youth care facility or ‘Bahay
Pag-asa’ shall be twelve (12) years old.”

“SEC. 20-A. Serious Crimes Committed by Children Who Are Exempt From Criminal
Responsibility. – A child who is above twelve (12) years of age up to fifteen (15) years of age
and who commits parricide, murder, infanticide, kidnapping and serious illegal detention where
the victim is killed or raped, robbery, with homicide or rape, destructive arson, rape, or
carnapping where the driver or occupant is killed or raped or offenses under Republic Act No.
9165 (Comprehensive Dangerous Drugs Act of 2002) punishable by more than twelve (12)
years of imprisonment, shall be deemed a neglected child under Presidential Decree No. 603,
as amended, and shall be mandatorily placed in a special facility within the youth care faculty or
‘Bahay Pag-asa’ called the Intensive Juvenile Intervention and Support Center (IJISC).

“In accordance with existing laws, rules, procedures and guidelines, the proper petition for
involuntary commitment and placement under the IJISC shall be filed by the local social welfare
and development officer of the LGU where the offense was committed, or by the DSWD social
worker in the local social welfare and development officer’s absence, within twenty-four (24)
hours from the time of the receipt of a report on the alleged commission of said child. The court,
where the petition for involuntary commitment has been filed shall decide on the petition within
seventy-two (72) hours from the time the said petition has been filed by the DSWD/LSWDO.
The court will determine the initial period of placement of the child within the IJISC which shall
not be less than one (1) year. The multi-disciplinary team of the IJISC will submit to the court a
case study and progress report, to include a psychiatric evaluation report and recommend the
reintegration of the child to his/her family or the extension of the placement under the IJISC. The
multi-disciplinary team will also submit a report to the court on the services extended to the
parents and family of the child and the compliance of the parents in the intervention program.
The court will decide whether the child has successfully completed the center-based
intervention program and is already prepared to be reintegrated with his/her family or if there is
a need for the continuation of the center-based rehabilitation of the child. The court will
determine the next period of assessment or hearing on the commitment of the child.”

“SEC. 20-B. Repetition of Offenses. – A child who is above twelve (12) years of age up to fifteen
(15) years of age and who commits an offense for the second time or oftener: Provided, That
the child was previously subjected to a community-based intervention program, shall be
deemed a neglected child under Presidential Decree No. 603, as amended, and shall undergo
an intensive intervention program supervised by the local social welfare and development
officer: Provided, further, That, if the best interest of the child requires that he/she be placed in a
youth care facility or ‘Bahay Pag-asa’, the child’s parents or guardians shall execute a written
authorization for the voluntary commitment of the child: Provided, finally, That if the child has no
parents or guardians or if they refuse or fail to execute the written authorization for voluntary
commitment, the proper petition for involuntary commitment shall be immediately filed by the
DSWD or the LSWDO pursuant to Presidential Decree No. 603, as amended.”
“SEC. 20-C. Exploitation of Children for Commission of Crimes. – Any person who, in the
commission of a crime, makes use, takes advantage of, or profits from the use of children,
including any person who abuses his/her authority over the child or who, with abuse of
confidence, takes advantage of the vulnerabilities of the child and shall induce, threaten or
instigate the commission of the crime, shall be imposed the penalty prescribed by law for the
crime committed in its maximum period.”

“SEC. 20-D. Joint Parental Responsibility. – Based on the recommendation of the multi-
disciplinary team of the IJISC, the LSWDO or the DSWD, the court may require the parents of a
child in conflict with the law to undergo counseling or any other intervention that, in the opinion
of the court, would advance the welfare and best interest of the child.

“As used in this Act, ‘parents’ shall mean any of the following:

“(a) Biological parents of the child; or

“(b) Adoptive parents of the child; or

“(c) Individuals who have custody of the child.

“A court exercising jurisdiction over a child in conflict with the law may require the attendance of
one or both parents of the child at the place where the proceedings are to be conducted.

“The parents shall be liable for damages unless they prove, to the satisfaction of the court, that
they were exercising reasonable supervision over the child at the time the child committed the
offense and exerted reasonable effort and utmost diligence to prevent or discourage the child
from committing another offense.”

“SEC. 20-E. Assistance to Victims of Offenses Committed by Children. – The victim of the
offense committed by a child and the victim’s family shall be provided the appropriate
assistance and psychological intervention by the LSWDO, the DSWD and other concerned
agencies.”

- De la Cruz vs. People, 741 SCRA 97 (2014)

G.R. No. 189405. November 19, 2014


SHERWIN DELA CRUZ, petitioner, vs. PEOPLE OF THE PHILIPPINES and
CARLOS ALBERTO L. GONZALES, in behalf of his deceased brother, JEFFREY
WERNHER L. GONZALES, respondents.
PERALTA, J.:
FACTS:
Sherwin was charged with the crime of Homicide in an Information dated March
2, 2005, which alleged: That on or about the 1st day of January 2005, in the City of
Makati, Philippines and within the jurisdiction of this Honorable Court, the above named
accused, with intent to kill and with the use of an unlicensed firearm, did then and there
willfully, unlawfully and feloniously attack, assault and shoot one JEFFREY WERNHER
GONZALES Y LIM on the head, thereby inflicting upon the latter serious and moral
gunshot wound which directly caused his death.
According to the prosecution, on January 1, 2005, at around 2:30 in the
afternoon, petitioner went to the office of Sykes Asia, Inc. located at the 25th Floor of
Robinson’s Summit Center, Ayala Avenue, Makati City. When petitioner was already
inside the building, he went to the work station of the deceased victim, Jeffrey Wernher
L. Gonzales (Jeffrey), who, by the configuration of the eyewitness Antonette
Managbanag’s sketch, was seated fronting his computer terminal, with his back towards
the aisle. As petitioner approached Jeffrey from the back, petitioner was already holding
a gun pointed at the back of Jeffrey’s head. Jeffrey managed to deflect the hand of
petitioner holding the gun, and a short struggle for the possession of the gun ensued
thereafter. Petitioner won the struggle and remained in possession of the said gun.
Petitioner then pointed the gun at Jeffrey’s face, pulled the trigger four (4) times, the
fourth shot finally discharging the bullet that hit Jeffrey in the forehead, eventually killing
him. Finally, after shooting Jeffrey, petitioner fled the office.
The defense claimed that on January 1, 2005, at around 2:30 in the afternoon,
more or less, petitioner, together with his children, went to Sykes Asia, the workplace of
his wife, Darlene Dela Cruz (Darlene), located at the 25th Floor of Robinson’s Summit
Building in Makati City, to fetch the latter so that their family could spend time and
celebrate together the New Year’s Day. Petitioner underwent the regular security
checkup/procedures. He was frisked by the guards-on-duty manning the main entrance
of said building and no firearm was found in his possession. He registered his name at
the security logbook and surrendered a valid I.D. Upon reaching 25th floor, petitioner
was frisked again and found no gun in his possession and was allowed to enter.
He was not able to find his wife on her table, so petitioner approached a certain man
(Jeffrey the deceased victim) and asked the latter as to the possible whereabouts of
Darlene. After casually introducing himself as the husband of Darlene, Jeffrey curtly told
him, “Bakit mo hinahanap si Darlene?” to which he answered, “Nagpapasundo kasi sa
akin.” The response given by Jeffrey shocked and appalled petitioner: “Ayaw na nga ng
asawa mo sayo sinusundo mo pa!”
The petitioner being shocked asked Jeffrey who he was. But Jeffrey suddenly
cursed petitioner. Then, Jeffrey suddenly picked up something in his chair which
happened to be a gun and pointed the same at petitioner’s face followed by a clicking
sound. The gun, however, did not fire.
Seeing imminent danger to his life, petitioner grappled with Jeffrey for the possession of
the gun. While grappling, the gun clicked for two (2) to three (3) more times. Again, the
gun did not fire. Petitioner was able to wrest away the gun from Jeffrey and tried to run
away to avoid any further confrontation with the latter. However, Jeffrey immediately
blocked petitioner’s path and shouted, “Guard! Guard!” Immediately then, Jeffrey took
hold of a big fire extinguisher, aimed and was about to smash the same on petitioner’s
head. Acting instinctively, petitioner parried the attack while still holding the gun. While
in the act of parrying, the gun accidentally fired due to the reasonable force and contact
that his parrying hand had made with the fire extinguisher and the single bullet
discharged hit the forehead of Jeffrey, which caused the latter to fall on the floor and
die.
After said incident, Darlene abandoned petitioner and brought with her their two (2)
young children. Petitioner later learned that Darlene and Jeffrey had an illicit relationship
when he received a copy of the blog of Darlene, dated January 30, 2005, sent by his
friend.
During the trial of the case, the prosecution presented the oral testimonies of
Marie Antonette Managbanag (Manag banag), Maria Angelina Pelaez (Pelaez) and
Carlos Alberto Lim Gonzales (Gonzales), respectively. The prosecution likewise
formally offered several pieces of documentary evidence to support its claim.
For its part, the defense presented as witnesses, petitioner himself; his brother, Simeon
Sander Dela Cruz III (Cruz), Greg Lasmarias Elbanvuena (Elbanvuena) and
Managbanag, who was recalled to the witness stand as witness for the defense.
On February 26, 2007, the Regional Trial Court (RTC) of Makati City, Branch 147,
rendered a Decision5 finding petitioner guilty beyond reasonable doubt of the crime of
Homicide.
On March 28, 2007, petitioner filed a Notice of Appeal, while private respondent,
through the private prosecutor, filed a Notice of Appeal on April 11.
The court denied their motion for reconsideration, petitioner elevated the case to the
Court of Appeals (CA). However, the latter denied their appeal and affirmed the RTC
decision with modification on the civil liability of petitioner.
Hence, the present petition.

ISSUES:

1. Whether or not all the requisites of the justifying circumstance of self-defense,


as provided for by law and settled jurisprudence, are present in this case.
2. Whether or not the firing of the gun wherein only a single bullet was discharged
therefrom was merely accidental which occurred during the time that the petitioner-
appellant was still in the act of defending himself from the continuous unlawful
aggression of the deceased victim.
3. Whether or not the prosecution was able to prove all the essential elements
constituting the crime of homicide.
4. Whether or not the privileged mitigating circumstance of self-defense is applicable in
this case.
5. Whether or not petitioner-appellant may be held civilly liable for the death of the
victim arising from the accident that transpired.

RULING:

1. No. The essential requisites of self-defense are the following: (1) unlawful
aggression the part of the victim; (2) reasonable necessity of the means employed to
prevent or repel such aggression; and (3) lack of sufficient provocation.

The evidence on record does not support petitioner’s contention that unlawful
aggression was employed by the deceased-victim, Jeffrey, against him. There is
aggression, only when the one attacked faces real and immediate threat to his life.

In the case at bar, other than petitioner’s testimony, the defense did not adduce
evidence to show that Jeffrey condescendingly responded to petitioner’s questions or
initiated the confrontation before the shooting incident; that Jeffrey pulled a gun from his
chair and tried to shoot petitioner but failed — an assault which may have caused
petitioner to fear for his life.

Even assuming that the gun originated from Jeffrey and an altercation transpired, and
therefore, danger may have in fact existed, the imminence of that danger had already
ceased the moment petitioner disarmed Jeffrey by wresting the gun from the latter.

Petitioner’s contention that Jeffrey’s unlawful aggression was continuous and imminent
is, therefore, devoid of merit.

2. No. The court find it highly specious for petitioner to go through the process of
tussling and hassling with Jeffrey, and in the end, shooting the latter on the forehead,
not only once, but four times, the last shot finally killing him, if he had no intention to hurt
Jeffrey. The means employed by a person resorting to self-defense must be rationally
necessary to prevent or repel an unlawful aggression. The opposite was, however,
employed by petitioner, as correctly pointed out by the RTC, thus: The victim was
holding the fire extinguisher while the second was holding the gun. The gun and the
discharge thereof was unnecessary and disproportionate to repel the alleged
aggression with the use of fire extinguisher. The rule is that the means employed by the
person invoking self-defense contemplates a rational equivalence between the means
of attack and the defense. (People v. Obordo, 382 SCRA 98) It was the accused who
was in advantage position as he was armed with a gun, as against the victim who was
armed, so to speak, with a fire extinguisher, which is not a deadly weapon. Under the
circumstances, accused’s alleged fear was unfounded. The Supreme Court has ruled
that neither an imagined impending attack nor an impending or threatening attitude is
sufficient to constitute unlawful aggression (Catalina Security Agency v. Gonzales-
Decano, XMRconfronted by a real threat on his life and limb; and the peril sought to be
avoided is imminent and actual, not merely imaginary (Senoja v. People, 440 SCRA
695).

3. Yes. The court said that there was no question that petitioner authored the death
of the deceased-victim, Jeffrey since the Prosecution’s eyewitnesses were consistent in
declaring that while there was prior struggle for the possession of the gun, it was
nevertheless accused who was holding the gun at the time of the actual firing thereof.

4. No. As what the court said, the three essential requisites of self defense were not
present. Therefore, self-defense in this case is void as a mitigating circumstance
making it invalid in this case.

5. Yes. Court cited its previous ruling in Nacar v. Gallery Frames, an interest of six
percent (6%) per annum on the aggregate amount awarded for civil indemnity and
damages for loss of earning capacity shall be imposed, computed from the time of
finality of the decision of court until full payment thereof. The petition then is DENIED.
Finding petitioner Sherwin Dela Cruz guilty beyond reasonable doubt of the crime of
Homicide. The court AFFIRMED with MODIFICATIONS, to wit;
(1) Petitioner shall be sentenced to an indeterminate penalty of from ten (10) years and
one (1) day of prisión mayor maximum, as the minimum penalty, to seventeen (17)
years, four (4) months and one (1) day of reclusion temporal maximum, as the
maximum penalty;
(2) Petitioner is likewise ORDERED to pay the heirs of the victim the following:
a. the amount of P50,000.00 as civil indemnity;
b. the amount of P50,000.00 as moral damages;
c. the amount of P25,000.00 as temperate damages;
d. the amount of P30,000.00 as exemplary damages;
e. the amount of P3,022,641.71 as damages for loss of earning capacity;
f. for the civil indemnity and the damages for loss of earning capacity, an interest of six
percent (6%) per annum, computed from the time of finality of this Decision until full
payment thereof; and
g. the costs of the litigation.

- People vs. Gonzales, 672 SCRA 590 (2012)


Nacnac vs. People, 668 SCRA 846 (2012)

Nacnac vs. People

Facts:
· On February 20, 2003, SPO2 Nacnac (accused-appellant), the SPO1 Doddie Espejo
(victim) and several other police officers were on duty. Nacnac, being the highest-ranking officer
during the shift, was designated the officer-of-the-day.
· Shortly before 10:00 in the evening, Espejo, together with then SPO1 Basilio, took the
patrol tricycle from the station grounds. When Nacnac saw this, he stopped the victim and his
colleague from using the tricycle.
· Espejo told Nacnac that he needed it to go to Laoag City to settle a previous
disagreement with a security of a local bar. Nacnac still refused. He told Espejo that he is
needed at the station and, at any rate, he should stay at the station because he was drunk.
· This was not received well by Espejo. He told Nacnac in Ilocano: “Iyot ni inam kapi”
(Coitus of your mother, cousin!). Espejo alighted from the tricycle. SPO1 Basilio did the same,
went inside the office, and left Nacnac and Espejo alone.
· Espejo took a few steps and drew his .45 caliber gun which was tucked in a holster on the
right side of his chest. Nacnac then fired his M-16 armalite upward as a warning shot.
Undaunted, Espejo still drew his gun. Nacnac then shot the victim on the head, which caused
the Espejo’s instantaneous death.
· Nacnac later surrendered to the stations Chief of Police.
· The RTC found Nacnac guilty beyond reasonable doubt of the crime of homicide. It held
that the claim of self-defense by Nacnac was unavailing due to the absence of unlawful
aggression on the part of Espejo.
· On appeal, CA affirmed the findings of RTC. It held that the essential and primary
element of unlawful aggression was lacking. It gave credence to the finding of the trial court that
no one else saw the victim drawing his weapon and pointing it at accused Nacnac.
· In its comment dated April 27, 2011, the OSG avers that petitioner is entitled to an
acquittal, or at the very least, not one but two mitigating circumstances.
Issue: Whether or not the justifying circumstances of the petitioner’s acts constitutes a valid self-
defense.

Held: YES.

LEGAL BASIS

Article 11 of the Revised Penal Code states that anyone who acts in defense of his person or
rights do not incur any criminal liability, provided that the following circumstances concur:
(i) unlawful aggression;
(ii) reasonable necessity of the means employed to prevent or repel it; and
(iii) lack of sufficient provocation on the part of the person defending himself.
APPLICATION OF LEGAL BASIS
Unlawful aggression is an indispensable element of self-defense; and ordinarily, there is a
difference between the act of drawing one’s gun and the act of pointing one’s gun at a target in
determining the presence of unlawful aggression. The former cannot be said to be an unlawful
aggression on the part of the victim, while the latter is generally considered unlawful aggression.
Here, a warning shot fired by fellow police officer (petitioner) was left unheeded as the victim
reached for his own firearm and pointed it at petitioner. Petitioner was justified in defending
himself from an inebriated and disobedient colleague.

As to the second circumstance above, the nature and number of wounds inflicted by the
accused are constantly and unremittingly considered as important indicia of the means
employed by the accused, which must be reasonably commensurate to the nature and the
extent of the attack sought to be averted. Here, the lone gunshot was a reasonable means
chosen by petitioner in defending himself in view of the proximity of the armed victim, his
drunken state, disobedience of an unlawful order, and failure to stand down despite a warning
shot.

There is also lack of sufficient provocation on the part of the person defending himself or herself
in this case. Petitioner gave the victim a lawful order and fired a warning shot before shooting
the armed and drunk victim. There was no evidence on petitioner sufficiently provoking the
victim prior to the shooting. Petitioner was only defending himself on the night he shot his fellow
policer.
DISPOSITIVE PORTION

WHEREFORE, petitioner's Motion for Reconsideration is GRANTED. The CA Decision dated


July 20, 2009 in CA-G.R. CR-H.C. No. 30907 is REVERSED and SET ASIDE. Petitioner SPO2
Lolito T. Nacnac is ACQUITTED of homicide on reasonable doubt. The Director of the Bureau of
Prisons is ordered to immediately RELEASE petitioner from custody, unless he is being held for
some other lawful cause, and to INFORM this Court within five (5) days from receipt of this
Decision of the date petitioner was actually released from confinement.

- People vs. Sotelo, G.R. No. L-33304, Dec. 13, 1930

G.R. No. L-33304             December 13, 1930


THE PEOPLE OF THE PHILIPPINE ISLANDS, plaintiff-appellee, vs. CONSTANTE
SOTELO, ET AL., defendants. CONSTANTE SOTELO, appellant.
Alberto Reyes for appellant.
Attorney-General Jaranilla for appellee.

VILLAMOR, J.:

The Sotelo brothers, namely, Constante, Dominador, and Vicente, were prosecuted in the Court
of First Instance of Ilocos Sur for the crime of homicide under the following information:

That on or about the night of December 24, 1929, in the municipality of Narvacan,
Province of Ilocos Sur, Philippine Islands, the said accused Constante, Dominador, and
Vicente Sotelo, armed with a penknife, a stick, and an iron bar, respectively, acting
together and helping one another, did willfully, maliciously, unlawfully, and feloniously
with treachery and evident premeditation attack, beat up, and commit assault upon the
person of Ignacio Cambaliza, inflicting a mortal wound upon him on the level of the left
nipple, which penetrated the left lung and the left ventricle of the heart, another on the
outward surface of the right arm, a bruise on the nose and another on the upper lip: as a
result of which said Ignacio Cambaliza died after a few minutes.
Contrary to law; with the aggravating circumstance of abuse of superior strength.

After the trial, the court below found the defendant Constante Sotelo guilty of the crime of
homicide, and the defendants Vicente and Dominador Sotelo of slight physical injuries,
sentencing the former to suffer twelve years and one day of reclusion temporal, to indemnify the
heirs of the deceased in the sum P1,000, with the accessories of law, and to pay one-third of
the costs; and ordering the release of Vicente and Dominador Sotelo in view of the fact that they
had already been imprisoned since December 24, 1929, with two-thirds of the costs de oficio.

The defendant Constante Sotelo appealed from this judgment, and his counsel has made the
following assignments of error:

I. The trial court erred in accepting the whole theory of the prosecution:

Despite the obvious incongruity between the information and the evidence adduced at
the trial;lawphi1>net

Despite the proof that the iron bar, Exhibit B of the prosecution and Exhibit 1 of the
defense, belong to the principal witness, Baltazar Capistrano, and not to any of the
defendants;

Despite the fact that it has been proved that the incident took place in the yard of the
defendants' house and not on the public road;

Despite the fact that Baltazar Capistrano deliberately concealed the place where the
deceased expired, fearing his participation in the tragedy should come to light; and

Despite the fact that the chief of police acted with evident partiality towards Cambaliza
and Capistrano, in intervening immediately after the incident had occurred, hiding the
whip or riding crop, which the other local authorities found hanging from the right arm of
the deceased, and for the other reasons.

II. The trial court erred in finding the following facts: that Dominador Sotelo hit the
deceased across the mouth with the crop Exhibit B, Vicente Sotelo stabbed him on the
right shoulder with the penknife Exhibit D, and Constante Sotelo stabbed him under the
nipple.

III. The trial court erred in refusing to consider the plea of self-defense alleged and
proved by the defendant-appellant, Constante Sotelo, notwithstanding the fact that said
defense is strongly corroborated by Exhibits O, 5, and 4, which form a part of the res
gestae.

IV. The lower court erred in refusing to consider the motion of February 1, 1930,
declaring afterwards that, as the defendant has shown graphically, the latter could not
have touched the victim's left breast in stabbing him, or the wound would not have been
in the direction described in Dr. Nolasco's certificate; in spite of the fact that it had made
contrary declarations during the trial, which estop it from making the subsequent
holdings.
V. The trial judge erred in convicting the defendant appellant, Constante Sotelo.

The record shows that at about 8 o'clock in the evening of the 24th of December, 1929, Ignacio
Cambaliza started for the barrio of Ravadabia, in the municipality of Narvacan, Ilocos Sur,
accompanied by Baltazar Capistrano. They took the provincial road leading to said barrio and
as they approached the Sotelo house, they bid the time of day, asking whether they might pass
by, according to the custom of the place. They were barely 20 meters away from the house,
when the defendant Constante Sotelo, who had just finished his supper, descended from the
house towards the road, and, from the entrance of his yard, turned his flashlight on the passers-
by to see who they were. When Ignacio Cambaliza saw this, he walked back to where
Constante Sotelo stood and inquired why he turned his flashlight on them, and what it was he
wanted, winding up with a vulgar remark. When Constante's brothers, who were then in the yard
on the side of the road, saw Cambaliza's attitude, they approached their brother to separate or
defend him, whereupon Cambaliza's commenced beating them with his iron crop, once striking
Constante's arm. The brothers, in turn, fell upon Cambaliza, Dominador striking him across the
face with the stick he carried, and Vicente wounding him in the right shoulder with a penknife. At
this juncture, Capistrano attempted to intervene, but he was warned by Vicente and probably by
Dominador also, for which reason he withdrew from the scene, and the fight then continued
between Cambaliza and Constante. In the course of this fight, Constante thrust a penknife into
Cambaliza at about the level of the left nipple, producing a wound which penetrated the left lung
into the left ventricle of the heart, resulting in his death a few minute later.

Witness Baltazar Capistrano, who was with the deceased, says that after Ignacio Cambaliza
had fallen lifeless, he went over to the municipal building to ask for help, and at once the chief of
police, the justice of the peace, and the municipal president repaired to the place where the
incident had occurred, placing the brothers under arrest that same night: Constante with a
penknife, Vicente with another penknife, and Dominador with a cane.

Doctor Antonio Nolasco examined Ignacio Cambaliza's body, and found a knife wound at about
the level of the left nipple, 2 1/2 centimeters long, and 2 1/2 inches deep; a knife wound on the
outward surface of the right arm, 4 centimeters deep; and bruises at the base of the nose and
on the upper lip, produced by a blunt instrument. According to the doctor the breast wound
which pierced the left lung and the left ventricle of the heart was the cause of Ignacio
Cambaliza's death.

The appellant admits he inflicted the injury which resulted in Ignacio Cambaliza's death, but
maintains he did so in self-defense. In support of this allegation it is insisted that the fight took
place in the yard of the defendants' house; that Vicente and Dominador went to help their
brother Constante when they saw Cambaliza attack him, but ran behind some sugar cane near
by in order to conceal themselves when pursued by Cambaliza; that the latter fought hand to
hand with Constante, choked him and threw himself upon him, and at that instant Constante
thrust a penknife into his ribs below the left nipple; that Capistrano answered Cambaliza's call,
saying that Constante had stabbed him, and, with the assistance of Capistrano, Cambaliza
succeeded in leaving the Sotelo yard and after a few steps fell lifeless on the roadside, where
his body was later found.

After examining the evidence of record, we believe the defense is, in a measure, supported by
the testimony of Baltazar Capistrano given before the justice of the peace of Narvacan. In
considering this proof it is well to remember that the justice of the peace of Narvacan testified in
the case that Baltazar Capistrano made two statements before him — in the investigation prior
to the arrest of the defendants, which is Exhibit 9, and in the course of the preliminary
investigation, embodied in Exhibit 10. Counsel for the defense attempted to examine Capistrano
on these two statements, but the fiscal objected, and the court sustained the objection on the
ground that the best evidence would be Capistrano's own statements taken down in writing.
Counsel then required the fiscal to present said documents, and the latter delivered to him the
aforementioned Exhibits 9 and 10, which were offered in evidence by the defense. The fiscal
reiterated his objection to that evidence on the ground that the documents were not identified;
but the record shows that they had been delivered by the fiscal himself to counsel for the
defense, and he is therefore precluded from setting up the lack of identification, whereupon the
court doubtless ruled them in, and the fiscal failed to take exception therefrom.

The record further shows that counsel for the appellant sought to have the witness Capistrano
explain the contradiction between his statement in Exhibit 10 and his testimony at the hearing,
but the fiscal objected and the court sustained the objection. Capistrano has thus failed to
explain the contradiction noted between Exhibit 10 and his testimony before the trial court.

We believe the trial judge erred in sustaining the fiscal's objection to having witness Capistrano
explain the contradiction between his statement in Exhibit 10, and his testimony before the trial
court at the hearing. But be that as it may, we are of opinion that said documents Exhibits 9 and
10 have been duly introduced into the case as evidence for the defense and must therefore be
taken into consideration in rendering judgment.

In said Exhibit 10, witness Capistrano, among other things, affirms the following:

As we passed by Constante was flashlighting us and he was standing by the door of


their yard. I did not see any body by him. We were then about 20 meters away from him
when he rushed to us.

Ignacio was the first one who uttered bad words against Constante. Ignacio had a whip
wrapped with lead.

Other than this time I declared before the justice of the peace.

As Dominador came he struck Ignacio, but Ignacio defended. Vicente struck and
Constante rushed in and then they wrestled against each other and I tried to separate
them.

I was only 5 meters away from them when they first wrestled. It was dark at that time but
I saw what happened by the aid of my flashlight.

I am very sure that Exhibit A was the bar which Vicente was holding and Exhibit B was
the knife that Constante used in stabbing Ignacio.

While they were wrestling Constante was under Ignacio during which time, I saw
Constante bring out his knife.

The same witness testified before the trial court as follows:


Q. Do you know whether anything extraordinary took place along the road? — A. Yes
sir; for when we came near the house of Francisco Sotelo, we saw Constante,
Dominador, and Vicente Sotelo in the yard of the house. I and my companion said "we
are passing, sir," but they did not answer, and they focused their flashlight on us. And
when we came to within ten meters, they still kept the flashlight focused on us, and then
my companion said: "Why do you turn your flashlight on us? Have we not greeted you in
passing?"

Q. What did the Sotelo brothers, Constante, Dominador, and Vicente do when they
heard Ignacio Cambaliza say this? — A. After Ignacio Cambaliza had said that, they
switched off the light, and we continued on.

Q. What else? — A. After we had proceeded about 20 meters, someone came up


behind us with a flashlight saying: "Wait! your mother's . . .! you cannot say bad words
when you pass here."

Q. What else? — A. When they had said this, Ignacio Cambaliza stopped and said: "I
have used no bad word."

Ignacio Cambaliza interrupted Constante Sotelo saying "I did not say that." And Dominador, in
turn, said, "You didn't say anything, your mother's . . .!"

Q. And what did Dominador Sotelo do then? — A. Immediately after saying, "you didn't
say anything, your mothers . . .! he struck him across the face with a stick.

x x x           x x x          x x x

Q. And what else happened? — A. After that blow delivered by Dominador Sotelo, he
was stunned. While he was so stunned, Vicente Sotelo stabbed him with a penknife on
the right shoulder; and Ignacio Cambaliza, still stunned, turning about; and Constante
Sotelo stabbed him in the region of the heart with a penknife.

Comparing the two statements quoted above, it will be seen that the witness Capistrano
affirmed in Exhibit 10 that he saw Constante standing at the entrance of his yard, alone; that the
deceased was the first to make vulgar remarks to Constante; and that during the fight Constante
was under Cambaliza when he drew his penknife to stab him. On the other hand, testifying
before the trial court, the said witness Capistrano stated that he and the deceased saw the three
brothers, Constante, Dominador, and Vicente Sotelo in the yard of the house; that one of these
brothers was the first to make insulting remarks to Cambaliza, and that while the two brothers
were attacking Cambaliza, Constante stabbed him in the chest with a penknife.

We believe Capistrano's testimony appearing in Exhibit 10 as to Constante's position when he


wounded Cambaliza, must be accepted, not only because it was given two days after the
incident, but because it has been corroborated by the witnesses for the defense. (U.
S. vs. Capisonda, 1 Phil., 575; and U. S. vs. Rafael, 23 Phil., 184.)

As to who started the aggression, there is an obvious contradiction between the testimony of
Capistrano and that of the witnesses for the defense. But in the light of sound judgment, we are
inclined to believe that the deceased started the aggression, provoked by the offensive
language used by Constante and his brothers, imputing to him the utterance of vulgar language
against them. In such a situation the deceased naturally used his whip against those who were
in front of him, striking Constante's arm. We therefore believe this is a case of incomplete self-
defense, wherein the appellant was unlawfully attacked by the deceased and compelled to
employ reasonable means to defend himself, but he is responsible for provoking the attack. (U.
S. vs. Ancheta, 1 Phil., 30; U. S. vs. McCray, 2 Phil., 545.) According to article 86 of the Penal
Code the penalty next below that provided in article 404 of said Code must be imposed upon
the appellant, that is, prision mayor in its minimum degree, or six years and one day, with the
accessories of law, and to indemnify the family of the deceased in the amount of P500. And with
this modification the judgment appealed from is affirmed in all other respects, with costs against
the appellant. So ordered.

Johnson, Street, Malcolm, Ostrand, Johns and Villa-Real, JJ., concur

People vs. Gutierrez, 611 SCRA 633

PEOPLE OF THE PHILIPPINES, appellee, vs. FORD GUTIERREZ y DIMAANO, appellant.


G.R. No. 188602. February 4, 2010.
Ponente: NACHURA, J.

FACTS:

Ford Gutierrez admitted having killed Regis and wounding Dalit insists to reverse the
decision of the Court for the crime of murder, frustrated murder and (3) counts of attempted
murder for he asserted that is was an act of self-defense and that he contended that the Court
gravely erred in convicting him of the crimes charged when his guilt has not been proven
beyond reasonable doubt and the court only gave weight to the testimonies of the prosecution
eyewitness.

ISSUE:
Whether or not the requisites to prove the claim of self-defense by the herein appellant is
present in the case at bar.

FACTS:

According to the Court, self-defense is an affirmative allegation and offers exculpation from
liability for crimes only if satisfactorily proved. It requires (a) unlawful aggression on the part of
the victim; (b) reasonable necessity of the means employed by the accused to repel it; and (c)
lack of sufficient provocation on his part.

In the case at bar, the appellant utterly failed to discharge the burden of proving unlawful
aggression. His version of the events was uncorroborated, and his testimony was found to be
less credible by the trial court. As a rule, the appellate court gives full weight and respect to the
determination by the trial court of the credibility of witnesses, since the trial judge has the best
opportunity to observe their demeanor.

The Court also agrees with the trial court in appreciating treachery as a qualifying circumstance.
The essence of treachery is the sudden and unexpected attack by the aggressor on
unsuspecting victims, depriving the latter of any real chance to defend themselves, thereby
ensuring its commission without risk to the aggressor, and without the slightest provocation on
the part of the victims.

The pieces of evidence gleaned by the trial court, the facts, are enough to show that treachery
was employed by appellant. The attack was sudden, as testified to by the witnesses, and
unexpected. Provocation on the part of the victims was not proven, and appellant's testimony
that the victims were about to attack him cannot be given credence. The victims had no inkling
that an attack was forthcoming and had no opportunity to mount a defense. Thus, treachery was
correctly appreciated as a circumstance to qualify the crime to murder.

The Court ruled that, the appellant is found guilty beyond reasonable doubt for the crime of
murder and (4) counts of attempted murder.

- People vs. Manulit, 635 SCRA 426

People vs. Dennis Manulit


G.R. No. 192581

FACTS:

Appellant, Dennis Manulit was charged in a crime of murder for shooting Reynaldo Juguilon with
his live-in partner, Anabel Bautista.
On July 6, 2003, Juguilon and Bautista were walking along Dagupan Extension, Tondo, Manila
on their way home when they passed by Manulit, who was then sitting in front of his house
across the barangay hall. Manulit stood up and shot Juguilon at the back resulting in his death.
Manulit offered a story of self-defense.
In his testimony, he stated that while he and his cousin were drinking at the ground floor of his
house, the victim suddenly barged in and poked a gun at him. They grappled for the gun and
when he was able to obtain possession of it, the victim opened a fan-knife. This resulted in his
act of shooting down the victim.

ISSUE:

Whether or not Manulit acts in self-defense?

HELD:
No, the accused-appellant did not act in self-defense because he failed to prove the existence
of unlawful aggression. Unlawful aggression is an actual physical assault, or at least a threat to
inflict real imminent injury, upon a person. He wants the court to believe that the victim was the
aggressor, not him.
The Court is not convinced. Therefore, since no unlawful aggression was present, the accused-
appellant cannot successfully invoke self-defense. Dennis Manulit is guilty of the crime charged.

Other Version/ Case Digest

PEOPLE OF THE PHILIPPINES, plaintiff-appellee,


vs. DENNIS D. MANULIT, accused-appellant
G.R. No. 192581. November 17, 2010.
Ponente: VELASCO, JR., J.

FACTS:
Accused Dennis Manulit shot Reynaldo Juguilon while the latter was walking with his
live-in partner Anabel. The shot resulted in Reynaldo’s death.
He then tucked the gun in his waist, raised his hands and shouted “wala akong
ginawang kasalanan at wala kayong nakita.” And he ran towards the basketball court adjoining
the barangay hall. Lydia Juguilon, Manulit’s aunt and the victim’s sister in law, saw what
happened but kept quiet about it until bothered by her conscience, she decided to issue a
statement before the prosecutor of Manila. Manulit offered a story of self-defense.

ISSUE:
Whether or not the accused act in self-defense?
RULING:
No the accused did not act in self-defense because he failed to prove the existence of
unlawful aggression which is one of the requisite to prove the claim of self-defense under article
11 of RPC.
RTC found him guilty of murder. The CA affirmed. The SC ruled that in the instant case,
Manulit failed to prove the existence of unlawful aggression. In addition, there was treachery
because the victim was only walking in the street when the accused-appellant suddenly shot
him at the back several times. He had no opportunity to defend himself, because he had no idea
that an attack was forthcoming. It likewise appears that the means was deliberately planned.
What is decisive is that the attack was executed in a manner that the victim was rendered
defenseless and unable to retaliate. Evidently, treachery attended the killing. Noteworthy also is
the fact that the accused-appellant held grudge against the victim since the victim filed a case
against the accused-appellant before the office of the city prosecutor.

- People v. Narvaez, GR L-33466-67

PEOPLE v. MAMERTO NARVAEZ, GR Nos. L-33466-67, 1983-04-20


Facts:
This is an appeal from the decision of the Court of First Instance of South Cotabato... for murder
which, after a joint trial, resulted in the conviction of the accused in a decision rendered on
September 8, 1970... finding Mamerto Narvaez guilty beyond reasonable doubt
"At about 2:30 in the afternoon of August 22, 1968, Graciano Juan, Jesus Verano and Cesar
Ibañez, together with the two deceased Davis Fleischer and Flaviano Rubia, were fencing the
land of George Fleischer, father of deceased Davis Fleischer. The place was in the boundary
of... the highway and the hacienda owned by George Fleischer. This is located in the
municipality of Maitum, South Cotabato. At the place of the fencing is the house and rice drier of
appellant Mamerto Narvaez
.
om getting into his house and the bodega of his ricemill. So he addressed the group, saying-
'Pare, if possible you stop destroying my house and if possible we will talk it over - what is
good,' addressing the deceased Rubia, who is appellant's compadre. The deceased Fleischer,
however, answered: 'No, gademit, proceed, go ahead.' Appellant apparently lost his...
equilibrium and he got his gun and shot Fleischer, hitting him. As Fleischer fell down, Rubia ran
towards the jeep, and knowing there is a gun on the jeep, appellant fired at Rubia, likewise
hitting him
. Both Fleischer and Rubia died as... a result of the shooting
Appellant now questions the propriety of his conviction, assigning the following errors:
"First Assignment of Error: That the lower court erred in convicting defendant-appellant despite
the fact that he acted in defense of his person; and
"Second Assignment of Error: That the court a quo also erred in convcting defendant-appellant
although he acted in defense of his rights" (p. 20 of Appellant's Brief, p. 145, rec.).
The act of killing of the two deceased by appellant is not disputed. Appellant admitted having
shot them from the window of his house with the shotgun which he surrendered to the police
authorities. He claims, however, that he did so in defense of his person and of his... rights, and
therefore he should be exempt from criminal liability.
l liability.
Defense of one's person or rights is treated as a justifying circumstance under Art. 11, par. 1 of
the Revised Penal Code, but in order for it to be appreciated, the following requisites must
occur:
"First. Unlawful aggression;
"Second. Reasonable necessity of the means employed to prevent or repel it;
"Third. Lack of sufficient provocation on the part of the person defending himself" (Art. 11, par.
1, Revised Penal Code, as amended).
Issues:
Appellant now questions the propriety of his conviction, assigning the following errors:
"First Assignment of Error: That the lower court erred in convicting defendant-appellant despite
the fact that he acted in defense of his person; and
"Second Assignment of Error: That the court a quo also erred in convcting defendant-appellant
although he acted in defense of his rights" (p. 20 of Appellant's Brief, p. 145, rec.).
Ruling:
Defense of one's person or rights is treated as a justifying circumstance under Art. 11, par. 1 of
the Revised Penal Code, but in order for it to be appreciated, the following requisites must
occur:
"First. Unlawful aggression;
"Second. Reasonable necessity of the means employed to prevent or repel it;
"Third. Lack of sufficient provocation on the part of the person defending himself" (Art. 11, par.
1, Revised Penal Code, as amended).
Other Version

People vs. Narvaez, 121 SCRA 389 (1983)

FACTS:
Mamerto Narvaez has been convicted of murder (qualified by treachery) of David Fleischer and
Flaviano Rubia. On August 22, 1968, Narvaez shot Fleischer and Rubia during
the time the two were constructing a fence that would prevent Narvaez from getting into his
house and rice mill. The defendant was taking a nap when he heard sounds of construction and
found fence being made. He addressed the group and asked them to stop destroying his house
and asking if they could talk things over. Fleischer responded with "No, gadamit, proceed, go
ahead." Defendant lost his "equilibrium," and shot Fleisher with his shotgun. He also shot Rubia
who was running towards the jeep where the deceased's gun was placed. Prior to the
shooting, Fleischer and Co. (the company of Fleischer's family) was involved in a legal battle
with the defendant and other land settlers of Cotabato over certain pieces of property. At the
time
of the shooting, the civil case was still pending for annulment (settlers wanted granting of
property to Fleisher and Co. to be annulled). At time of the shooting, defendant had leased his
property from Fleisher (though case pending and ownership uncertain) to avoid trouble. On
June 25, defendant received letter terminating contract because he allegedly didn't pay rent.
He was given 6 months to remove his house from the land. Shooting was barely 2 months after
letter. Defendant claims he killed in defense of his person and property. CFI ruled that
Narvaez was guilty. Aggravating circumstances of evident premeditation offset by the mitigating
circumstance of voluntary surrender. For both murders, CFI sentenced him to reclusion
perpetua, to indemnify the heirs, and to pay for moral damages.

ISSUES:

1. Whether or not CFI erred in convicting defendant-appellant despite the fact that he acted in
defense of his person.

No. The courts concurred that the fencing and chiselling of the walls of the house of the
defendant was indeed a form of aggression on the part of the victim. However, this
aggression was not done on the person of the victim but rather on his rights to property. On the
first issue, the courts did not err. However, in consideration of the violation of property rights, the
courts referred to Art. 30 of the civil code recognizing the right of owners to close and fence their
land.

Although is not in dispute, the victim was not in the position to subscribe to the article because
his ownership of the land being awarded by the government was still pending, therefore putting
ownership into question. It is accepted that the victim was the original aggressor.

2. WON the court erred in convicting defendant-appellant although he acted in defence of his
rights.

Yes. However, the argument of the justifying circumstance of self-defense is applicable only if
the 3 requirements are fulfilled. Art. 11(1) RPC enumerates these requisites:
 Unlawful aggression. In the case at bar, there was unlawful aggression towards
appellant's property rights. Fleisher had given Narvaez 6 months and he should have left him in
peace before time was up, instead of chiseling Narvaez's house and putting up fence. Art. 536
of the Civil Code also provides that possession may not be acquired through force or
intimidation; while Art. 539 provides that every possessor has the right to be respected in his
possession
 Reasonable necessity of means employed to prevent or repel attack. In the case, killing
was disproportionate to the attack.
 Lack of sufficient provocation on part of person defending himself. Here, there was no
provocation at all since he was asleep
Since not all requisites present, defendant is credited with the special mitigating circumstance of
incomplete defense, pursuant to Art. 13(6) RPC. These mitigating circumstances are: voluntary
surrender and passion and obfuscation (read p. 405 explanation) Crime is homicide (2 counts)
not murder because treachery is not applicable on account of provocation by the deceased.
Also, assault was not deliberately chosen with view to kill since slayer acted instantaneously.
There was also no direct evidence of planning or preparation to kill. Art. 249 RPC: Penalty for
homicide is reclusion temporal. However, due to mitigating circumstances and incomplete
defense, it can be lowered three degrees (Art. 64) to arrestomayor.

3. WON he should be liable for subsidiary imprisonment since he is unable to pay the civil
indemnity due to the offended party.

No. He is not liable to be subsidiarity imprisoned for nonpayment of civil indemnity. RA 5465
made the provisions of Art. 39 applicable to fines only and not to reparation of damage caused,
indemnification of consequential damages and costs of proceedings. Although it was enacted
only after its conviction, considering that RA 5465 is favorable to the accused who is not a
habitual delinquent, it may be given retroactive effect pursuant to Art. 22 of the RPC.

Judgment: Defendant guilty of homicide but w/ mitigating circumstances and extenuating


circumstance of incomplete self defense. Penalty is 4 months arresto mayor and to indemnify
each group of heirs 4,000 w/o subsidiary imprisonment and w/o award for moral damages.
Appellant has already been detained 14 years so his immediate release is ordered.

Gutierrez, dissenting. Defense of property can only be invoked when coupled with form of
attack on person defending property. In the case at bar, this was not so. Appellant should then
be sentenced to prision mayor. However, since he has served more than that, he should be
released.

Defense of relatives
- People vs. Agacer, 662 SCRA 461 (2011)

G.R. No. 177751 December 14, 2011


PEOPLE OF THE PHILIPPINES, Appellee,
vs.
FLORENCIO AGACER, EDDIE AGACER, ELYNOR AGACER, FRANKLIN AGACER and
ERIC* AGACER, Appellants.
DEL CASTILLO, J.:

FACTS:

That on or about April 2, 1998, in the municipality of Sta. Ana, Province of Cagayan, and within
the jurisdiction [of] this Honorable Court, the above-named accused, armed with a long firearm,
a bow and arrow, a bolo and stones, with intent to kill, with evident premeditation and with
treachery, conspiring together and helping one another, did then and there wilfully, unlawfully
and feloniously assault, attack, stone and shoot one Cesario Agacer, inflicting upon the latter
[bruises] and multiple gunshot wounds in his body which caused his death.

According to prosecution witnesses Genesis and Roden, it was at that moment while Cesario
was tending to his farm when appellants suddenly emerged from a nearby banana plantation
and surrounded Cesario. Visibly intimidated, Cesario moved backwards and retreated to where
the other farm laborers were working. However, Franklin set afire the rice straws that covered
Cesario’s rice seedlings. This prompted Cesario to return to put out the fire and save his rice
seedlings. At this point, Franklin and Eric started throwing stones at Cesario which forced the
latter to retreat again. Thereafter, Florencio, while standing side by side with Eric, signaled
Cesario to come closer. Cesario obliged but when he was just around five meters away from the
group, Eddie suddenly pulled out a gun concealed inside a sack and, without warning, shot
Cesario hitting him in the left portion of his chest. Almost simultaneously, Elynor took aim at
Cesario with his bow and arrow but missed his mark. As Cesario fell, appellants fled towards the
irrigation canal, where another gunshot rang. Thereafter, a short firearm was thrown from where
the appellants ran towards the direction of Cesario’s fallen body. Appellants then immediately
left the scene of the crime onboard a hand tractor and a tricycle.

The appellants denied the accusations against them and claimed that Florencio only
acted in self-defense and in defense of relatives. As proof, appellants presented Florencio
who testified that on April 2, 1998, he proceeded to Dungeg, Sta. Ana, Cagayan, from his
residence in Merde, also in Sta. Ana, Cagayan, to prepare seed beddings in the ricefield over
which he and his uncle Cesario had an existing dispute. At around 8:00 a.m., he claimed that
Cesario attempted to prevent him from preparing the seed beds. When Florencio persisted and
argued that he inherited the land from his father, Cesario departed through a cogonal area.
Moments later, Cesario returned and shouted at him not to continue working on the land. At that
time, Florencio noticed that Cesario was holding an object. Suspecting that Cesario may be
armed, he shouted to Eric, Franklin, Eddie and Elynor, who had just arrived, to run away. The
four heeded his warning and scampered in different directions. Cesario then chased Florencio
who ran and jumped into the irrigation canal to hide in the tall cogon grasses. However, Cesario
was not deterred and continued to search for him. When Florencio saw that Cesario was
already close, he suddenly grabbed Cesario’s buckshot gun and successfully disarmed him.
Thereupon, Cesario drew another firearm and shot Florencio several times. As Cesario was
shooting him, Florencio also fired the gun he earlier grabbed from Cesario and hit the latter.
Finding out that he too was hit in the arm, he shouted to his nephews for help. They responded
by taking him to a hospital for treatment. On April 16, 1998, he went to the police to surrender.
ISSUE:
· W/N the appellants’ contention of self defenses do have merit?

HELD:
· While it is the burden of the prosecution to establish the guilt of the accused beyond
reasonable doubt, this burden shifts when the accused admits the killing and pleads self-
defense by way of justification. It therefore becomes vital for the accused to show clear and
convincing evidence that he acted in self-defense. In so doing, he must rely on the strength of
his own evidence and not on the weakness of the prosecution’s evidence.38
The accused must also prove the following elements of self-defense: (1) there was
unlawful aggression on the part of the victim; (2) there was reasonable necessity of the
means employed to prevent or repel the attack; and (3) the lack of sufficient provocation
on the part of the person defending himself. 39 In the justifying circumstance of self-defense,
unlawful aggression is a condition sine qua non.40 Self-defense, complete or incomplete, cannot
be considered a justification, unless the victim commits an unlawful aggression against the
person defending himself.41

Here, Florencio failed to prove that he defended himself against the unlawful aggression of
Cesario. He failed to present any evidence to substantiate his claim that there was an actual or
imminent peril to his life or limb. Aside from his unreliable and self-serving claim, there is no
proof that Cesario assaulted and shot him with a firearm during their struggle or, if at all, that
there was indeed a struggle between them. On the other hand, the separate testimonies of
prosecution witnesses Genesis and Roden negate Florencio’s claim of unlawful aggression. The
testimonies of these witnesses established that it was the appellants who emerged from a
nearby banana plantation; that they surrounded Cesario and set to fire the rice straws covering
his rice seedlings; that appellants were armed with different kinds of weapons, while Cesario
was not; that Franklin and Elynor cast stones upon Cesario; and, that the one who pulled a gun
from a sack and shot Cesario was Eddie, not Florencio. We thus hold that if there was unlawful
aggression here, it came from appellants’ end and not from Cesario. Hence, there being no
unlawful aggression on the part of Cesario, Florencio’s claim of self-defense must fail.

RULING:

WHEREFORE, the Court AFFIRMS the November 17, 2006 Decision of the Court of Appeals in
CA-G.R. CR-H.C. No. 01543 which affirmed the August 7, 2001 Decision of the Regional Trial
Court, Branch 8, Aparri, Cagayan, finding appellants Florencio, Franklin, Elynor, Eddie and Eric,
all surnamed Agacer, guilty beyond reasonable doubt of the crime of murder, with the
following modifications:

(1) actual damages is DELETED;


(2) the appellants are ORDERED to pay the heirs of Cesario Agacer P25,000.0 as temperate
damages; and
(3) the appellants are ORDERED to pay the heirs of Cesario Agacer interest at the legal rate of
six percent (6%) per annum on all the amounts of damages awarded, commencing from the
date of finality of this Decision until fully paid.
SO ORDERED.[3]
Costs against appellants.
- People vs. Lopez, 585 SCRA 529

G.R. No. 177302. April 16, 2009


PEOPLE OF THE PHILIPPINES, appellee, vs. JAIME
LOPEZ, ROGELIO REGALADO, AND ROMEO ARAGON, appellants.
CARPIO-MORALES, J.:

FACTS:

Appellant Rogelio Regalado who was outside a tailoring shop, called out to victim,
Edencito Chu and prompted him to come out of his mother’s bakery. Chu thereupon emerged
from the bakery, put his arms around Regalado’s shoulders and asked forgiveness. Regalado
however pushed his arms aside, drew a curved knife and stabbed Chu on the left nipple. As
Chu ran towards Villaluz Street, Regalado chased him and picked up two pieces of firewood
along the way with which he hit Chu. Appellant Jaime Lopez in the meantime surfaced from the
back of the tailoring shop and also joined the chase. Soon appellant Aragon also surfaced from
the back of the tailoring shop and joined the chase. The three caught up with Chu. Aragon
boxed Chu, causing the latter to fall. He then kicked the victim. Lopez stabbed Chu several
times as Regalado looked on. When Chu was no longer moving, the three appellants left.

Regalados Claim: At 3:00 P.M. on April 25, 1996, after he bought a hotcake from the
hotcake stand of Angelina Aragon (Angelina), wife of appellant Aragon and daughter of
appellant Regalado, at the corner of Bandola and Villaluz streets, Chu approached and choked
him. He elbowed Chu and extricated himself. He then left but Chu pursued him as he
(Regalado) proceeded to Angelina’s house at the corner of España and Villaluz streets where
he hid for around two minutes. When he returned to the hotcake stand, his son-in-law appellant
Lopez summoned him, telling him “I have done something, you accompany me in going to the
police station because I am going to surrender.” He and Lopez thereupon boarded a tricycad
and repaired to the police station where Lopez surrendered, handed a knife to the police, and
was detained. As he (Regalado) was about to go home, he was restrained as he might be
waylaid by Chu. The following morning, he was detained because the police found him to have
participated in the killing of Chu.

Lopez supported the claim Regalados and admitted that the killing of Chu was his doing.
He said it was “defense of relative” and “self-defense” as he claims Chu is Threatening him by
saying, “Are you going to defend your father-in-law?”

Aragon invoked alibi, claiming that at 3:00 P.M. of April 25, 1996, he went to the wharf
which is 40 meters away from Angelina’s hotcake stand to buy fish. He waited for 30 minutes for
fishermen but no one came, so he went home.

ISSUE:

1. Whether or not the branch 29 of RTC in Bislig City err in finding that conspiracy
attended the killing;
2. Whether or not the branch 29 of RTC in Bislig City err in considering the defense of
the appellants.
RULING:

1. NO. The court said that since the prosecution’s version is culled from the testimony of
eyewitnesses, appellants’ disclaimer of the presence of conspiracy fails. The evidence shows
that they cooperated in a common design to kill Chu. Regalado initiated the killing when he
stabbed Chu on the chest, and the two other appellants joined Regalado in chasing Chu, with
Regalado hitting Chu with firewood along the way. Then, when the three of them had cornered
Chu, Aragon boxed and kicked Chu enabling Lopez to stab him several times. These indicates
a conspiracy.

Aragon’s alibi does not persuade. As the trial court held: “From the ocular inspection of
the wharf conducted in Hinatuan, Surigao del Sur on February 26, 2000,16 it was established
that the wharf was located at the dead-end Villaluz Street. Aragon was at the wharf at about the
same date and time of the stabbing incident, allegedly to buy fish. He was seated at the last
step of the wharf. He stayed there for thirty (30) minutes to wait for a pump boat bringing in fish
but there was none. At about the time of the incident, the water level was supposed to be low
tide so that no pump boat, if there was any, can dock on the wharf. Applying common sense,
nobody in his right mind would wait for about thirty (30) minutes just to buy fish where no pump
boat is in sight.

2. NO. Lopez’s “defense of relative.” As the Court of Appeals held: “Under [Paragraph 2
of Article 11 of the Revised Penal Code], the elements of the justifying circumstance of defense
of relatives are as follows: 1. Unlawful aggression; 2. Reasonable necessity of the means
employed to prevent or repel it; 3. In case provocation was given by the person attacked, that
the one making the defense had no part therein.

And in the case at bar justifying circumstances were absent. Even assuming that there was
unlawful aggression on Chu’s part when he chased Regalado, Lopez was not justified in
stabbing Chu since as admitted by him, he did not see accused-appellant Regalado anymore
when he was able to catch up with Chu. The unlawful aggression of Chu, had it indeed been
present, had already ceased when upon reaching Chu, as Regalado, whom Lopez allegedly
wanted to protect, was no longer there. When an unlawful aggression that has begun no longer
exists, the one who resorts to self-defense has no right to kill or even to wound the former
aggressor. Also, the fact that Chu allegedly boxed and taunted him prompting him to stab the
victim several times negates the reasonableness of the means employed to repel Chu’s
aggression assuming that indeed, Chu started the aggression.

Defense of strangers
- Cabuslay vs. People, 471 SCRA 241 (2005)

CABUSLAY V. PEOPLE G.R. NO. 129875 September 30,2005


FACTS:

Paquito Umas-as earned a living as a collector of payments for assorted articles that he sold on
credit. One fateful morning, he was halted by a police upon reaching a checkpoint. The police
asked him to show his ID. When he took out his ID from his left pocket and when it reached the
front man, one of the policemen, who was identified as the petitioner, opened fire at the collector
whose right hand was then raised. Petitioner, who was four meters away from
the
collector, consumed the entire magazine of his M-16 armalite in firing at him. The
collector fell to the ground and was still moving when the police placed him on board a
vehicle and brought him to Kolambugan for medical attention. Petitioner interposed self-
defense and acting in the lawful performance of his duty as he claimed that the victim
fired first at Regencia, the police who was asking for the ID at the checkpoint.
ISSUE: Whether self-defense and lawful performance of duty under the circumstances
exempts one from criminal liability.

HELD:

No. The 8 gunshot wounds suffered by Paquito negate any claim of self-defense. Had
petitioner merely defended himself from the victims unlawful aggression, one shot to
immobilize him would have been enough. The nature and number of wounds inflicted by
the accused are important indicia which disprove a plea for self-defense or defense of
stranger because they demonstrate a determined effort to kill the victim and not just
defend oneself.
That the killing of Paquito resulted from the lawful performance of duty as a police
officer is of no defense where the victim was not committing any offense at the time.
Cabuslay has not sufficiently proven that the victim had indeed fired at Regencia where
the alleged gun used by the victim was not even presented in evidence. Killing the
victim under the circumstances of this case cannot be considered a valid performance
of a lawful duty by a man who had sworn to maintain peace and order and to protect the
lives of the people

Avoidance of greater evil

- People v. Ricohermoso, 56 SCRA 431

People vs. Ricohermoso

FACTS:

Severo Padernal and Juan Padernal appealed from the decision of the Circuit Criminal Court
at Lucena City, convicting them of murder, sentencing each of them to reclusion perpetua and
ordering them to pay solidarily the sum of twelve thousand pesos to the heirs of Geminiano de
Leon and to pay the costs (Criminal Case No. CCCIX-37-Quezon or 1922-CFI-Gumaca).
In the same decision they were convicted of lesiones leves. Each one was sentenced to suffer
the penalty of fifteen (15 days of arresto menor and to pay the costs. Rosendo Perpeñan, Rito
Monterey and Macario Monterey were acquitted.
At about nine o'clock in the morning of January 30, 1965 Geminiano de Leon, together with his
thirty-three-year old common-law wife Fabiana Rosales, his twenty four-year old son Marianito
de Leon and one Rizal Rosales, encountered Pio Ricohermoso in Barrio Tagbacan Silañgan,
Catanauan, Quezon.

Geminiano owned a parcel of land in that barrio which Ricohermoso cultivated as kaingin.
Geminiano asked Ricohermoso about his share of the palay harvest. He added that he should
at least be allowed to taste the palay harvested from his land. Ricohermoso invited him to his
house to get the palay.

When Geminiano returned to Barrio Tagbacan Silañgan, he stopped at Ricohermoso's place. It


was about two o'clock in the afternoon. Geminiano sat on a sack beside Fabiana Rosales in
front of the house while Marianito (his son) stood about three meters behind his father. A .22
caliber rifle was slung on Marianito's right shoulder.

Geminiano asked Ricohermoso about the palay. The latter, no longer conciliatory and evidently
hostile, answered in a defiant tone: "Whatever happens, I will not give you palay." Geminiano
remonstrated: "Why did you tell us to pass by your house, if you were not willing to give the
palay?"

At that juncture, as if by pre-arrangement, Ricohermoso unsheathed his bolo and approached


Geminiano from the LEFT, while Severo Padernal (Ricohermoso's father-inlaw) got an axe and
approached Geminiano from the RIGHT. The latter looked up to the sexagenarian Severo
Padernal, with both hands raised and pleaded: "Mamay (Grandpa), why will you do this to us.
We will not fight you." While Geminiano was still looking up to Severo Padernal on his right,
Ricohermoso walked to Geminiano's left, and, when about one meter from him, stabbed him on
the neck with his bolo. Geminiano fell face downward on the ground. While in that helpless
position, he was hacked on the back with an axe by Severo Padernal.

At that same place and time, while Severo Padernal and Ricohermoso were assaulting
Geminiano de Leon, another episode was taking place. Juan Padernal (Ricohermoso's brother-
in-law and the son of Severo) suddenly embraced Marianito de Leon from behind, with his right
arm locked around Marianito's neck and his left hand pressing Marianito's left forearm. They
grappled and rolled downhill towards a camote patch. Marianito passed out. When he regained
consciousness, his rifle was gone. He walked uphill, saw his mortally wounded father
Geminiano in his death throes, and embraced him. He carried Geminiano for a short distance.
The fifty-one year old Geminiano died at two o'clock on that same day.

The appellants filed their brief on February 6, 1970. Later, Severo Padernal withdrew his
appeal. The withdrawal was granted in the resolution dated November 3, 1970

Appellant Juan Padernal invokes the justifying circumstance of avoidance of a greater evil
or injury (par. 4, Art. 11, Revised Penal Code) in explaining his act of preventing Marianito de
Leon from shooting Ricohermoso and Severo Padernal.

ISSUES:
1. Whether or not there is merit in the Appellant’s invoking of justifying circumstances in his
conviction of murder.
2. Whether or not Juan Padernal conspired with Ricohermoso and Severo Padernal to kill
Geminiano de Leon.

HELD:

1. NO. His reliance on that justifying circumstance is erroneous. The act of Juan Padernal in
preventing Marianito de Leon from shooting Ricohermoso and Severo Padernal, who were the
aggressors, was designed to insure the killing of Geminiano de Leon without any risk to his
assailants.

2. YES. The trial court rationalized its conclusion that there was conspiracy by stating that
their conduct revealed unity of purpose and a concerted effort to encompass Geminiano's
death.
RATIONALE:
The circumstances surrounding the killing of Geminiano de Leon alevosia or treachery. Juan
Padernal's role of weakening the defense, by disabling Marianito de Leon, was part and parcel
of the means of execution deliberately resorted to by the assailants to insure the assassination
of Geminiano de Leon without any risk to themselves (Par. 16, Article 14, Revised Penal Code).
- Ty v. People (27 Sept 2004)

VICKY C. TY v. PEOPLE, GR No. 149275, 2004-09-27

Facts:

This case stemmed from the filing of seven (7) Informations for violation of B.P. 22 against Ty
before the RTC of Manila.
prosecution shows that Ty's mother... was confined at the Manila Doctors' Hospital (hospital)
from 30 October 1990 until 4 June 1992.
the Statement of Account... shows the total liability of the mother in the amount of P657,182.40.
Ty's sister, Judy Chua, was also confined at the hospital... incurring hospital bills in the amount
of P418,410.55.
The total hospital bills of the two patients amounted to P1,075,592.95. On 5 June 1992, Ty
executed a promissory note wherein she assumed payment of the... obligation in installments.
To assure payment of the obligation, she drew several postdated checks against Metrobank
payable to the hospital. The seven (7) checks, each covering the amount of P30,000.00, were
all deposited on their due... dates. But they were all dishonored by the drawee bank and
returned unpaid to the hospital due to insufficiency of funds, with the "Account Closed" advice.
demand... letters were not heeded, complainant filed the seven (7) Informations subject of the
instant case.
Ty claimed that she issued the checks because of "an uncontrollable fear of a greater injury."
She averred that she was forced to issue the checks to obtain release for her mother whom the
hospital inhumanely and harshly treated and would not discharge... unless the hospital bills are
paid. She alleged that her mother was deprived of room facilities, such as the air-condition
unit, refrigerator and television set, and subject to inconveniences such as the cutting off of the
telephone line, late delivery of... her mother's food and refusal to change the latter's gown and
bedsheets.
She also bewailed the hospital's suspending medical treatment of her mother.
that... the latter contemplated suicide if she would not be discharged from the hospital.
trial court rendered a Decision finding Ty guilty of seven (7) counts of violation of B.P. 22...
appellate court affirmed the judgment of the trial court with modification. It set aside the penalty
of imprisonment and instead sentenced Ty "to pay a fine of
P60,000.00... equivalent to... double the amount of the check, in each case."
Court of Appeals rejected Ty's defenses of involuntariness in the issuance of the checks and the
hospital's knowledge of her checking account's lack of funds. It held that B.P. 22 makes
the mere act of issuing a... worthless check punishable as a special offense, it being a malum
prohibitum. What the law punishes is the issuance of a bouncing check and not the purpose for
which it was issued nor the terms and conditions relating to its issuance.

Issues:

whether the defense of uncontrollable fear is tenable to warrant her exemption from criminal
liability

Ruling:

We find the petition to be without merit and accordingly sustain Ty's conviction.
For this exempting circumstance to be invoked successfully, the following requisites must
concur: (1)... existence of an uncontrollable fear; (2) the fear must be real and imminent; and (3)
the fear of an injury is greater than or at least equal to that committed.
It must appear that the threat that caused the uncontrollable fear is of such gravity and
imminence that the ordinary man would have succumbed to it.
It should be based on a real, imminent or reasonable fear for one's life or limb.
A mere threat of a future injury is not enough. It should not be speculative, fanciful, or remote.
A person invoking uncontrollable fear must show therefore that the compulsion was such that it
reduced him to a mere instrument... acting not only without will but against his will as well.
It must be of such character as to leave no opportunity to the accused for escape.
In this case, far from it, the fear, if any, harbored by Ty was not real and imminent. Ty claims
that she was compelled to issue the checks a condition the hospital allegedly demanded of her
before her mother could be discharged for fear that her mother's health might... deteriorate
further due to the inhumane treatment of the hospital or worse, her mother might commit
suicide. This is speculative fear; it is not the uncontrollable fear contemplated by law.
there was no showing that the mother's illness was so life-threatening such that her continued
stay in the hospital suffering all its alleged unethical treatment would induce a well-grounded
apprehension of her death.
the fear she invokes was not impending or insuperable as to deprive her of all volition and to
make her a mere instrument without... will, moved exclusively by the hospital's threats or
demands.
In the instant case, the evil sought to be avoided is merely expected or anticipated.
If the evil sought to be avoided is merely expected or anticipated or may happen in the future,
this defense is not applicable.
Ty could have taken advantage of... an available option to avoid committing a crime. By her
own admission, she had the choice to give jewelry or other forms of security instead of
postdated checks to secure her obligation.
Moreover, for the defense of state of necessity to be availing, the greater injury feared should
not have been brought about by the negligence or imprudence, more so, the willful inaction of
the actor.
In this case, the issuance of the bounced checks... was brought about by Ty's own failure to pay
her mother's hospital bills.
Fulfillment of duty or lawful exercise of right/office

- People v. Delima (1922)

THE PEOPLE OF THE PHILIPPINE ISLANDS, PLAINTIFF AND APPELLEE, VS. FELIPE
DELIMA, DEFENDANT AND APPELLANT.
G.R. No. 18660, December 22, 1922
Ponente: ROMUALDEZ, J

FACTS:

Felipe Delima, a policeman was tried and convicted for the crime of homicide after killing
a certain Lorenzo Napilon who had escaped from the jail and who at the time when he was
caught in the house of Jorge Alegria he ran away and was armed with a pointed piece of
bamboo. Delima appealed for the reversal of the decision for the killing was done in the
performance of a duty.

ISSUE:
Whether or not the act of the herein defendant-appellant incur any criminal liability.

RULING:
Felipe Delima committed no crime, and he is hereby acquitted for the killing was done in
the performance of a duty. The deceased was under the obligation to surrender, and had no
right, after evading service of his sentence, to commit assault and disobedience with a weapon
in the hand, which compelled the policeman to resort to such an extreme means, which,
although it proved to be fatal, was justified by the circumstances.

Mamagun v. People (2007)

RUFINO S. MAMAGUN, Petitioner,


vs. PEOPLE OF THE PHILIPPINES, Respondent.
G.R. No. 149152, February 2, 2007
Ponente: GARCIA, J.

Facts:
Petitioner was one of the policemen who responded to a robbery hold-up that was in
progress in Bulacan. He was accompanied by PO2 Cruz and PO2 Diaz to the rooftop of the
house where the suspect was allegedly taking refuge. They searched the rooftop and saw a
man whom they thought was the suspect. Petitioner fired his handgun and hit the man. The man
turned out to be Gener Contreras who was not the robbery suspect. Contreras died from the
gunshot wound. The Sandiganbayan found petitioner guilty of the crime homicide. Unable to
accept the conviction, the petitioner went to the Supreme Court alleging that the Sandiganbayan
committed reversible error in failing to apply paragraph 5, Article 11 of the Revised Penal Code,
which would absolve him from criminal liability. He alleges that the shooting was done in
performance of a duty or in the lawful exercise of a right or office.

Issue:
Whether or not Article 11, paragraph 5 of the Revised Penal Code applies to petitioner.
Held:

The Supreme Court held that proof that the shooting and death of Contreras was a
necessary consequence of the due performance of petitioner’s duty should be provided.
Petitioner’s allegation that he shot Contreras because the latter tried to strike him with a steel
pipe was held to be self-serving and merely an afterthought. Petitioner did not report the
incident to the police investigator when he reported back to the police station. It was only when
a lead pipe was discovered that he conveniently remembered that Contreras tried to hit him.
Such vital information could not have escaped the petitioner’s mind. Thus, Article 11, Paragraph
5 of the Revised Penal Code does not apply to petitioner.

Obedience to an order issued for some lawful purpose

- People v. Beronilla 96 SCRA 566

People v. Beronilla, et al.


Plaintiff-appellee: People of the Phil.
Defedants-appellants: Manuel Beronilla, Filipino Velasco, Policarpo Paculdo, Jacinto
Adriatico

Facts:

 Setting: After Japanese occupation


 Borjal served as mayor during the Japanese occupation.

 Beronilla, after being appointed as Military Mayor of La Paz, Abra, received a copy of a
memorandum issued by Lt. Col. Arnold authorizing all military mayors to appoint a hury of 12
bolomen to try persons accused of treason, espionage or the aiding of the enemy.
 He also received a list of all puppet officials of the province of Abra (Borjal included) with a
memorandum instructing all Military Mayors to investigate said persons and gather against them
complaints from people of the municipality.
 Beronilla, upon the return of Borjal who left La Paz because of an attempt on his life, placed
Borjal under custody.
 Pursuant to his instructions, complaints were gathered, a 12-man jury was appointed,
prosecutors and a clerk of the jury were assigned.
 Trial lasted for 19 days and the jury found Borjal guilty on all counts (espionage, aiding the
enemy, abuse of authority). Death penalty was imposed.
 Beronilla forwarded the records of the case to the Headquarters of the 15th Infantry for review.
 The records were returned by Lt. Col. Arnold adding that the matter was best handled by the La
Paz Government and whatever disposition taken was approved.
 Upon receipt of the letter, Beronilla then ordered the execution of Borjal. The execution was
reported to Col. Arnold and Beronilla received compliments based on the reply of his superior.
 Two years after, those who were involved were indicted in the CFI of Abra for murder for
allegedly conspiring and confederating in the execution of Borjal.
 The defendants were convicted, thus this appeal.

Issue: Are the defendants guilty of murdering Borjal?

Held: No.

Ratio Decidendi:

 The records are ample to sustain the claim of the defense that the arrest, prosecution and trial of
Borjal were done pursuant to express orders of the 15th Infantry HQ.
 The state however contends that there was a radiogram from Col. Volckmann to Lt. Col. Arnold,
specifically noting the illegality of Borjal’s conviction and sentence, which the prosecution claims that
Beronilla was informed about this but still pursued with the execution.
 A witness, a relative of Borjal, asserts that he was present when Beronilla received the message
and was able to read it over Beronilla’s shoulder. But basing on his affidavit, it can be imported that
witness was not with Beronilla the message alleged to have arrived.
 Moreover, Beronilla would not have informed Lt. Col. Arnorld about the execution if he did it after
the receipt of the message since this would be in violation of superior orders. And more importantly,
Arnold complimented him in his reply.
 The Court concludes that Lt. Col. Arnold failed to transmit the Volckmann message to Beronilla.
The charge of criminal conspiracy to do away with Borjal must be rejected because the accused
had no need to conspire against a man who was, to their knowledge, duly sentenced to death.
 The accused acted as military subordinates only following superior orders, thus there is no
criminal intent.

 Judgement appealed from reversed and the appellants were acquitted.


Other Digest

Obedience to Lawful Order of a Superior

Facts:

Manuel Beronilla, Policarpio Paculdo, Filipino Velasco and Jacinto Adriatico file an appeal from
the judgement of the Abra CFI, which convicted them of murder for the execution of Arsenio
Borjal, the elected mayor of La, Paz, Abra (at the outbreak of war), which was found to be aiding
the enemy.
Borjal moved to Bangued because of death threats was succeeded by Military Mayor Manuel
Beronilla, who was appointed by Lt. Col. Arbold, regimental commander of the 15th Infantry of
the Phil. Army, operating as guerilla unit in Abra. Simultaneously upon his appointment,
Beronilla received a memorandum which authorized him to appoint a jury of 12 bolo men to try
persons accused of treason, espionage and aiding or abetting the enemy.
Upon the return of Borjal and his family to Abra, to escape bombing in Bangued, he was placed
under custody and tried and sentenced to death by the jury based on various complaints made
by the residents. Beronilla reported this to Col. Arnold who replied, saying “…I can only
compliment you for your impartial but independent way of handling the whole case.”

Two years thereafter, Beronilla, along with the executioner, digger and jury, were indicted for the
murder of Borjal. Soon after, President Manuel Roxas issued Executive Proclamation 8, which
granted amnesty to persons who committed acts in furtherance of the resistance to the enemy
against persons aiding in the war efforts of the enemy.

The rest of defendants applied and were granted amnesty, but Beronilla and others were
convicted on the grounds that the crime was made on purely personal motives and that the
crime was committed after the expiration of time limit for amnesty proclamation.

Issue: W/N the defendant-appellants’ actions are covered by justifying circumstances for


obedience to lawful order of superior
 
Held:

Yes. The accused acted upon orders of their superior officers, which as military subordinates,
they could not question and obeyed in good faith without the being aware of its illegality.
The evidence is sufficient to sustain the claim of the defense that arrest, prosecution and trial of
Borjal was done in pursuant to express orders of superiors. Additionally, it could not be
established that Beronilla received the radiogram from Colonel Volckmann, overall area
commander, which called attention to the illegality of Borjal’s conviction and sentence. Had
Beronilla known the violation, he would not have dared to report it to Arnold. The conduct of the
accused also does not show malice on their part because of the conduct of the trial, defense
through counsel given to Borjal, suspension of trial based on doubts of illegality and death
sentence review sent to the superior officers.

Criminal intent then could not be established. The maxim here is actus non facit reum, nisi
mens rea (Crime is not committed if the mind of the person performing the act complained of to
be innocent).
Additionally, the lower court should not have denied their claim to the benefits of the Guerilla
Amnesty Proclamation No. 8 inspite of contradictory dates of liberation of La Paz, Abra. Even if
the dates were contradictory, the court should have found for the Beronila, et al because if there
are “any reasonable doubt as to whether a given case falls within the (amnesty) proclamation
should be resolved in favor of the accused.”

Judgement reversed, appellants acquitted.

Battered woman syndrome as self-defense


- People vs. Genosa, 341 SCRA 493

G.R. No. 135981 January 15, 2004


PEOPLE OF THE PHILIPPINES, appellee, vs. MARIVIC GENOSA, appellant.
DECISION
PONENTE : PANGANIBAN, J.:

FACTS:
·
"Appellant and Ben Genosa were united in marriage on November 19, 1983 in Ormoc City.
Thereafter, they lived with the parents of Ben in their house at Isabel, Leyte. For a time, Ben's
younger brother, Alex, and his wife lived with them too. Sometime in 1995, however, appellant
and Ben rented from Steban Matiga a house at Barangay Bilwang, Isabel, Leyte where they
lived with their two children, namely: John Marben and Earl Pierre.
·
"On November 16, 1995, appellant asked Erlinda Paderog, her close friend and neighbor
living about fifty (50) meters from her house, to look after her pig because she was going to
Cebu for a pregnancy check-up. Appellant likewise asked Erlinda to sell her motorcycle to their
neighbor Ronnie Dayandayan who unfortunately had no money to buy it.
·
"That same day, about 12:15 in the afternoon, Joseph Valida was waiting for a bus going to
Ormoc when he saw appellant going out of their house with her two kids in tow, each one
carrying a bag, locking the gate and taking her children to the waiting area where he was.
Joseph lived about fifty (50) meters behind the Genosas' rented house. Joseph, appellant and
her children rode the same bus to Ormoc. They had no conversation as Joseph noticed that
appellant did not want to talk to him.
·
"Then, Ben purportedly nagged appellant for following him, even challenging her to a fight.
She allegedly ignored him and instead attended to their children who were doing their
homework. Apparently disappointed with her reaction, Ben switched off the light and, with the
use of a chopping knife, cut the television antenna or wire to keep her from watching television.
·
According to appellant, Ben was about to attack her so she ran to the bedroom, but he got
hold of her hands and whirled her around. She fell on the side of the bed and screamed for
help. Ben left. At this point, appellant packed his clothes because she wanted him to leave.
·
Seeing his packed clothes upon his return home, Ben allegedly flew into a rage, dragged
appellant outside of the bedroom towards a drawer holding her by the neck, and told her 'You
might as well be killed so nobody would nag me.'
·
Appellant testified that she was aware that there was a gun inside the drawer but since Ben
did not have the key to it, he got a three-inch long blade cutter from his wallet. She however,
'smashed' the arm of Ben with a pipe, causing him to drop the blade and his wallet.
·
Appellant then 'smashed' Ben at his nape with the pipe as he was about to pick up the
blade and his wallet. She thereafter ran inside the bedroom.
·
"Appellant, however, insisted that she ended the life of her husband by shooting him. She
supposedly 'distorted' the drawer where the gun was and shot Ben. He did not die on the spot,
though, but in the bedroom."
ISSUE:

Whether or not unlawful aggression was present to merit the self defense as appealed by the
accused?

HELD:

From the expert opinions discussed earlier, the Court reckons further that crucial to the BWS
defense is the state of mind of the battered woman at the time of the offense [60] -- she must
have actually feared imminent harm from her batterer and honestly believed in the need to kill
him in order to save her life.

Settled in our jurisprudence, however, is the rule that the one who resorts to self-defense must
face a real threat on one's life; and the peril sought to be avoided must be imminent and actual,
not merely imaginary.[61] Thus, the Revised Penal Code provides the following requisites and
effect of self-defense:[62]

"Art. 11. Justifying circumstances. -- The following do not incur any criminal liability:

"1. Anyone who acts in defense of his person or rights, provided that the following
circumstances concur;

First. Unlawful aggression;


Second. Reasonable necessity of the means employed to prevent or repel it;
Third. Lack of sufficient provocation on the part of the person defending himself."

Unlawful aggression is the most essential element of self-defense. [63] It presupposes


actual, sudden and unexpected attack -- or an imminent danger thereof -- on the life or safety of
a person.[64] In the present case, however, according to the testimony of Marivic herself,
there was a sufficient time interval between the unlawful aggression of Ben and her fatal
attack upon him. She had already been able to withdraw from his violent behavior and escape
to their children's bedroom. During that time, he apparently ceased his attack and went to bed.
The reality or even the imminence of the danger he posed had ended altogether. He was no
longer in a position that presented an actual threat on her life or safety.

We reiterate the principle that aggression, if not continuous, does not warrant self-
defense.[67] In the absence of such aggression, there can be no self-defense -- complete
or incomplete -- on the part of the victim.[68] Thus, Marivic's killing of Ben was not completely
justified under the circumstances.

RULING:

The penalty for parricide imposed by Article 246 of the Revised Penal Code is reclusion
perpetua to death. Since two mitigating circumstances and no aggravating circumstance have
been found to have attended the commission of the offense, the penalty shall be lowered by
one (1) degree, pursuant to Article 64 of paragraph 5[88] of the same Code.[89] The penalty of
reclusion temporal in its medium period is imposable, considering that two mitigating
circumstances are to be taken into account in reducing the penalty by one degree, and no other
modifying circumstances were shown to have attended the commission of the offense. [90] Under
the Indeterminate Sentence Law, the minimum of the penalty shall be within the range of that
which is next lower in degree -- prision mayor -- and the maximum shall be within the range of
the medium period of reclusion temporal.

Considering all the circumstances of the instant case, we deem it just and proper to impose the
penalty of prision mayor in its minimum period, or six (6) years and one (1) day in prison as
minimum; to reclusion temporal in its medium period, or 14 years 8 months and 1 day as
maximum. Noting that appellant has already served the minimum period, she may now apply
for and be released from detention on parole.[91]

Art. 12 - R.A. No. 9344, Sec. 6

Insanity as a defense
- People vs. Tibon, 622 SCRA 510 (2010)
- Herrera vs. Sandiganbayan, 579 SCRA 32
- People vs. Domingo, 580 SCRA 436

- Sierra vs. People, 591 SCRA 666

ROBERT SIERRA Y CANEDA v. PEOPLE, GR No. 182941, 2009-07-03


609 Phil. 446
SECOND DIVISION
[ G.R. No. 182941, July 03, 2009 ]
ROBERT SIERRA Y CANEDA, PETITIONER, VS. PEOPLE OF THE PHILIPPINES,
RESPONDENT.
DECISION
BRION, J.

Facts

In August 2000, thirteen-year-old AAA[5] was playing with her friend BBB in the second floor of
her family's house in Palatiw, Pasig. The petitioner arrived holding a knife and told AAA and
BBB that he wanted to play with them. The petitioner then undressed
BBB and had sexual intercourse with her. Afterwards, he turned to AAA, undressed her, and
also had sexual intercourse with her by inserting his male organ into hers. The petitioner warned
AAA not to tell anybody of what they did.
AAA subsequently disclosed the incident to Elena Gallano (her teacher) and to Dolores
Mangantula (the parent of a classmate), who both accompanied AAA to the barangay office.
AAA was later subjected to physical examination that revealed a laceration on her hymen...
consistent with her claim of sexual abuse. On the basis of the complaint and the physical
findings, the petitioner was charged with rape
The petitioner pleaded not guilty to the charge and raised the defenses of denial and alibi.
He claimed that he was selling cigarettes at the... time of the alleged rape. He also claimed that
AAA only invented her story because she bore him a grudge for the beatings he gave her.
The parties' mother (CCC) supported the petitioner's story; she also stated that AAA was a
troublemaker. Both CCC and son testified that the petitioner was fifteen (15) years old when the
alleged incident happened.[7]
The defense also presented BBB who denied that the petitioner raped her; she confirmed the
petitioner's claim that AAA bore her brother a grudge.
the RTC convicted the petitioner of qualified rape
The petitioner elevated this RTC decision to the CA by attacking AAA's credibility. He also
invoked paragraph 1, Section 6 of R.A. No. 9344 (Juvenile Justice and Welfare Act of 2006)[9]
to exempt him from criminal liability considering that he was... only 15 years old at the time the
crime was committed.
The CA nevertheless affirmed the petitioner's conviction with modification as to penalty
The CA denied the petitioner's subsequent motion for reconsideration; hence, the present
petition.

Issues:
Whether or not the CA erred in not applying the provisions of R.A. No. 9344 on the petitioner's
exemption from criminal liability;
(2)
Whether or not the CA erred in ruling that it was incumbent for the defense to present the
petitioner's birth certificate to invoke Section 64 of R.A. No. 9344 when the burden of proving his
age lies with the prosecution by express provisions of R.A. No.9344; an
Ruling:
In August 2000, thirteen-year-old AAA[5] was playing with her friend BBB in the second floor of
her family's house in Palatiw, Pasig. The petitioner arrived holding a knife and told AAA and
BBB that he wanted to play with them. The petitioner then undressed
BBB and had sexual intercourse with her. Afterwards, he turned to AAA, undressed her, and
also had sexual intercourse with her by inserting his male organ into hers. The petitioner warned
AAA not to tell anybody of what they did.
AAA subsequently disclosed the incident to Elena Gallano (her teacher) and to Dolores
Mangantula (the parent of a classmate), who both accompanied AAA to the barangay office.
AAA was later subjected to physical examination that revealed a laceration on her hymen...
consistent with her claim of sexual abuse. On the basis of the complaint and the physical
findings, the petitioner was charged with rape under the following Information:
The threshold issue in this case is the determination of who bears the burden of proof for
purposes of determining exemption from criminal liability based on the age of the petitioner at
the time the crime was committed.

THE COURT'S

We grant the petition.


The records show that the prosecution established all the elements of the crime charged
through the credible testimony of AAA and... the other corroborating evidence; sexual
intercourse did indeed take place as the information charged.
While the defense, on appeal, raises a new ground - i.e., exemption from criminal liability under
R.A. No. 9344 - that implies an admission of guilt, this consideration in no way swayed the con...
c
, exemption from criminal liability under R.A. No. 9344 - that implies an admission of guilt, this
consideration in no way swayed the conclusion... conclusion
While the defense, on appeal, raises a new ground - i.e., exemption from criminal liability under
R.A. No. 9344 - that implies an admission of guilt, this consideration in no way swayed the
conclusion we made above, as the defense is entitled to present all alternative... defenses
available to it, even inconsistent ones. We note, too, that the defense's claim of exemption from
liability was made for the first time in its appeal to the CA. While this may initially imply an
essential change of theory that is usually disallowed on appeal for reasons... of fairness, [22] no
essential change is really involved as the claim for exemption from liability is not incompatible
with the evidence submitted below and with the lower courts' conclusion that the petitioner is
guilty of the crime charged. An exempting... circumstance, by its nature, admits that criminal and
civil liabilities exist, but the accused is freed from criminal liability; in other words, the accused
committed a crime, but he cannot be held criminally liable therefor because of an exemption
granted by law. In admitting... this type of defense on appeal, we are not unmindful, too, that the
appeal of a criminal case (even one made under Rule 45) opens the whole case for review,
even on questions that the parties did not raise.[23] By mandate of the Constitution, no less,
we... are bound to look into every circumstance and resolve every doubt in favor of the accused.
[24] It is with these considerations in mind and in obedience to the direct and more specific
commands of R.A. No. 9344 on how the cases of children in conflict with... the law should be
handled that we rule in this Rule 45 petition.
In the present case, the petitioner claims total exemption from criminal liability because he was
not more than 15 years old at the time the rape took place. The CA disbelieved this claim for the
petitioner's failure to present his birth certificate as required by Section 64 of
R.A. No. 9344.[29] The CA also found him disqualified to avail of a suspension of sentence
because the imposable penalty for the crime of rape is reclusion perpetua to death.
The CA seriously erred when it rejected testimonial evidence showing that the petitioner was
only 15 years old at the time he committed the crime. Section 7 of R.A. No. 9344 expressly
states how the age of a child in conflict with the law may be determined
All these conditions are present in this case. First, the petitioner and CCC both testified
regarding his minority and age when the rape was committed.[39
The retroactive application of R.A. No. 9344 is also justified under Article 22 of the RPC, as
amended, which provides that penal laws are to be given retroactive effect insofar as they favor
the accused who is not found to be a habitual criminal. Nothing in the records of this... case
indicates that the petitioner is a habitual criminal.
The crime for which the petitioner should have been found criminally... liable should therefore
only be simple rape
We appreciate dwelling as an aggravating circumstance based on AAA's testimony that the
rape was committed in their house.[50] While dwelling... as an aggravating circumstance
WHEREFORE, premises considered, the instant petition is GRANTED.

Minority (Superseded by RA 9344)


- People v. Capistrano, GR L-4549

- Llave v. People (2006)

NIEL F. LLAVE, Petitioner,


vs. PEOPLE OF THE PHILIPPINES, Respondent.
G.R. No. 166040, April 26, 2006

FACTS

That on or about the 24th day of September 2002, NIEL F. LLAVE, a minor 12 years of
age but acting with discernment, by means of force threat and intimidation, did then and there
willfully, unlawfully, feloniously have carnal knowledge of the complainant, DEBBIELYN
SANTOS, a minor, seven (7) years of age, against her will and consent.

Debbielyn testified that on September 24, 2002, she arrived home at past 6:00 p.m. She
changed her clothes and proceeded to her mother’s store. Marilou asked her daughter to bring
home the container with the unsold quail eggs.11 Debbielyn did as told and went on her way. As
she neared the vacant house, she saw petitioner, who suddenly pulled her behind a pile of
hollow blocks which was in front of the vacant house. There was a little light from the lamp
post.12 She resisted to no avail.13 Petitioner ordered her to lie down on the cement. Petrified,
she complied. He removed her shorts and underwear then removed his own. He got on top of
her.14 She felt his penis being inserted into her vagina. He kissed her.15 She felt pain and
cried.16 She was sure there were passersby on the street near the vacant house at the time.

RTC RULING: At the conclusion of the trial, the court rendered judgment convicting Neil of the
crime charged. Sentenced him to prision mayor minimum, Six (6) years and One (1) day to
Eight (8) years, and pay civil indemnity of Fifty Thousand Pesos (Php50,000.00).

CA RULING: The CA rendered judgment affirming the decision with modification. Sentenced to
an indeterminate penalty of two (2) years and four (4) months of prision correccional medium as
the minimum to eight (8) years and one (1) day of prision mayor medium as the maximum.
Additionally, the accused-appellant is ordered to pay the complaining witness the amount of
₱50,000 by way of moral damages and ₱20,000 by way of exemplary damages.

ISSUE

1. WON he was deprived of his right to a preliminary investigation.

2. WON he acted with discernment

3. WON the penalty imposed by the appellate court is correct; and

4. WON he is liable to pay moral damages to the private complainant.

RULING

1. Yes. Section 7, Rule 112 of the Revised Rules of Criminal Procedure provides:

SEC. 7. When accused lawfully arrested without warrant. – When a person is lawfully
arrested without a warrant involving an offense which requires a preliminary investigation,
the complaint or information may be filed by a prosecutor without need of such investigation
provided an inquest has been conducted in accordance with existing rules. In the absence
or unavailability of an inquest prosecutor, the complaint may be filed by the offended party
or a peace officer directly with the proper court on the basis of the affidavit of the offended
party or arresting officer or person. The Office of the Solicitor General (OSG) avers that
petitioner was subjected to an inquest investigation under Section 7, Rule 112 of the
Revised Rules of Criminal Procedure, as gleaned from the Certification of the City
Prosecutor incorporated in the Information.

Before the complaint or information is filed, the person arrested may ask for a
preliminary investigation in accordance with this Rule, but he must sign a waiver of the
provisions of Article 125 of the Revised Penal Code, as amended, in the presence of his
counsel. Notwithstanding the waiver, he may apply for bail and the investigation must be
terminated within fifteen (15) days from its inception.

After the filing of the complaint or information in court without a preliminary investigation,
the accused may, within five (5) days from the time he learns of its filing, ask for a
preliminary investigation with the same right to adduce evidence in his defense as
provided for in this Rule.

As gleaned from the Certification of the City Prosecutor which was incorporated in the
Information, petitioner did not execute any waiver of the provisions of Article 125 of the
Revised Penal Code before the Information was filed. He was arraigned with the
assistance of counsel on October 10, 2002, and thereafter filed a petition for bail.63
Petitioner’s failure to file a motion for a preliminary investigation within five days from
finding out that an Information had been filed against him effectively operates as a
waiver of his right to such preliminary investigation.

2. Yes. Discernment, as used in Article 12(3) of the Revised Penal Code is defined as
follows: "the discernment that constitutes an exception to the exemption from
criminal liability of a minor under fifteen (15) years of age but over nine (9), who
commits an act prohibited by law, is his mental capacity to understand the difference
between right and wrong" (People v. Doquena, 68 Phil. 580 [1939]).

whether a minor accused acted with discernment, his mental capacity to understand the
difference between right and wrong, which may be known and should be determined by
considering all the circumstances disclosed by the record of the case, his appearance,
his attitude and his behavior and conduct, not only before and during the commission of
the act, but also after and even during the trial should be taken into consideration
(People v. Doquena, supra). In the instant case, petitioner’s actuations during and after
the rape incident, as well as his behavior during the trial showed that he acted with
discernment. The fact appears undisputed that immediately after being discovered by
the prosecution’s witness, Teofisto Bucud, petitioner immediately stood up and ran
away. Shortly thereafter, when his parents became aware of the charges against him
and that private complainant’s father was looking for him, petitioner went into hiding.

During the trial, petitioner submitted documentary evidence to show that he was a
consistent honor student and has, in fact, garnered several academic awards. This
allegation further bolstered that he acted with discernment, with full knowledge and
intelligence. The fact that petitioner was a recipient of several academic awards and was
an honor student further reinforces the finding that he was possessed of intelligence well
beyond his years and thus was able to distinguish, better than other minors of his age
could, which conduct is right and which is morally reprehensible. Hence, although
appellant was still a minor of twelve years of age, he possessed intelligence far beyond
his age. It cannot then be denied that he had the mental capacity to understand the
difference between right and wrong.

3. Yes. The trial court correctly ruled that the petitioner acted with discernment when
he had carnal knowledge of the offended party; hence, the CA cannot be faulted for
affirming the trial court’s ruling.

4. No. Under Article 2231, of the New Civil Code, exemplary damages may be
awarded if the crime was committed with one or more aggravating circumstances. In
this case, no aggravating circumstance was alleged in the Information and proved by
the People; hence, the award must be deleted.

LEGAL PREMISES

Rape is consummated if there is some degree of penetration within the vaginal surface.
Corroborative evidence is not necessary to prove rape. As long as the testimony of the victim is
credible, such testimony will suffice for conviction of consummated rape.

Discernment, as used in Article 12(3) of the Revised Penal Code is defined as follows:
"the discernment that constitutes an exception to the exemption from criminal liability of a minor
under fifteen (15) years of age but over nine (9), who commits an act prohibited by law, is his
mental capacity to understand the difference between right and wrong" (People v. Doquena, 68
Phil. 580 [1939]).

For a minor above nine but below fifteen years of age, he must discern the rightness or
wrongness of the effects of his act (Guevarra v. Almodova, G.R. No. 75256, January 26, 1989).

Professor Ambrocio Padilla, in his annotation of Criminal Law (p. 375, 1998 Ed.), writes
that "discernment is more than the mere understanding between right and wrong. Rather, it
means the mental capacity of a minor between 9 and 15 years of age to fully appreciate the
consequences of his unlawful act" (People v. Navarro, [CA] [51 O.G. 4062]).

Judging whether a minor accused acted with discernment, his mental capacity to
understand the difference between right and wrong, which may be known and should be
determined by considering all the circumstances disclosed by the record of the case, his
appearance, his attitude and his behavior and conduct, not only before and during the
commission of the act, but also after and even during the trial should be taken into consideration
(People v. Doquena, supra).

- Hubilla v. People

Accident without fault or intention of causing it


People v. Bandian, GR 45186

PEOPLE vs. BANDIAN G.R. No. 45186 September 30, 1936 Infanticide, Article 12
Exempting Circumstances
NOVEMBER 20, 2017

FACTS:

At About 7 in the morning of January 31, 1936, Valentine Aguilar, the apellant’s neighbor, saw
the appelant go to the thicket about four or five brazas from her house, apparently to respond to
a call of nature because it was there that the people of the place used to go for that purpose. A
few minutes later, he then again saw her emerge from the thicket with her clothes stained with
blood both in front and back, stagerring and visibly showing signs of not being able to support
herself. He ran to her aid and having noted that she was very weak and dizzy, he supported and
helped her go up to her house and placed her in her bed.

Upon being asked before Aguilar brought her to her house, what had happened to her, the
appellant answered that she was very dizzy. Not wishing to be alone with the appellant in such
circumstances, Aguilar called Adriano Comcom who lived nearby to be there and help him and
the appellant. He asked Comcom to take bamboo leaves to stop the hemhorrage which had
come upon the appellant. Comcom had scarcely gone about five brazas when he saw the body
of newborn baby near a path adjoining the thicket where the appellant had gone a few moments
before. Comcom informed Aguilar of it and the latter told him to bring the body to the appellant’s
house. Upon being askes whether the babywhich was shown to her was hers or not, the
appellant answered in the affirmative.

In the afternoon of the said day, Dr. Emilio Nepomuceno, president of the sanitary division went
to the appellant’s house and found her still lying in bed still bleeding. In his opinion, the
physician declared that the appellant gave birth in her house, and afterwhich, he threw the child
into the thicket to kill it for the purpose of concealing her dishonor from the man, Luis Kirol, with
whom she was married to, because the child was not his but with another man with whom she
had previously has amorous relations. Nepomuceno testified that the appellant admitted killing
her child.

ISSUE: What was the crime committed by appellant?

RULING: The evidence certainly does not show that the appellant , in causing her child’s
death in one way or another, or in abandoning it in the thicket, did so willfully, consciously, or
imprudently. She had no cause to kill or abandon it, to expose it to death , because her affair
with a former lover, Luis Kirol took place three years before the incident. The husband of the
appellant testified at the trial affirming the belief that the child was his.
Infanticide and abandonment of a minor, to be punishable must be committed willfully and
consciously, or at least it must be the result of a voluntary, conscious and free act or omission.
Even in cases where said crimes are committed through mere imprudence, the person who
commits them, under said circumstances, must be in the full enjoyment of his mental facilities,
or must be conscious of his acts, in order that he may be held liable.

The law exempts from criminal liability any person who acts under the circumstances in which
the appellant acted in this case, by giving birth to a child in the thicket and later abandoning it,
not because of imprudence or any other cause than that she was overcome by severe dizziness
and extreme debility, with no fault or intention on her part. She has in her favor the fourth and
the seventh exempting circumstances.

In conclusion, taking into account the foregoing facts and considerations, and granting that the
appellant was aware of her involuntary childbirth in the thicket and that she later failed to take
her child therefrom, having been so prevented by reason of causes entirely independent of her
will, it should be held that… under such circumstances, appellant has the fourth and seventh
exempting circumstances of article 12 of the Revised Penal Code in her favor.

- People v. Abrazaldo. GR 124392

- People v. Latosa, GR 186128

PEOPLE OF THE PHILIPPINES, Plaintiff-Appellee,


vs. SUSAN LATOSA y CHICO, Accused-Appellant
G.R. No. 186128, June 23, 2010
Ponente: VILLARAMA, JR., J.:

Facts:
On February 5, 2002, at around 2:00 in the afternoon, Susan Latosa, herein appellant,
together with his husband Major Felixberto Sr. and two children Sassymae and Michael, were in
their house in Fort Bonifacio, Taguig. While Major Felixberto Sr. was asleep, Sassymae saw
her mother take Felixberto Sr.’s gun and leave. She asked her mother where she was going and
if she could come along, appellant refused. Moments later, appellant returned and told
Sassymae to buy ice cream. After Sassymae left, appellant instructed Michael to join his sister,
but he refused. Appellant thereafter turned up the volume of the television and radio to full.
Shorty after that, appellant gave her son money to buy food.
After buying his food, Michael went back to their house and thereupon saw his friend
Mac-Mac who told him that he saw appellant running away from their house. Moments later, a
certain Sgt. Ramos arrived and asked if something had happened in their house. Michael
replied in the negative then entered their house. At that point, he saw his father lying on the bed
with a hole in the left portion of his head and a gun at his left hand.
Michael immediately went outside and informed Sgt. Ramos about what happened. Sgt.
Ramos told him that appellant had reported the shooting incident to the Provost Marshall office.
Then, Sassymae arrived and saw her father with a bullet wound on his head and a gun near his
left hand.
Appellant claimed that the killing was an accident, that when Felixberto, Sr. woke up, he
asked her to get his service pistol from the cabinet adjacent to their bed. As she was handing
the pistol to him it suddenly fired, hitting Felixberto, Sr. who was still lying down.

● The RTC found appellant guilty beyond reasonable doubt for the crime of parricide. The
RTC, in finding appellant guilty, considered the following circumstantial evidence
established by the prosecution: (1) shortly before the shooting, appellant asked her two
(2) children to do errands for her which were not usually asked of them; (2) at the time of
the shooting, only the appellant and Felixberto, Sr. were in the house; (3) appellant was
seen running away from the house immediately after the shooting; (4) when Michael
went inside their house, he found his father with a hole in the head and a gun in his left
hand; (5) the medico-legal report showed that the cause of death was intracranial
hemorrhage due to the gunshot wound on the head with the point of entry at the left
temporal region; (6) the Firearms Identification Report concluded that appellant fired two
(2) shots; (7) Felixberto, Sr. was right-handed and the gun was found near his left hand;
(8) Sassymae testified that she heard Sta. Inez tell appellant “bakit mo inamin. Sana
pinahawak mo kay Major iyong baril saka mo pinutok”; (9) appellant’s children testified
that they were informed by Felixberto, Sr. regarding the threat of appellant’s paramour,
Sta. Inez, to the whole family; and (10) Francisco Latosa presented a memorandum
showing that appellant was terminated from her teaching job by reason of immorality.

● The CA upheld the decision of the RTC. The CA held that since appellant admitted
having killed her husband albeit allegedly by accident, she has the burden of proving the
presence of the exempting circumstance of accident to relieve herself of criminal
responsibility. She must rely on the strength of her own evidence and not on the
weakness of the prosecution, for even if this be weak, it cannot be disbelieved after the
appellant has admitted the killing.

Issue:
WON appellant has strongly established the exempting circumstance of accident to
relieve him from criminal liability.

Held:
No. SC held that it was incumbent upon appellant to prove with clear and convincing
evidence, the following essential requisites for the exempting circumstance of accident. To
prove the circumstance she must rely on the strength of her own evidence and not on the
weakness of that of the prosecution, for even if this be weak, it can not be disbelieved after the
accused has admitted the killing.
SC find no merit in appellant’s contention that the prosecution failed to prove by
circumstantial evidence her motive in killing her husband. Intent to kill and not motive is the
essential element of the offense on which her conviction rests.
The following circumstantial evidence considered by the RTC and affirmed by the CA
satisfactorily established appellant’s intent to kill her husband and sustained her conviction for
the crime.
WHEREFORE, the appeal of Susan Latosa y Chico is DISMISSED. The April 23, 2008
Decision of the Court of Appeals in CA-G.R. CR-H.C. No. 02192 is hereby AFFIRMED with
MODIFICATION. The amount of exemplary damages is increased to P30,000.00.

- People v. Fallorina, GR 137347


Irresistible force and uncontrollable fear

- People vs. Anod, 597 SCRA 205

- People vs. Gonzales, G.R. No. 195534, June 13, 2012

- People v. Bandian, 63 Phil. 530

Stand-your-ground rule

- U.S. v. Domen, 37 Phil. 57

C. Chapter Three: Mitigating Circumstances

Art. 13 - R.A. No. 9344, as amended by R.A. No. 10630


- People vs. Martin, 89 Phil. 18
- U.S. vs. Rivera, 41 Phil. 472 - People vs. de Jesus, 118
SCRA 616
- People vs. Toring, 191 SCRA 38

Incomplete justifying or exempting circumstances


- People vs. Oanis, 74 Phil. 257

No intention to commit so grave a wrong


- People v. Jaurigue, GR 384
- People v. Ural, 56 SCRA 138
- People v. Regato, 127 SCRA 287
Sufficient provocation or threat
- People v. Pagal, 76 SCRA 570
Immediate vindication of a grave offense
- People v. Diokno, 63 Phil 601
- People v. Benito (1976)
Passion and obfuscation
- People v. Guhiting, 88 Phil 672
- People v. Muit, 111 SCRA 696 –
People v. Oloverio, GR 211159
Voluntary surrender and confession of guilt
- People v. Crisostomo, 160 SCRA 47 (1988)
- People v. Dulos, 237 SCRA 141 (1994)
- People v. Viernes, GR 136733-35
- People v. Sales, GR 177218
Illness
- People v. Javier (1999)

D. Chapter Four: Aggravating Circumstances


Art. 14
Advantage be taken by the offender of his public position
- People vs. Mandolado, G.R. No. L-51304
- People vs. Gapasin, G.R. No. 7348
- People vs Villa, GR No. 129899
- PP vs. Villamor, GR No. 140407-08
- People vs. Fallorina, G.R. No. 137347
Crime committed in contempt of or with insult to the public authorities
- People vs. Rodil, G.R. No. L-3515

With insult or lack or regard due to offended party by reason of rank, age, or
sex, dwelling
- People vs Santos, GR No. L-4189
- People vs Daniel, G.R. No. L-40330
- People vs. Rodil, G.R. No. L-35156

People v. Tano, G.R. No. 133872

PEOPLE v. ALEXANDER TAÑO Y CABALLERO, GR No. 133872, 2000-05-05

Facts:

The appellant cannot be convicted of the special complex crime of robbery with rape because
the asportation was conceived and carried out as an afterthought and only after the rape has
been consummated. Dwelling cannot be appreciated as an aggravating... circumstance in this
case because the rape was committed in the ground floor of a two-story structure, the lower
floor being used as a video rental store and not as a private place of abode or residence.
On November 6, 1997, at around 7:30 p.m., Amy de Guzman (Amy) was tending a Video Rental
Shop owned by her employer and cousin, Ana Marinay (Ana) located at 153 Loreto Street,
Morning Breeze [S]ubdivision, Caloocan City (Tsn., January 8, 1998, p. 3). Thereupon,...
accused-appellant Alexander Taño, a relative of Ana's husband Gerry Marinay (Gerry), arrived
at said shop (ibid., p. 4). Alexander Taño then asked Amy about the time when Gerry would be
coming home, to which she replied, 10:00 p.m. (id.). He then asked about the time when Ana...
would be coming home and Amy replied that she did not know (id.).
"Thereafter, but still on the same date, Alexander Taño kept on going in and out of the Video
Shop, and on the last time that he went inside said shop, he jumped over the counter of the
shop to where Amy was and seized the latter by placing one of his arms around Amy['s] neck,...
while his other hand held a knife which he poked at her neck (id., pp. 4-5).
"Terrified by the attack, Amy started shouting for help but Alexander Taño increased the volume
of a karaoke which was on at the time to drown Amy's cries for help
Alexander Taño then dragged Amy to the kitchen of the shop where, at knife point, he ordered
the latter to undress and he thereafter started raping her
However, while Alexander Taño was raping Amy, somebody knocked at the door of the shop
prompting the former to stop what he was doing and ordered Amy to put on her clothes
"Alexander Taño then directed Amy to go upstairs to the second floor of the shop to change
clothes as he will be taking her with him (id., p. 7). But suddenly thereafter, Taño pulled her
down and punched her in the stomach thrice causing her to lose her balance (id.). Taño then...
started cursing her and again placed himself on top of her while poking a knife at her neck (id.).
Amy then pleaded with Taño to just take anything inside the shop and to spare her life, to which
Taño replied 'no, I will not leave you here alive.
Thereafter, Taño went upstairs and looted the place of valuables belonging to Amy's employer,
Ana. Amy, herself lost her ring, bracelet and wristwatch during the incident in question (id., p.
10).
"At about 9:00 o'clock p.m. of the same day, Amy's employer Ana arrived and found the shop in
disarray with the 'karaoke' in full volume (Tsn., 13, 1998, pp. 2-4). After turning off the
'karaoke['], Ana proceeded to the toilet where she found Amy bathed in blood (ibid.,... p.4).
"Ana immediately sought the help of Barangay officials of the place and Amy was brought to the
'MCU' Hospital where she was initially treated of her injuries (id., p. 5). Amy was, later on,
transferred to Jose P. Reyes Memorial Medical Center (JPRMMC) where she was confined
for... four (4) days.
Issues:
The lower court erred in not taking into consideration the testimonies of Dr. Godofredo
Balderosa and Dr. Maria Redencion Bukid-Abella which negate the rape [charge] imputed
against the accused.
Ruling:
After a careful review of the evidence on record, the Court finds that (a) appellant is guilty of two
separate crimes -- rape and robbery, (b) dwelling cannot be appreciated as an aggravating
circumstance, and (c) the proper penalty for rape is reclusion perpetua, not... death.
Appellant contends that the trial court failed to give due credence to the testimonies of Dr.
Godofredo Balderosa and Dr. Ma. Redencion Bukid-Abella, who both examined and treated
Amy de Guzman's physical injuries immediately after the incident. Both doctors similarly
stated... that the victim complained to them of physical assault and attempted rape only, not of
consummated rape.[11] Additionally, the findings of NBI Medico-Legal Officer Aurea Villena
were allegedly inconclusive as to whether there was sexual intercourse between... the appellant
and the victim.[12] Their testimonies supposedly bolster appellant's innocence of the rape
charge.
Otherwise stated, appellant claims that the failure of Amy de Guzman to immediately disclose
the rape to her examining physicians could only mean that she was not in fact sexually
assaulted.
In many criminal cases, especially of rape, this Court has acknowledged that the vacillation of
the victim in reporting the crime to the authorities is not necessarily an indication of a fabricated
charge. Neither does it always cast doubt on the credibility on the complaining... witness.[13]
The initial reluctance of a young, inexperienced lass to admit having been ravished is normal
and natural.[14] The Court takes judicial notice of the Filipina's inbred modesty and shyness and
her antipathy in publicly... airing acts which blemish her honor and virtue.[15] She cannot be
expected to readily reveal the fact of her sexual violation to total strangers.
It is thus perfectly understandable and consistent with common experience that Amy initially
tried to downplay the assault upon her chastity by telling the doctors that there was no
consummation of the act.
Time-honored is the doctrine that no young and decent woman would publicly admit that she
was ravished and her virtue defiled, unless such was true, for it would be instinctive for her to
protect her honor.[18] No woman would concoct a story of defloration,... allow an examination of
her private parts and submit herself to public humiliation and scrutiny via an open trial, if her
sordid tale was not true and her sole motivation was not to have the culprit apprehended and
punished
It is a basic rule, founded on reason and experience, that... when a victim testifies that she has
been raped, she effectively says all that is necessary to show that rape was indeed committed.
[21]
In the case at bar, we find no reason to deviate from these doctrines. Amy de Guzman's
straightforward and convincing testimony, which will be detailed later, bears no badge of
material inconsistency which would bring doubt to its veracity. She stood firm on her tale
throughout... her court appearance. The trial judge observed her "to be candid, straightforward,
spontaneous and frank x x x [and she] remained consistent and unwavering despite the rigid
cross-examinations of the defense counsel x x x."[22]
Besides, no ill motive was imputed on her. Appellant offers us no plausible explanation why
Amy de Guzman cried rape against him. We believe she did so in order to bring out the truth
and to obtain justice.
Appellant's contention that the absence of genital and other injuries on Amy's body proves his
innocence is unacceptable. Time and again, we have ruled that hymenal laceration is not an
element of rape.[23] The victim need not sustain genital injuries, for... even the slightest
penetration of the labia by the male organ is equivalent to consummated rape
We do not, however, agree with the trial court that appellant is guilty of the special complex
crime of robbery with rape. This felony contemplates a situation where the original intent of the
accused was to take, with intent to gain, personal property belonging to another; and... rape is
committed on the occasion thereof or as an accompanying crime.[38]
Such factual circumstance does not obtain here. As related by Private Complainant Amy de
Guzman, accused-appellant suddenly jumped over the counter, strangled her, poked a knife at
the left side of her neck, pulled her towards the kitchen where he forced her to undress, and...
gained carnal knowledge of her against her will and consent. Thereafter, he ordered her to
proceed upstairs to get some clothes, so he could bring her out, saying he was not leaving her
alive. At this point, appellant conceived the idea of robbery because, before they could... reach
the upper floor, he suddenly pulled Amy down and started mauling her until she lost
consciousness; then he freely ransacked the place. Leaving Amy for dead after repeatedly
banging her head, first on the wall, then on the toilet bowl, he took her bracelet, ring and...
wristwatch. He then proceeded upstairs where he took as well the jewelry box containing other
valuables belonging to his victim's employer.
Under these circumstances, appellant cannot be convicted of the special complex crime of
robbery with rape. However, since it was clearly proven beyond reasonable doubt that he raped
Amy de Guzman and thereafter robbed her and Ana Marinay of valuables totaling P16,000, he...
committed two separate offenses -- rape with the use of a deadly weapon and simple robbery
with force and intimidation against persons.
In the case at bar, the building where the two offenses were committed was not entirely for
dwelling purposes. The evidence shows that it consisted of two floors: the ground floor, which
was being operated as a video rental shop, and the upper floor, which was used as a...
residence. It was in the video rental shop where the rape was committed. True, the victim was
dragged to the kitchen and toilet but these two sections were adjacent to and formed parts of
the store. Being a commercial shop that caters to the public, the video rental outlet was... open
to the public. As such, it is not attributed the sanctity of privacy that jurisprudence accords to
residential abodes. Hence, dwelling cannot be appreciated as an aggravating circumstance in
the crime of rape.

- People vs. Reyes, G.R. No. 153119


- People vs. Evangelio, G.R. No. 181902
Crime committed with abuse of confidence or obvious
ungratefulness - People vs. Caliso, G.R. No. 37271
- People vs. Lora, G.R. No. L-49430
- People vs Mandolado, G.R. No. L-51304-05
- People v. Arrojado, G.R. No. 130492

Crime committed in the palace of the chief executive or in his presence, or where
public authorities are engaged in the discharge of their duties, or in a place
dedicated to religious worship
- People vs Jaurigue, C.A. No. 384
Crime committed in the night time, or in an uninhabited place (despoblado), or
by a band –
People vs. Laguardia GR No. L-63423
-People vs Marra, G.R. No. 108494
-People vs Desalisa, G.R. No. 95262
-People v. Librando, G.R. No. 132251
-People vs. Ventura, G.R. No. 148145-46
Crime committed with the aid of armed men, or persons who insure or afford
impunity - People vs. Lozano, GR Nos. 137370-71
- People vs. Carino, G.R. No. 131117
- People vs. Enoja, GR No. 204894
Crime committed in consideration of a price, reward, or promise
- U.S. v. Gamao, G.R. No. L-6942
Committed with evident premeditation
- People vs. Lug-aw, G.R. No. 85735
- People vs Ilao-a, G.R. No. 94308
- People vs. Zeta, G.R. No. 178541
- People vs. Falabrica, GR No. 188124
- People vs. Sanchez, G.R. No. 188610 - People
v. Gabrino, G.R. No. 189981 - PP vs. Alinao, GR
No. 191256 - PP vs. Sabangan, GR No. 191722
Craft, fraud or disguise be employed - People vs.
Marquez, 117 SCRA 165 (1982) - People vs. Empacis,
222 SCRA 59 (1994) - People v. Labuguen G.R. No.
127849
Act be committed with treachery (alevosia)
- People vs Jose, GR No. 130666
- People vs Pansensoy, GR No. 140634
- People vs. Villacorta, G.R. No. 186412
- People vs Yanson, GR No. 179195
- People vs. Agacer, G.R. No. 177751
- People vs Nelmida, GR No. 184500
- People vs. Aquino G.R. No. 201092
- People vs. Gunda, G.R. No. 195525
- People vs. Feliciano, G.R. No. 196735
- People vs Matibag G.R. No. 206381
Ignominy
- People vs. Jose, et. al, 37 SCRA 450 (1971)
- People v. Bumidang G.R. No. 130630

- PP vs. Cachola, GR Nos. 148712-15

PEOPLE VS. CACHOLA EN BANC G.R. Nos. 148712-15, January 21, 2004 PEOPLE OF
THE PHILIPPINES, APPELLEE, VS. DOMINADOR CACHOLA Y SALAZAR, ERNESTO
AMAY Y PASCUA, NESTOR MARQUEZ Y MANUEL, BENJAMIN LAEGEN Y CAMADO,
RODOLFO SAGUN Y JIMENEZ, RODEMIR GUERZO Y LATAOAN, MELLKE IGNACIO Y
SALVADOR, AND NELSON C. ECHABARIA, APPELLANTS.

(People vs. Cachola G.R. Nos. 148712-15 January 21, 2004)


DECISION
PER CURIAM:
In just an instant, 12-year-old Jessie E. Barnachea lost his mother, an elder brother, an uncle,
and a cousin as a result of the carnage that took place at around 6:00 p.m. of 28 December
1999 right inside their house in Barangay Calumbaya, Bauang, La Union. Their horrible death
was attributed to herein appellants, who, however, pleaded not guilty to the four separate
informations for murder.
At the trial before the Regional Trial Court of Bauang, La Union, Branch 67, the prosecution
presented as witnesses Jessie and his brother and neighbors, as well as several police officers.
Their testimonies disclose as follows:
Jessie was about to leave their house to watch cartoons in his uncle's house next door when
two armed men suddenly entered the front door of their house. The two ordered Jessie to drop
to the floor, and then hit him in the back with the butt of a long gun. Without much ado, the
intruders shot to death Jessie's uncle, Victorino V. Lolarga, who was then in the living room.
Jessie forthwith crawled and hid under a bed, from where he saw the feet of a third man who
had also entered the house. The men entered the kitchen and continued shooting. When the
rampage was over and after the malefactors had already departed, Jessie came out of his
hiding place and proceeded to the kitchen. There he saw his mother, Carmelita Barnachea; his
brother Felix Barnachea, Jr.; and his cousin Rubenson Abance - all slaughtered.[1]
Meanwhile, Jessie's eldest brother, Robert E. Barnachea, was in his uncle's house watching
television with his aunt and young cousins when he sensed a commotion outside. When Robert
went out to see what was transpiring, he saw armed men running towards their house. One of
them turned and pointed a gun at him, prompting him to scamper away and hide at the back of
his uncle's house. From where he was hiding, he noticed a stainless jeep, with blue rim and
marking "fruits and vegetables dealer," parked in front of the fence of their house. Standing
behind the jeep were three armed men wearing bonnets, with only their nose and eyes
exposed. In the next instant, he heard gunshots and then saw men running from his house. The
men hurriedly boarded the jeep and left the place.[2]
The jeep did not go unnoticed by the neighbors. Russel Tamba was with some friends in front of
Roda's Store, around 100 meters away from the Barnachea residence, when the jeep passed
by very slowly going towards the Barnachea residence. According to him, the jeep had a
marking "El Shaddai" in front, aside from the marking "fruits and vegetables dealer" on the
sides.[3] Francisco Andrada was also talking with some people in front of the Calumbaya
Barangay Hall, only five meters away from Roda's Store, when he noticed that jeep, with the "El
Shaddai" marking, pass by.[4] Not long after, both heard gunshots and later saw the jeep pass
by again, this time running very fast.[5]
The incident was immediately reported to the police, and the description of the "El Shaddai"
jeep used by the malefactors was relayed through radio to the police stations in the province of
La Union.[6] At around 7:45 p.m., the jeep was intercepted at a checkpoint set up in the highway
by the police force in Aringay, La Union. On board were the eight appellants. No firearms were
found in the vehicle. The jeep and the eight appellants were thereafter brought to the Aringay
police station and then turned over to the Bauang police.[7]
When the Chief of Police of Bauang Benjamin M. Lusad was informed of the apprehension of
the eight appellants, he went to the Barnachea residence, where he came to know that Jessie
was an eyewitness. He invited Jessie to ride with him to pick up the suspects. While Lusad was
supervising the boarding of the suspects into the vehicle, Jessie was in another police vehicle
with PO3 Juan Casern, Jr., to see whether he could recognize any one of the eight men. Jessie
pointed to Dominador Cachola and Ernesto Amay as the two armed men who entered his house
and killed his relatives. During the police line-up at the Bauang Police Station, Jessie again
identified Cachola and Amay as the assailants.[8] The next day, when the police conducted the
third confirmatory investigation, which was to present Jessie with photographs of the suspects,
Jessie identified the two for the third time.
The eight appellants were thereafter subjected to paraffin test. But only the right hands of
Cachola and Amay yielded positive results for gunpowder nitrates.[9]
The Death Certificates attest to the gruesome and merciless killings. Carmelita sustained one
gunshot wound on her head and three on her body;[10] Felix, Jr., two gunshot wounds on his
head and on his body, and stab wounds on his chest and arms;[11] Victorino, two gunshot
wounds on his head, three on his body, and with his penis excised;[12] Rubenson, one gunshot
wound on his head and a stab wound that lacerated his liver.[13]
The testimonies of the other prosecution witnesses were dispensed with upon the stipulation by
the public prosecutor and the counsels for the appellants that the nature of their testimonies
would be that (1) PO3 Juan Casern, Jr., was inside the police car with Jessie when the latter
recognized appellants Cachola and Amay; (2) Mark Garcia would corroborate the testimony of
Felix Andrada regarding the description of the jeep; (3) Felix Barnachea, Sr., suffered actual
damages amounting to P177,000 as a result of the death of his wife Carmelita and son Felix,
Jr.; (4) a police officer of Aringay, La Union, flagged down the jeep at the checkpoint and saw
the appellants on board; and (5) a police officer of Bauang, La Union, would identify the pictures
taken at the crime scene.
After the prosecution had rested its case, the defense counsels orally asked for leave of court to
file a demurrer to evidence. The trial court denied the motion outright and set the schedule for
the presentation of the evidence for the defense.[14] Instead of presenting their evidence,
however, the appellants, through their respective counsels, filed a Demurrer to Evidence[15]
even without leave of court.
On 26 September 2000, the trial court rendered a decision[16] (1) convicting (a) Cachola and
Amay, as principals, of four counts of murder and sentencing them to suffer four counts of the
supreme penalty of death; and (b) Marquez, Laegen, Sagun, Guerzo, Ignacio, and Echabaria,
as accomplices, of four counts of murder and sentencing them to suffer four counts of the
indeterminate penalty of eight years of prision mayor as minimum to twelve years and one day
of reclusion temporal as maximum; and (2) ordering all of them to pay the heirs of the victims a
total of P300,000 as death indemnity; P200,000 as moral damages; and P177,000 as actual or
compensatory damages.
Before us on automatic review, appellants Cachola, Sagun, Ignacio, and Marquez contend that
the trial court erred (1) in finding conspiracy among them and their co-appellants; (2) in finding
proof beyond reasonable doubt; and (3) in not dismissing the informations outright despite a
motion before arraignment, there having been a clear illegal arrest and denial of due process.
As for appellants Amay, Guerzo, Laegen, and Echabaria, they assert that the trial court erred
(1) in finding appellant Amay guilty beyond reasonable doubt as principal in the crime of murder;
and (2) in not acquitting appellants Guerzo, Laegen, and Echabaria for insufficiency of evidence
and on reasonable doubt.
In its Consolidated Reply Brief, the Office of the Solicitor General (OSG) recommends the
affirmance of the conviction for murder of appellants Cachola and Amay, and the acquittal of the
other appellants for failure of the prosecution to establish their identity and participation beyond
reasonable doubt.
We agree with the recommendation of the OSG to acquit appellants Sagun, Ignacio, Marquez,
Guerzo, Laegen, and Echabaria. Upon a thorough review of the records of the case, we found
nothing that would show their participation in the commission of the crimes. Not one of the
prosecution witnesses identified them as among the malefactors who were at the Barnachea
residence on that fateful day. Surprisingly, even as the trial court declared that the prosecution
failed to establish the actual participation of the other appellants in the commission of the crime,
it found that "they cooperated in the execution of the offense by previous or simultaneous
acts."[17] It appears, however, that the only reason why they were implicated was that they
were with Cachola and Amay on board the jeep that was intercepted in Aringay, La Union,
almost two hours after the killings. What constitute previous or simultaneous acts that would
make them liable as accomplices are not found in the decision or in any evidence on record.
To hold a person liable as an accomplice, two elements must concur: (1) community of design,
which means that the accomplice knows of, and concurs with, the criminal design of the
principal by direct participation; and (2) the performance by the accomplice of previous or
simultaneous acts that are not indispensable to the commission of the crime.[18] In the present
case, neither element was proved. The mere presence of the six appellants in the company of
appellants Cachola and Amay on board a jeep is not evidence of their knowledge of, or assent
to, the criminal design to perpetuate the massacre.[19] That they were found to be with
appellants Cachola and Amay almost two hours after the commission of the crime does not
constitute previous or simultaneous act. Absent a link between the crime and their presence in
the jeep two hours later, we cannot consider their participation even as accessories to the
crime.
It is a basic evidentiary rule in criminal law that the prosecution has the burden of proving the
guilt of the accused beyond reasonable doubt.[20] If the prosecution fails to discharge that
burden, the accused need not present any evidence.[21] Thus, for utter lack of evidence against
the six appellants, their acquittal is in order.
However, as regards appellants Cachola and Amay, we concur with the trial court and the OSG
that the prosecution had presented sufficient evidence to prove their guilt beyond reasonable
doubt. The credible testimony of, and positive identification by, Jessie Barnachea, which are
corroborated by forensic evidence, i.e., the positive results of the paraffin test on the right hands
of Cachola and Amay, constitute sufficient evidence to sustain their conviction.
As to the credibility of Jessie Barnachea, the trial court made the following observations:
The Court observed the demeanor of Jessie Barnachea on the witness stand and ... did not
observe any indication of falsehood in his narration. He showed obvious readiness to answer
questions propounded to him. His reactions and answers to the questions displayed evident
respect for truth. He remained consistent on cross-examination. He positively identified accused
Amay and Cachola as the one who shot and killed his family. The Court did not observe any
hesitancy or indication of uncertainty - and his recital of the events appeared spontaneous.[22]
There is nothing on record that gives this Court cause to interfere with the trial court's
determination of the credibility of Jessie. Indeed, his testimony was unwavering despite
attempts of the defense counsels to confuse or trap him. The alleged inconsistency between
Jessie's sworn statement and testimony on the number of malefactors, if at all, does not detract
from his credibility. That Jessie saw two armed men enter his house is clear. While the defense
claims ambiguity as to the presence of a third man, Jessie's statement easily reveals that the
third man was not immediately mentioned because he (the third man) only followed the two and
Jessie did not see his face.
It is also pointed out that Jessie's identification of Cachola and Amay runs counter to Robert's
testimony that the armed men were wearing bonnets. Again, from their testimonies, it is
apparent that the brothers saw different men. Besides, Robert also stated that one of the men
did not have his head covered. As to the alleged improbability of the lookouts wearing bonnets
while the principal shooters were unmasked, or of the malefactors sparing Jessie, suffice it to
say that such circumstances are not so incredible as to cast reasonable doubt on the truth of the
narrated events.
In sum, none of the alleged inconsistencies, minor as they are, could leave us with doubt that
Jessie was present in his house and saw armed men shoot his relatives. Barely two hours had
passed since he witnessed the gruesome murders when Jessie identified appellants Cachola
and Amay as the malefactors. Reasonably, the memory of their faces was still fresh on his
mind. Moreover, Jessie identified the two appellants two times more at the police station and
once in open court, and he never faltered in his identification.
Significantly, the appellants have not imputed any ill motive to Jessie for testifying against
Cachola and Amay. Where there is no evidence to show a doubtful reason or improper motive
why a prosecution witness should testify against the accused or falsely implicate him in a crime,
the said testimony is trustworthy and should be accorded full faith and credit.[23]
In all, there does not appear on record to be "some fact or circumstance of weight and influence
which the trial court has overlooked or the significance of which it has misapprehended or
misinterpreted. "[24] We rely, therefore, on the competence of the trial court to decide the
question of credibility of the witnesses, having heard them and observed their deportment and
manner of testifying during the trial."[25]
The reliance by appellant Cachola on People v. Teehankee[26] is misplaced. In that case the
negative result of the paraffin test did not preclude a finding of guilt by the trial court, the reason
being that the accused was tested for the presence of nitrates only after more than 72 hours
had lapsed from the time of the shooting. In the present case, the paraffin test was conducted
on the same night the shooting incident occurred; hence, the lapse of only a few hours
increases its reliability. While the presence of nitrates on accused's hand is not conclusive of
guilt, it bolsters the testimony of an eyewitness that the accused fired a gun.
As to whether the trial court erred in not allowing the appellants to present evidence after filing
their demurrer to evidence without leave of court, then Section 15, Rules 119 of the Rules of
Court[27] is clear on the matter, thus:
SEC. 15. - Demurrer to evidence. - After the prosecution has rested its case, the court may
dismiss the case on the ground of insufficiency of evidence: (1) on its own initiative after giving
the prosecution an opportunity to be heard; or (2) on motion of the accused filed with prior leave
of court.
If the court denies the motion for dismissal, the accused may adduce evidence in his defense.
When the accused files such motion to dismiss without express leave of court, he waives the
right to present evidence and submits the case for judgment on the basis of the evidence for the
prosecution. (Underscoring supplied).
The filing by the appellants of a demurrer to evidence in the absence of prior leave of court was
a clear waiver of their right to present their own evidence. To sustain their claim that they had
been denied due process because the evidence they belatedly sought to offer would have
exculpated them would be to allow them to "wager on the outcome of judicial proceedings by
espousing inconsistent viewpoints whenever dictated by convenience."[28] Furthermore, it
cannot be said that the waiver was not clear. The trial court postponed the hearings on the
motion for demurrer, even after leave of court had been denied, and then granted extensions to
Amay until he finally adopted the position of his co-appellants. At no time other than in this
automatic review was there any attempt that is contrary to the waiver of the presentation of
evidence.
Neither can the question of the legality of the warrantless arrest of the appellants be raised for
the first time before this Court. As arrests fall into the question of the exercise by the trial court
of its jurisdiction over the person of the accused, the question should have been raised prior to
their arraignment. That the appellants objected to the arrests prior to the arraignment[29] is
unsubstantiated. Their claim that they requested an extension of time to file a motion to quash
the information or to dismiss the case,[30] which the trial court allegedly denied, cannot save the
day for them. The fact remains that before arraignment, no such motion was filed. Even
assuming that their arrest was illegal, their act of entering a plea during their arraignment
constituted a waiver of their right to question their arrest.[31]

We now discuss the circumstances that attended the commission of the crimes.
The information alleges the qualifying circumstances of treachery and evident premeditation.
There is no doubt that the killings were done with treachery, considering that the assailants
suddenly barged in and immediately went on a shooting rampage. We have time and again
ruled that when the attack is sudden and unexpected, there is treachery.[32] The presence of
even this single qualifying circumstance is sufficient to qualify the killing to murder.[33]
As to the qualifying circumstance of evident premeditation, we find the same lacking, for there is
no evidence of planning or preparation to kill, much less of the time when the plot was
conceived.[34]
It may not be amiss to mention that the death certificate of Victorino Lolarga reveals that his
penis was excised. One may wonder whether such circumstance amounted to ignominy that
can aggravate the offense.
For ignominy to be appreciated, it is required that the offense be committed in a manner that
tends to make its effect more humiliating, thus adding to the victim's moral suffering. Where the
victim was already dead when his body or a part thereof was dismembered, ignominy cannot be
taken against the accused.[35] In this case, the information states that Victorino's sexual organ
was severed after he was shot and there is no allegation that it was done to add ignominy to the
natural effects of the act. We cannot, therefore, consider ignominy as an aggravating
circumstance.
However, as regards Carmelita and Felix, Jr., we appreciate the aggravating circumstance of
dwelling, since it was alleged in the information and proved during the trial that they were killed
inside their house. Appellants Cachola and Amay, therefore, violated the sanctity of the said
victims' home.
Article 248 of the Revised Penal Code provides that the penalty for murder is reclusion perpetua
to death. In conjunction, Article 63 of the Revised Penal Code provides that when the law
prescribes two indivisible penalties, the greater penalty shall be imposed when in the
commission of the deed, there is present one aggravating circumstance. In the cases of
Carmelita and Felix Jr., in Criminal Cases Nos. 2324 and 2325, there is one aggravating
circumstance and no mitigating circumstance to offset it; hence, the higher penalty of death
imposed by the trial court stands.
Three members of the Court maintain their adherence to the separate opinions expressed in
People vs. Echegaray[36] that Republic Act No. 7659, insofar as it prescribes the penalty of
death, is unconstitutional; nevertheless they submit to the ruling of the majority that the law is
constitutional and that the death penalty should accordingly be imposed.
But in the cases of Victorino and Rubenson, in Criminal Cases Nos. 2323 and 2326, there being
no aggravating or mitigating circumstance, the penalty should be reclusion perpetua, which is
the lower of the two indivisible penalties prescribed by law.

As regards the civil liability of appellants Cachola and Amay, we hold them jointly and severally
liable to pay the heirs of each of the victims death indemnity and moral damages each in the
amount of P50,000, or a total of P400,000. They are further ordered to pay the respective heirs
of Carmelita and Felix Jr. exemplary damages in the amount of P25,000, or a total of P50,000,
in view of the presence of one aggravating circumstance in the commission of the crime against
the said victims. As to the claim for damages by Felix Barnachea Sr. in the amount of P177,000,
we sustain the same even if only a list of expenses,[37] not official receipts, was submitted
because such amount was admitted by the defense during the trial.[38] Moreover, although
there is no evidence as to the amount spent as a result of the death of Victorino and Rubenson,
their respective heirs shall be awarded temperate damages in the amount of P25,000, since
they clearly incurred funeral expenses.[39]

WHEREFORE, the assailed decision dated 26 September 2000 of the Regional Trial Court of
Bauang, La Union, Branch 67, is hereby AFFIRMED insofar as DOMINADOR CACHOLA y
SALAZAR and ERNESTO AMAY y PASCUA are found GUILTY of four counts of murder in
Criminal Cases Nos. 2323-26 and sentenced to suffer the supreme penalty of death in Criminal
Cases Nos. 2324 and 2325. The said decision is, however, MODIFIED in that they are (1)
sentenced to suffer the penalty of reclusion perpetua, instead of death, in Criminal Cases Nos.
2323 and 2326; and (2) ordered to pay, jointly and severally, the following damages:
P50,000 as death indemnity in favor of the heirs of each victim, or a total of P200,000;
P50,000 as moral damages in favor of the heirs of each victim, or a total of P200,000;
P25,000 as exemplary damages in favor of the respective heirs of Carmelita Barnachea and
Felix Barnachea Jr., or a total of P50,000;
P177,000 as actual damages in favor of the heirs of Carmelita Barnachea and Felix Barnachea
Jr.; and
P25,000 as temperate damages in favor of the respective heirs of Rubenson Abance and
Victorino Lolarga, or a total of P50,000.
The assailed decision is REVERSED insofar as appellants NESTOR MARQUEZ y MANUEL,
BENJAMIN LAEGEN y CAMADO, RODOLFO SAGUN y JIMENEZ, RODEMIR GUERZO y
LATAOAN, MELLKE IGNACIO y SALVADOR, and NELSON C. ECHABARIA are concerned,
and another one is hereby rendered (1) acquitting them of the crimes charged for insufficiency
of evidence; (2) ordering their immediate release from confinement unless their further detention
is warranted by virtue of any lawful cause; and (3) directing the Director of the Bureau of
Corrections to submit a report on their release within five days from notice hereof.

Costs de oficio.

SO ORDERED.

Davide, Jr., C.J., Puno, Vitug, Panganiban, Quisumbing, Ynares-Santiago, Sandoval-Gutierrez,


Carpio, Austria-Martinez, Corona, Carpio-Morales, Callejo, Sr., Azcuna, and Tinga, JJ., concur.

Crime be committed after an unlawful entry. - People vs.


Baello, 224 SCRA 218 (1993)
Cruelty - People vs. Ilao-a, 233 SCRA 231 (1994) -
People v. Catian G.R. No. 139693 - People v.
Guerrero G.R. No. 134759 - Simangan vs. People,
G.R. No. 157984
E. Chapter Five: Alternative Circumstances
Art. 15 - People vs. Fontillas, 638 SCRA 721 -
People vs. Mondigo, 543 SCRA 384 - People vs.
Abello, 582 SCRA 378 - People vs. Latag, 420
SCRA 695 - People vs. Sumarago, 422 SCRA
324 - People vs. Ulit, 423 SCRA 374 - People vs.
Lauas, 58 Phil. 742 - People vs. Semanada, 103
Phil. 790

IV. Title Two: Persons Criminally Liable


Arts. 16 – 20
Principals - People vs Quillosa, GR No. 115867 -
People vs. Janjalani, GR No. 188314 - Guillergan
vs. People, GR No. 185493 - Ambagan vs
People, GR NO. 204481-82
Accomplices - People vs. Mandolado, 123 SCRA 133
(1983) - People vs Doctolero, 193 SCRA 632 (1991) -
People vs Tamayo, GR No. 138608 - People vs. Cruz,
Jr., GR No. 168446 - People vs. PO1 Eusebio G.R. No.
182152 - People vs. Salvador, GR No. 201443 -
People vs. Gambao, GR No. 172707
Accessories - People vs. Inovera, et.al., 65 OG 3168 - Viño vs.
People, G.R. No. 84163 - People vs. Ortega Jr. et.al., G.R. No.
116736, July 24, 1997 - People vs Talingdan, 84 SCRA 19
V. Title Three: Penalties
A. Chapter One: Penalties in general
Arts. 21 - U.S. vs. Macasaet, 11 Phil.
447
Art. 22 - Art. 62, RPC - People v.
Villaraza, 81 SCRA 95
Retroactive application of special penal laws -
Gonzales v. Court of Appeals, 277 SCRA 518 –

Go v. Dimagiba, 460 SCRA 451 (2005)

SUSAN GO v. FERNANDO L. DIMAGIBA, GR NO. 151876, 2005-06-21

Facts:

Respondent Fernando L. Dimagiba issued to Petitioner Susan Go thirteen (13) checks which,
when presented to the drawee bank for encashment or payment on the due dates, were
dishonored for the reason "account closed.
Dimagiba was subsequently prosecuted for 13 counts of violation of BP 22
On February 27, 2001, Dimagiba filed a Motion for Reconsideration of the MTCC Order. He
prayed for the recall of the Order of Arrest and the modification of the final Decision, arguing
that the penalty of fine only, instead of imprisonment also, should have been imposed on... him.
The RTC held that this rule should be retroactively applied in favor of Dimagiba.[23] It further
noted that (1) he was a first-time... offender and an employer of at least 200 workers who would
be displaced as a result of his imprisonment; and (2) the civil liability had already been satisfied
through the levy of his properties.[24]... questions of law

Issues:

whether the Petition for habeas corpus was validly granted

Ruling:

The issue of retroactivity of SC-AC No. 12-2000 was settled in De Joya v. Jail Warden
In the present case, the Petition for a writ of habeas corpus was anchored on the ruling in Vaca
and on SC-AC No. 12-2000, which allegedly prescribed the imposition of a fine, not
imprisonment, for convictions under BP 22.
Respondent sought the retroactive... effect of those rulings, thereby effectively challenging the
penalty imposed on him for being excessive.
The following alternative penalties are imposable under BP 22: (1) imprisonment of not less
than 30 days, but not more than one year; (2) a fine of not less or more than double the amount
of the check, a fine that shall in no case exceed P200,000; or (3) both such fine and...
imprisonment, at the discretion of the court
When the circumstances of the case clearly indicate good faith or a clear mistake of fact...
without taint of negligence, the imposition of a fine alone may be considered as the preferred
penalty.
Should the... judge deem that imprisonment is appropriate, such penalty may be imposed
SC-AC No. 12-2000 did not delete the alternative penalty of imprisonment. The competence to
amend the law belongs to the legislature, not to this Court.
Petitioners argue that respondent is not entitled to the benevolent policy enunciated in SC-AC
No. 12-2000, because he is not a "first time offender... t is the trial court's discretion to impose
any penalty within the confines of the law. SC-AC No. 13-2001 explains thus:... good faith or a
clear mistake of fact... without taint of negligence, the imposition of a fine alone should be
considered as the more appropriate penalty. Needless to say, the determination of whether the
circumstances warrant the imposition of a fine alone rests solely upon the Judge.
The Judges concerned, may in the exercise of sound discretion, and taking into consideration
the peculiar circumstances of each case, determine whether the imposition of a fine alone would
best serve the interests of justice, or whether forbearing to impose imprisonment... would
depreciate the seriousness of the offense, work violence on the social order, or otherwise be
contrary to the imperatives of justice
The Court notes that the Petition for a writ of habeas corpus relied mainly on the alleged
retroactivity of SC-AC No. 12-2000, which supposedly favored BP 22 offenders
The rule on retroactivity states that criminal laws may be applied retroactively if favorable to the
accused.
SC Admin. Circular No. 12-2000 is not a penal law; hence, Article 22 of the Revised Penal Code
is not applicable.
The circular applies only to those cases pending as of the date of its effectivity and not to cases
already terminated by final judgment.
SC Admin. Circular No. 12-2000 merely lays down a rule of preference in the application of the
penalties for violation of B.P. Blg. 22. It does not amend B.P. Blg. 22, nor defeat the legislative
intent behind... the law. SC Admin. Circular No. 12-2000 merely urges the courts to take into
account not only the purpose of the law but also the circumstances of the accused -- whether he
acted in good faith or on a clear mistake of fact without taint of negligence -- and such other...
circumstance which the trial court or the appellate court believes relevant to the penalty to be
imposed.
Because the Circular merely lays down a rule of preference, it serves only as a guideline for the
trial courts. Thus, it is addressed to the judges, who are directed to consider the factual
circumstances of each case prior to imposing the appropriate penalty.
In... other words, the Administrative Circular does not confer any new right in favor of the
accused, much less those convicted by final judgment.
Indeed, SC-AC No. 12-2000 necessarily requires a review of all factual circumstances of each
case. Such a review can no longer be done if the judgment has become final and executory.
Hence, RTC-Branch 5 did not have the jurisdiction to modify the lawful judgment in the guise of
granting a writ of habeas corpus.
Respondent seeks the retroactive application of SC-AC No. 12-2000 in his favor on the basis
alone of the alleged settlement of his civil liability.[
WHEREFORE, the Petition is GRANTED and the assailed Orders NULLIFIED.
Respondent's Petition for habeas corpus is hereby DENIED.
Principles:
writ of habeas corpus... required the imposition of a fine only... if the accused was not a
recidivist or a habitual delinquent.
The writ of habeas corpus applies to all cases of illegal confinement or detention in which
individuals are deprived of liberty.
The writ may not be availed of when the person in custody is under a judicial process or by
virtue of a valid judgment
However, as a post-conviction remedy, it may be allowed when, as a consequence of a judicial
proceeding, any of the following... exceptional circumstances is attendant: (1) there has been a
deprivation of a constitutional right resulting in the restraint of a person; (2) the court had no
jurisdiction to impose the sentence; or (3) the imposed penalty has been excessive, thus voiding
the... sentence as to such excess... it is merely... a rule of preference as to which penalty should
be imposed under the peculiar circumstances of a case.
The law punishes the act not as an offense against property but an offense against public
order."

B. Chapter Two: Classification of Penalties


Arts. 25 & 26 - R.A.
No. 9346
C. Chapter Three: Duration and Effect of Penalties
Arts. 27 – 29
- People vs. Penillos, 205 SCRA 546, 565 (1992)
- People v. Morilla, 715 SCRA 452, 463-465 (2014)
- Baking vs. Dir. of Prisons, 28 SCRA 851
- R.A. No. 7659
- People v. Bensig, G.R. No. 138989, Sept. 17, 2002
- People v. Gregorio, G.R. No. 109614-15, March 29, 1996

- R.A. No. 10592

[REPUBLIC ACT NO. 10592]

AN ACT AMENDING ARTICLES 29, 94, 97, 98 AND 99 OF ACT NO. 3815, AS AMENDED,
OTHERWISE KNOWN AS THE REVISED PENAL CODE

Be it enacted by the Senate and House of Representatives of the Philippines in Congress


assembled:

SECTION 1. Article 29 of Act No. 3815, as amended, otherwise known as the Revised Penal
Code, is hereby further amended to read as follows:

“ART. 29. Period of preventive imprisonment deducted from term of imprisonment. – Offenders
or accused who have undergone preventive imprisonment shall be credited in the service of
their sentence consisting of deprivation of liberty, with the full time during which they have
undergone preventive imprisonment if the detention prisoner agrees voluntarily in writing after
being informed of the effects thereof and with the assistance of counsel to abide by the same
disciplinary rules imposed upon convicted prisoners, except in the following cases:
“1. When they are recidivists, or have been convicted previously twice or more times of any
crime; and

“2. When upon being summoned for the execution of their sentence they have failed to
surrender voluntarily.

“If the detention prisoner does not agree to abide by the same disciplinary rules imposed upon
convicted prisoners, he shall do so in writing with the assistance of a counsel and shall be
credited in the service of his sentence with four-fifths of the time during which he has undergone
preventive imprisonment.

“Credit for preventive imprisonment for the penalty of reclusion perpetua shall be deducted from
thirty (30) years.

“Whenever an accused has undergone preventive imprisonment for a period equal to the
possible maximum imprisonment of the offense charged to which he may be sentenced and his
case is not yet terminated, he shall be released immediately without prejudice to the
continuation of the trial thereof or the proceeding on appeal, if the same is under review.
Computation of preventive imprisonment for purposes of immediate release under this
paragraph shall be the actual period of detention with good conduct time allowance: Provided,
however, That if the accused is absent without justifiable cause at any stage of the trial, the
court may motu proprio order the rearrest of the accused: Provided, finally, That recidivists,
habitual delinquents, escapees and persons charged with heinous crimes are excluded from the
coverage of this Act. In case the maximum penalty to which the accused may be sentenced is
lestierro, he shall be released after thirty (30) days of preventive imprisonment.”

SEC. 2. Article 94 of the same Act is hereby further amended to read as follows:

“ART. 94. Partial extinction of criminal liability. – Criminal liability is extinguished partially:

“1. By conditional pardon;

“2. By commutation of the sentence; and

“3. For good conduct allowances which the culprit may earn while he is undergoing preventive
imprisonment or serving his sentence.”

SEC. 3. Article 97 of the same Act is hereby further amended to read as follows:

“ART. 97. Allowance for good conduct. – The good conduct of any offender qualified for credit
for preventive imprisonment pursuant to Article 29 of this Code, or of any convicted prisoner in
any penal institution, rehabilitation or detention center or any other local jail shall entitle him to
the following deductions from the period of his sentence:

“1. During the first two years of imprisonment, he shall be allowed a deduction of twenty days for
each month of good behavior during detention;
“2. During the third to the fifth year, inclusive, of his imprisonment, he shall be allowed a
reduction of twenty-three days for each month of good behavior during detention;

“3. During the following years until the tenth year, inclusive, of his imprisonment, he shall be
allowed a deduction of twenty-five days for each month of good behavior during detention;

“4. During the eleventh and successive years of his imprisonment, he shall be allowed a
deduction of thirty days for each month of good behavior during detention; and

“5. At any time during the period of imprisonment, he shall be allowed another deduction of
fifteen days, in addition to numbers one to four hereof, for each month of study, teaching or
mentoring service time rendered.

“An appeal by the accused shall not deprive him of entitlement to the above allowances for good
conduct.”

SEC. 4. Article 98 of the same Act is hereby further amended to read as follows:

“ART. 98. Special time allowance for loyalty. – A deduction of one fifth of the period of his
sentence shall be granted to any prisoner who, having evaded his preventive imprisonment or
the service of his sentence under the circumstances mentioned in Article 158 of this Code, gives
himself up to the authorities within 48 hours following the issuance of a proclamation
announcing the passing away of the calamity or catastrophe referred to in said article. A
deduction of two-fifths of the period of his sentence shall be granted in case said prisoner chose
to stay in the place of his confinement notwithstanding the existence of a calamity or
catastrophe enumerated in Article 158 of this Code.

“This Article shall apply to any prisoner whether undergoing preventive imprisonment or serving
sentence.”

SEC. 5. Article 99 of the same Act is hereby further amended to read as follows:”

“ART. 99. Who grants time allowances. – Whenever lawfully justified, the Director of the Bureau
of Corrections, the Chief of the Bureau of Jail Management and Penology and/or the Warden of
a provincial, district, municipal or city jail shall grant allowances for good conduct. Such
allowances once granted shall not be revoked.”

SEC. 6. Penal Clause. – Faithful compliance with the provisions of this Act is hereby mandated.
As such, the penalty of one (1) year imprisonment, a fine of One hundred thousand pesos
(P100,000.00) and perpetual disqualification to hold office shall be imposed against any public
officer or employee who violates the provisions of this Act.

SEC. 7. Implementing Rules and Regulations. – The Secretary of the Department of Justice
(DOJ) and the Secretary of the Department of the Interior and Local Government (DILG) shall
within sixty (60) days from the approval of this Act, promulgate rules and regulations on the
classification system for good conduct and time allowances, as may be necessary, to implement
the provisions of this Act.

SEC. 8. Separability Clause. – If any part hereof is held invalid or unconstitutional, the
remainder of the provisions not otherwise affected shall remain valid and subsisting.

SEC. 9. Repealing Clause. – Any law, presidential decree or issuance, executive order, letter of
instruction, administrative order, rule or regulation contrary to or inconsistent with the provisions
of this Act is hereby repealed, modified or amended accordingly.

SEC. 10. Effectivity Clause. – This Act shall take effect fifteen (15) days from its publication in
the Official Gazette or in at least two (2) new papers of general circulation.

Arts. 30 – 39 - R.A.
No. 10159
Arts. 40 - 45 - B.P. Blg. 881, Sec. 118

- PDEA vs Brodett, GR No. 196390

PHILIPPINE DRUG ENFORCEMENT AGENCY v. RICHARD BRODETT, GR No. 196390,


2011-09-28

Facts:
accused, conspiring and confederating together and mutually helping and aiding each... other,
they not being authorized by law, did then and there wilfully, unlawfully, and feloniously sell,
trade, deliver and give away to another, sixty (60) pieces of blue-colored tablets with Motorala
(M) logos, contained in six (6) self-sealing transparent plastic... sachets... filed another
information charging only Brodett with a violation of Section 11 of R.A. No. 9165... accused, not
being authorized by law, did then and there, wilfully, unlawfully, and feloniously have in his...
possession
It contends that the CA gravely erred in its ruling; that the Honda Accord car, registered under
the... name of Myra S. Brodett (Ms.Brodett), had been seized from accused Brodettduring a
legitimate anti-illegal operation and should not be released from the custody of the law
Issues:
The decisive issue is whether or not the CA erred in affirming the orderfor the release of the car
to Ms.Brodett.

Ruling:

The petition is meritorious.


It is undisputed that the ownership of the confiscated car belonged to Ms. Brodett, who was not
charged either in connection with the illegal possession and sale of illegal drugs involving
Brodett and Joseph that were the subject of the criminal proceedings in the RTC, or even... in
any other criminal proceedings.
There is no question, for even PDEA has itself pointed out, that the text of Section 20 of R. A.
No. 9165relevant to the confiscation and forfeiture of the proceeds or instruments of the
unlawful act is similar to that ofArticle 45 of the Revised Penal Code
Such proceeds and instruments or tools shall be confiscated and forfeited in favor of the
Government, unless they be the property of a third person not liable for the offense, but those
articles which are not subject of lawful commerce shall be destroyed.
To bar the forfeiture of the tools and instruments belonging to a third person,therefore, there
must be an indictment charging such third person either as a principal, accessory, or
accomplice.
We agree with PDEA and the Office of the City Prosecutor.
Ordering the release of the car at that pointof the proceedings was... premature, considering
that the third paragraph of Section 20, supra, expressly forbids the disposition, alienation, or
transfer of any property, or income derived therefrom, that has been confiscated from the
accused charged under R.A. No. 9165... during the pendency of the proceedings in the
Regional Trial Court.
Indeed, forfeiture, if warranted pursuant to either Article 45 of the Revised Penal Code and
Section 20 of R.A. No. 9165, would be a part of the penalty to be prescribed.
The determination of whetheror not the car... would be subject of forfeiture could be made only
when the judgment was to be rendered in the proceedings. Section 20 is also clear as to this.
To release it before the... judgment is rendered is to deprive the trial court and the parties
access to it as evidence.
Court need not annul the assailed orders of the RTC, or reverse the decision of the CA.
acquitting both Brodettand Joseph... and further ordering the return to the accused of all non-
drug evidence except the buy-bust money and the genuine money
This the Court must now do in view of the question about the confiscation... and forfeiture of
non-drug objects being susceptible of repetition in the future.
We rule that henceforth the Regional Trial Courts shall comply strictly with the provisions of
Section 20 of R.A. No. 9165, and should not release articles, whether drugs or non-drugs, for
the duration of the trial and before the rendition of the judgment, even if owned by a... third
person who is not liable for the unlawful act.

IN VIEW OF THE FOREGOING, the petition for review is DENIED.

C. Chapter Four: Application of Penalties


Arts. 46 – 61 - R.A. No. 934 - People vs. Bernardo, G.R. No.
198789, June 3, 2013 - People vs. Hernandez, 69 Phil. 515
Art. 48 - People vs. Guillen, 85 Phil. 307 - U.S.
vs. Ferrer, 1 Phil. 56 - People vs. Toling, 62
SCRA 17 - Ivler vs. San Pedro, 635 SCRA 191 -
Domingo vs. People, 603 SCRA 488 - People v.
Nelmida, 694 Phil. 529 (2012)
Delitocontinuado - Santiago v. Garchitorena, 228
SCRA 214
Arts. 62 -72 - People vs. Escares, 102 Phil. 677 - p.
482 - People vs. Jose, 37 SCRA 450
Arts. 73 -77 - People v. Ducosin, G.R. No. L-38332, December 14, 1933 -
Argoncillo v. People, 292 SCRA 313 - People vs. Temporada, 574
SCRA 258 - People vs. Gabres, 267 SCRA 581 - SC A.M. No. 15-
08-02 (Aug. 4, 2015)
P.D. No. 968, as amended - Santos To v.
Paño, 120 SCRA8 - People vs. Evangelista,
253 SCRA 714
D. Chapter Five: Execution and Service of Penalties
Arts. 78 – 80 - R.A. No. 9344, as amended by R.A. No.
10630 – Declarador vs. Gubaton, G.R No.59208
Padua vs. People, G.R. No. 168546 –
People vs. Sarcia, G.R. No. 169641 –
Reminedo vs. People, G.R. No. 184874 –

People vs. Jacinto, G.R. No. 182239


G.R. No. 182239, March 16, 2011
PEOPLE OF THE PHILIPPINES VS. HERMIE M. JACINTO,

Facts:

Appellant Hermie Jacinto was found guilty beyond reasonable doubt for the rape of the then 5-
year-old victim. The crime was committed when appellant was only 17; Judgment was rendered
when appellant was already 25.

Issue:
Whether or not, appellant may benefit from the provisions of RA9344 regarding criminal liability
of an accused who was a minor during the commission of the crime and the suspension of
sentence of one who is no longer a minor during the pronouncement of verdict.

Held:

The Court sustained the conviction of the appellant in view of the straightforward testimony of
the victim and the inconsistencies of the testimonies of the defense witnesses.

The Court did not exempt accused of his criminal liability although he was only 17 during the
commission of the crime since, in view of the circumstances to which accused committed the
felony, it was proved that he acted with discernment. (Sec 6, RA 9344). There was showing that
the accused understood the consequences of his action.

Applying, the provision of RA 9346, the accused was meted with reclusion perpetua instead of
the death penalty.

As to the civil liability of accused, his minority also had no bearing to the decision of the Court,
ordering accused to pay the victim for damages.

However, the Court afforded the accused the benefit of the suspension of his sentence provided
in Section38 of RA 9344, which made no distinction to an accused found guilty of a capital
offense. The Court stated that what was important was the intent of the Act to uphold the
welfare of a child in conflict with the law. What was to be considered was the fact that accused
committed the crime at a tender age.
The Court held that accused may be confined in an agricultural camp or any training facility in
accordance with Sec 51 of RA 9344. The case was remanded to the court of origin to take
appropriate action in accordance to the said provision.

People vs. Salcedo, G.R. No. 186523 –


People vs Ancajas, G.R. No. 199270 –
People vs Deliola, G.R. No. 200157
Arts. 81 – 88 - R.A. No. 9346 - People vs. de
Jesus, 80 Phil. 749 – p. 515
VI. Title Four: Extinction of Criminal Liability
A. Chapter One: Total Extinction
Arts. 89 – 93
- Act no. 3326, as amended by Act No. 3763 - Sec.
11, R.A. 3019, as amended - Proclamation No. 138,
August 11, 1987
Death pending appeal of judgment of conviction extinguishes his criminal liability,
as well as his civil liability ex delicto - People v. Amistoso, 704 SCRA 369 - People
v. Culas, G.R. No. 211166, June 5, 2017
Rules on survival of civil liability - Datu vs. People,
637 SCRA 754 – p. 578
A conditional pardon does not operate to defeat the consideration of
recidivism as an aggravating circumstance. - U.S. vs. Sotelo, 28 Phil. 147
Absolute pardon extinguishes all effects of the crime and it cannot be
considered as an aggravating circumstance. - U.S. vs. Francisco, 10 Phil. 185
Marriage by the offender with the offended party must be made with the intention
of making her his wife. - People vs. Santiago, 51 Phil. 68 - Art. 266-C
A Sunday or a legal holiday is not a legal efficient cause to interrupt the
prescription of a felony. -Yapdiangco vs. Buencamino, 122 SCRA 713
Reason for the 2-month prescriptive period for slight slander. - People vs.
Maceda, 73 Phil. 679
Until when a conditional pardon becomes operative. - Infante vs.
Warden, 92 Phil. 310
B. Chapter Two: Partial Extinction
Arts. 94 – 99 - Art.
29 RPC
Violation of conditional pardon - Director of Prisons v. Ang Cho
Kio, 33 SCRA 494 (1970)
Crediting of good conduct allowance vested on the Director of Prisons not on the
warden. - People vs. Tan, 19 SCRA 433 - R.A. No. 10592
Art. 103

- Carpio vs Doroja, G.R. No. 84516, December 5, 1989


DIONISIO CARPIO v. SERGIO DOROJA, GR No. 84516, 1989-12-05

Facts:

accused-respondent Edwin Ramirez, while driving a passenger Fuso Jitney owned and
operated by Eduardo Toribio, bumped Dionisio Carpio, a pedestrian crossing the street, as a
consequence of which the latter... suffered from a fractured left clavicle... and sustained injuries
which required medical attention for a period of (3) three months.
An information for Reckless Imprudence Resulting to Serious Physical Injuries was filed against
Edwin Ramirez... the accused voluntarily pleaded guilty... to a lesser offense and was
accordingly convicted for Reckless Imprudence Resulting to Less Serious Physical Injuries
At the early stage of the trial, the private prosecutor manifested his desire to present evidence to
establish the civil liability of either the accused driver or the owner-operator of the vehicle.
Accused's counsel moved that the... court summon the owner of the vehicle to afford the latter a
day in court, on the ground that the accused is not only indigent but also jobless and thus
cannot answer any civil liability that may be imposed upon him by the court.
The private prosecutor,... however, did not move for the appearance of Eduardo Toribio.
Thereafter, a writ of execution... was duly served upon the accused but was, however, returned
unsatisfied due to the insolvency of the accused as shown by the sheriff's return. Thus,
complainant moved for a subsidiary writ of execution... against the subsidiary liability of the
owner-operator of the vehicle. The same was denied by the trial court on two grounds, namely,
the decision of the appellate court made no mention of the subsidiary liability of Eduardo
Toribio, and the... nature of the accident falls under "culpa-aquiliana" and not "culpa-
contractual."
Issues:
whether or not the subsidiary liability of the owner-operator may be enforced in the same
criminal proceeding against the driver where the award was given, or in a separate civil action
Ruling:
The law involved in the instant case is Article 103 in relation to Article 100, both of the Revised
Penal Code, which reads
The subsidiary liability established in the next preceding article shall apply to employers,
teachers, persons, and corporations engaged in any kind of industry for felonies... committed by
their servants, pupils, workmen, apprentices, or employees in the discharge of their duties.
The subsidiary liability in Art. 103 should be distinguished from the primary liability of employers,
which is quasi-delictual in character as provided in Art.
2180 of the New Civil Code. Under Art. 103, the liability emanated from a delict. On the other
hand, the liability under Art. 2180 is founded on culpa... aquiliana. The present case is neither
an action for culpa?contractual nor for culpa-aquiliana. This is basically an action to enforce the
civil liability arising from crime under Art. 100 of the Revised
Penal Code. In no case can this be regarded as a civil action for the primary liability of the
employer under Art. 2180 of the New Civil Code, i.e., action for culpa aquiliana.
In order that an employer may be held subsidiarily liable for the employee's civil liability in the
criminal action, it should be shown (1) that the employer, etc. is engaged in any kind of industry,
(2) that the employee committed the... offense in the discharge of his duties and (3) that he is
insolvent
The subsidiary liability of the employer, however, arises only after conviction of the employee in
the criminal... action. All these requisites present, the employer becomes ipso facto subsidiarily
liable upon the employee's conviction and upon proof of the latter's insolvency. Needless to
say, the case at bar satisfies all... these requirements.
Considering the subsidiary liability imposed upon the employer by law, he is in substance and in
effect a party to the criminal case. Ergo, the employer's subsidiary... liability may be determined
and enforced in the criminal case as part of the execution proceedings against the employee.
the filing of a separate complaint against the operator for recovery of subsidiary liability is not
necessary since his liability is clear from the decision against the... accused. Such being the
case, it is not indispensable for the question of subsidiary liability to be passed upon by the
appellate court. Such subsidiary liability is already implied from the appellate court's decision.
The Court a quo is directed to hear and decide... in the same proceeding the subsidiary liability
of the alleged owner/operator of the passenger jitney.

- Yonoha v Court of Appeals, GR No 112346, March 29, 1996


- Pangonorom vs. People, GR No. 143380, April 11, 2005
- Philippine Rabbit Bus Lines vs. Heirs of Mangawang G.R.No. 160355
- LG Foods Corporation vs. Agraviador, G.R. No. 158995
- Safeguard Security Agency (SSA) vs Tangco, G.R. No. 165732

You might also like